Anda di halaman 1dari 57

A.

Searches and Seizures (Sec. 2, Art. III) 1. Scope of Protection CASES: Moncado v. Peoples Court, 80 Phil 1

(d) The personal file and the love letters of Mrs. Moncado to Dr. Moncado and vice versa; (e) Marriage certificate of Dr. Moncado with Mrs. Moncado issued at Reno, Nevada; (f) Private correspondence and letters of Dr. Moncado to and from his Filipino Federation of America in Hawaii and United States: (g) Several law books by Guevara, Albert, Francisco, Harvard Classics (complete set), books on diplomacy, international law; (h) A complete collection of the 'Tribunal' compilation of the same during occupation until the last day of its issuance; (i) Complete collection of American magazines, from 1940 to 1941 Los Angeles Examiner, San Francisco Chronicle, Los Angeles Evening Herald and newspapers edited and owned by Dr. Moncado and published in the United States; and National Geographic Society; (j) Personal letters of Dr. Moncado with several members of the United States Senate and Congress of the United States including a picture of President Hoover dedicated to Dr. Moncado; (k) Pictures with personal dedication and autograph to Dr. and Mrs. Moncado by actors and actresses from Hollywood, including Mary Astor, Binnie Barnes, Robert Montgomery, Clark Gable, Gary Cooper, Boris Karloff, Wallace Beery, William and Dick Powell, Myrna Loy, Bette Davis and Ceasar Romero; (l) Certificate as first flighter in the Pan-American Airways and even several stickers issued by Pan American Airways for passengers' baggage; (m) A promissory note of Dr. Moncado for fifty thousand pesos (P50,000) in favor of Architect Mr. Igmidio A. Marquez of Quezon City; (n) Three (3) volumes of modern ballroom dancing by Arthur MacMurray of New York, pamphlets of dancing obtained by Dr. Moncado while he was studying dancing at Waldorf-Astoria, New York; (o) two (2) volumes of rhumba, zamba and tango obtained from Mexico and Argentina by Dr. Moncado." (Pages 3 and 4, Petition for Certiorari and Injunction.) On June 27, 1946, petitioner filed with the People's Court a motion praying that the return of said documents and things be ordered. The petition was denied on July 9, 1946. Thereupon, petitioner filed with this Supreme Court on August 10, 1946, a petition praying that the lower court's order of July 9, 1946, be set aside, that said court be required to order the return of the documents and things in question to petitioner, and that the prosecutor be restrained from using and presenting them as evidence at the trial of the criminal case for treason. Before proceeding to consider the question of law raised in this case, we should not ignore three questions of fact raised in the answers of respondents: at to the identity of the documents and things, as to whether they were taken from the house at San Rafael or from the house at Rosario Heights, and as to whether they were taken at the time of petitioner's arrest or later. The fact that the return of the documents and things were opposed to in the lower court by the prosecutor, without disputing their identity, and that in the present proceeding the prosecutor admits

Separate Opinions HILADO, J., concurring: I concur, but I would further support the conclusion arrive at by the following additional considerations: In April, 1945, when the CIC Detachment of the United States Army made the search at petitioner's house and effect the seizure of his papers and effects mentioned in the majority decision, as is of general knowledge and within the judicial notice of this Court, fighting continued in Luzon; in fact, as late as June, 1945, the cannonades and shellings could still be clearly heard in this City of Manila, and there were still units of the Japanese Army resisting the liberation forces. Under such circumstances, the war was continuing not only technically but actuallyin the island of Luzon; and the military security and safety of the liberation forces demanded such measures as were adopted by the CIC Detachment of the United States Army in making said search and effecting said seizure to the end that the activities of pro-Japanese elements and their chances of effectively aiding the Japanese forces which thus still continued to resist might be brought down to a minimum and, if possible, entirely foiled. The difference between this case and the case in L-342, (Alvero vs. Dizon, 43 Off. Gaz., 429), is, to my mind, merely one of degree the principle involved is identical in both cases.

PERFECTO, J., dissenting: Petitioner stands accused of treason before the People's Court, the information against him having been filed by Prosecutor Juan M. Ladaw on February 28, 1946. Almost a year before, on April 4, 1945, at about 6:00 p.m., petitioner was arrested by members of the Counter Intelligence Corps of the United States Army at his residence at 199-A San Rafael St., Manila, without any warrant of arrest, and taken to the Bilibid Prison at Muntinglupa, where he was detained. On April 11, 1945, petitioner's wife, who transferred to their house at 3 Rosario Drive, Quezon City, was approached by several CIC officers, headed by Lt. Olves, and ordered to accompany them to the house at San Rafael to witness the taking of documents and things belonging to petitioner. Upon hearing from the officers that they did not have any search warrant for the purpose, she refused to go with them, but after the officers told her that with or without her presence they would search the house at San Rafael, Mrs. Moncado decide to accompany them. Upon arrival at the house, Mrs. Moncado noticed that their belongings had been ransacked by American officers and that the trunks which she had kept in the attic and in the garage when she left the house, had been ripped open and their contents scattered on the floor. Lt. Olves informed Mrs. Moncado that they were going to take a bundle of documents and things, which were separated from the rest of the scattered things, because they proved the guilt of her husband. Mrs. Moncado protested in vain. No receipt was issued to her. Subsequently, after making an inventory of their belongings at San Rafael, Mrs. Moncado found the following things missing: (a) Passes issued by Japanese friends for the personal safety and conduct of the petitioners; (b) Correspondences of the petitioner as president of the Neighborhood Association in Quezon City during the Japanese occupation; (c) Correspondence of the petitioner with certain Japanese officers;

to have them in his possession, without disputing their identity or correcting any error of description made by petitioner, convinced us that in petitioner's and respondent's minds there is no disagreement on the identity in question. There should not be any doubt that the papers and things described and claims by petitioner are the ones in the prosecutor's possession, otherwise, instead of objecting to the return on legal grounds, he would have alleged that such things are not in his possession, or he does not know where they are, or that they did not exist at all. Whether the things were taken at San Rafael or at Rosario Heights is completely immaterial. The fact is that is that the reality and existence of things and petitioners' ownership thereof, are undisputed, and that they were taken from a house of petitioner. That they were taken not at the time of petitioner's arrest but much later, is indisputably proved by petitioner's and his wife's depositions not contradicted by any other evidence. This case offers a conclusive evidence that fundamental ideas, rules and principles are in constant need of restatement if they are not to lose their vitality. So that they may continue radiating the sparks of their truth and virtue, they need the repeated pounding of intense discussion, as the metal hammered on the anvil. To make them glow with all their force, purity and splendor, they need the continuous smelting analysis and synthesis as the molten iron in a Bessemer furnace. Otherwise, they become rusty, decayed or relegated as useless scraps in the dumping ground of oblivion. What is worse, they are frequently replaced by their antitheses, which pose with the deceitful dazzle of false gods, clothed in tinsel and cellophane. The risks always lurking at every turn of human life, exacts continuous vigilance. Human minds must always be kept well tempered and sharpened as damask swords, ready to decapitate the hydra of error and overthrow the gilded idols from the muddy pedestals of pretense and imposture. May the government profit from an illegality, an unconstitutional act, or even a crime to serve its aims, including the loftiest? May justice be administered by making use of the fruits of a lawless action? If a private individual, when profiting from the fruits of a criminal offense, is punished by law as an accessory after the fact, why should the government or an official system of justice be allowed to ignore and mock the moral principle which condemns the individual? Is there a moral standard for the government different from the one by which private conduct is measured? While a private citizen is not allowed to violate any rule of decency and fair play, may the government follow a procedure which shocks the common sense of decency and fair play? If a person cannot enrich himself with stolen property, why should a government be allowed to profit and make use of property tainted by theft or robbery or smeared with the blood of crime? The above are among the elemental questions that must be answered in this case, if we are not lacking the moral courage to face all the issues raised by the parties. Other questions concern personal liberty as affected by illegal detention, personal security against illegal searches and seizures, judicial emancipation from colonial mental attitude. Respondents urge us to follow the decision in Alvero vs. Dizon (L342), which, besides having been rendered by a second Supreme Court, whose existence is violative of the Constitution, cannot claim better merit than a servile adherence to a wrong legal doctrine, decorated by the halo of authority of courts of a former metropolis. There are minds that forget that duty of thinking by ourselves and of not sticking to the teachings of foreign mentors has become more imperative since July 4, 1946. The seizure of the papers and effects in questions, having been made without any search warrant, was and is illegal, and was effected in open violation of the following provisions of the Constitution: The right of the people to be secure in their persons, houses, papers, and effects against unreasonable searches and seizures shall not be violated, and no warrants shall

issue but upon probable cause, to be determine by the judge after examination under oath or affirmation of the complainant and the witnesses he may produce, and particularly describing the place to be searched, and the persons or things to be seized. (Article III, section 1 [3] of the Constitution.) The privacy of communication and correspondence shall be inviolable except upon lawful order of the court or when public safety and order require otherwise. (Article III, section 1 [5] of the Constitution.) The seizure was also in open violation of sections 3, 10, and 11 of Rule 122, which are as follows: SEC. 3. Requisites for issuing search warrant. A search warrant shall not issue but upon probable cause to be determined by the judge or justice of the peace after examination under oath or affirmation of the complainant and the witnesses he may produce, and particularly describing the place to be searched, and the persons or things to be seized. SEC. 10. Receipt for the property seized. The officer seizing property under the warrant must give a detailed receipt for the same to the person on whom or in whose possession it was found, or in the absence of any person, must, in the presence of at least two witnesses, leave a receipt in the place in which he found the seized property. SEC. 11. Delivery of property and inventory there of to court. The officer must forthwith deliver the property to the justice of the peace or judge of the municipal court or of the Court of First Instance which issue the warrant, together with a true inventory thereof duly verified by oath. Even more, the illegality and unconstitutionality amounted to two criminal offenses, one of them heavily punished with prision correccional. The offenses are punished by articles 128 and 130 of the Revised Penal Code, which reads: 4. ART. 128. Violation of domicile. The penalty of prision correccional in its minimum period shall be imposed upon any public officer or employee who, not being authorized by judicial order, shall enter any dwelling against the will of the owner thereof, search papers or other effects found therein without the previous consent of such owner, or, having surreptitiously entered said dwelling, and being required to leave the premises, shall refuse to do so. If the offense be committed in the nighttime, or if any papers or effects not constituting evidence of a crime be not returned immediately after the search made by the offender, the penalty shall be prision correccional in its medium and maximum periods. ART. 130. Searching domicile without witnesses. The penalty of arresto mayor in its medium and maximum periods shall be imposed upon a public officer or employee who, in cases where a search is proper, shall search the domicile, papers or other belongings of any persons, in the absence of the latter, any member of his family, or in their default, without the presence of two witnesses residing in the same locality. The main authority upon which respondents rely is the decision of the Supreme Court of the United States in Bordeau vs. MacDowell (256 U.S., 465), the same followed in the decision in Alvero vs. Dizon (L-342). In the Bordeau case, certain documents were stolen from MacDowell. Upon finding that the documents contained evidence of the fraudulent use of the mails by MacDowell, the robbers delivered them to Bordeau, in charge of the prosecution against MacDowell. The latter filed a motion to prevent Bordeau from using the documents as evidence against him. The federal

Supreme Court denied the motion on the ground that there is no law or constitutional principle requiring the government to surrender papers which may have come into its possession where the government has not violated the constitutional rights of the petitioner. Two of the greatest American Justices, Justices Holmes and Brandeis, whose dissenting opinions, written twenty years ago, are now the guiding beacons of the Supreme Court of the United States, dissented, the latter saying: At the foundation of our civil liberty lies the principles which denies to government officials exceptional position before the law, and which subjects them to the same rules of conduct that commands to the citizen. And in the development of our liberty insistence upon procedural regularity has been a large factor. Respect for law will not be advanced by resort, in its enforcement, to means which shock the common man's sense of decency and fair play. Taking aside the great intellectual, moral and judicial prestige of the two dissenters, the poignant logic and rock-bottom sense of truth expressed by Justice Brandeis is enough to complete discredit the majority doctrine in the Bordeau case, a doctrine that in principle and by its evil effects appears to be irretrievably immoral. To merit respect and obedience, a government must be just. Justice cannot exist where the good is not distinguished from the wicked. To be just, the government must be good. to be good it must stick to the principles of decency and fair play as they are understood by a common man's sense, by universal conscience. Good ends do not justify foul means. No one should profit from crime. Principles are not to be sacrificed by any purpose. What is bad per se cannot be good because it is done to attain a good object. No wrong is atoned by good intention. These are some of the maxims through which the common sense of decency and fair play is manifested. Reason is a fundamental characteristic of man. there is no greater miracle than when its first sparks scintillated in the mind of a child. What before had only the vegetative life of a plant or the animal life of a mollusk or frog, suddenly begins to wield the prodigious power of understanding and intelligent grasping of the meaning and relations of the things with which he is in direct or remote contact though his senses. The power of understanding brings forth the freedom of choice. This freedom develops the faculty of discrimination between good and evil. That discrimination is further developed into a sense of justice. While the advent of the astounding miracle of reason has so much kindled the pride of men, to the extent of symbolizing it wit the fire stolen by Prometeus from the heavens, and of proclaiming himself as the king of the creation, man had taken millennia of struggles in order to develop the basic ideas which insure his survival and allow him to enjoy the greatest measure of well-being and happiness. He soon discovered that society is an indispensable condition to attain his ends. As a consequence, he fought against all anti-social ideas and conduct and had to discover or invent and then develop the principles and qualities of sociability. The struggle has been long and it will have to continue until the end of the centuries. It is the same eternal struggle between truth and error, between right and wrong. While man, in the multifarious ensemble of the universe, seems to be the lone and exclusive holder of the divine fire of reason, he had so far failed to find the key to always correct thinking. The solution to the failure of reason is riddle yet to be unlocked. Man is easily deceived into committing blunders or led into the most absurd aberrations. The mysterious genes which keep uninterrupted the chain of heredity, while permitting the transmission of the best qualities and characteristics, seems to lack the power of checking and staving off the tendencies of atavism. In the moral ctetology, either kind of characteristics and qualities may be originated and developed. The inconsistency of respondents is thus explainable. While they would raise their brows at the mere insinuation that a private individual may justifiably profit by the result s of fruits of a criminal offense, they would not measure the government with the same moral standard. That the inconsistency may be explained by its genesis is no ground why we should surrender to it. To set two moral standard, a strict one for private individuals and another

vitiated with laxity for the government, is to throw society into the abyss of legal ataxia. Anarchy and chaos will become inevitable. Such a double standard will necessarily be nomoctonous. The idea of double moral standard is incompatible with the temper and idiosyncracy of social order established by our constitution, and is repugnant to its provisions. all government authority emanates for the people in whom sovereignty resides. The Filipino people ordained and promulgated the constitution "in order to established a government that shall embody their ideals." Among these ideals are justice, democracy, the promotion of social justice equal protection of the laws to everybody. Such ideals are trampled down by the adoption of the double moral standard which can be taken its place in the ideology of the supporters of absolute monarchies. There is the maxim that "the king can do no wrong." The iniquities and misery havocked by such maxim would need hundreds or thousands of volumes to record them. The infamy of Japanese occupation gave our people the bitter taste of the operation of the double moral standard. It is the antithesis of the golden rule. It would place government in a category wholly apart from humanity, notwithstanding its being a human institution, an unredeemable absurdity. From "Brandeis, A Free Man's Life" by Alpheus Thomas Mason (pp. 568 and 569), we quote an analogous legal situation: "In the famous wire-tapping case Chief Justice Taft, delivering the opinion, overruled the defendants' claim that the evidence obtained when government agents tapped their telephone wires violated either unreasonable searches and seizures or the constitutional protection against self-incrimination. No tapped wires entered their homes and offices, Taft reasoned, so there was neither search nor seizure. "For Justice Brandeis such a narrow construction degraded our great charter of freedom to the level of a municipal ordinance. Quoting Chief Justice Marshall's famous 'We must never forget that it is a Constitution we are expounding' he pointed out that just as the power of Congress had by judicial interpretation been kept abreast of scientific progress, and extended the Fundamental Law to objects of which the Founding Fathers never dreamed, so also must the judges in construing limitations on the powers of Congress be ever mindful of changes brought about by discovery and invention. To have a living Constitution, limitations on power no less than grants of power must be construed broadly. 'Subtler and more far-reaching means of invading privacy have become available to the government,' Brandies observed. ... The progress of science in furnishing the government with means of espionage is not likely to stop with wire-tapping. Ways may someday be developed by which the government, without removing papers from secret drawers, can reproduce them in court, and by which it will be enabled to expose to a jury the most intimate occurrences of the home. . . . "'Our government is the potent, the omnipresent teacher. For good or ill, it teaches the whole people by example. Crime is contagious. If government becomes a law-breaker, it breeds contempt for law; it invites every man to become a law unto him; it invites anarchy. To declare that in the administration of the criminal law the end justifies the means to declare that the government may commit crimes in order to secure the conviction of a private criminal would bring terrible retribution. . . . "'The makers of our Constitution undertook to secure conditions favorable to the pursuit of happiness,' he emphasized.They recognized the significance of man's spiritual nature, of his feelings and of his intellect. they knew that only a part of the pain, pleasure, and satisfactions of life are to be found in material things. They sought to protect Americans in their beliefs, their thoughts, their emotions, and their sensations. They conferred, as against the government, the right to be let alone the most comprehensive of right and the right most valued by civilized men. . . . "'Experience should teach us to be most on our guard to protect liberty when the government's purpose are beneficent. Me born to freedom are naturally alert to repel invasion of their liberty by evilminded rulers. The greatest dangers to liberty lurk in insidious

encroachment by men of zeal, well-meaning, but without understanding.'" (Olmstead vs. U.S., 277 [U.S.] 438, [1928], pp. 473474, 478, 479, 485.) The argument that goods and personal properties illegally taken, stolen, or snatched from the owner of possessor without a duly issued search warrant can be retained by the prosecution for use as evidence in a criminal case instituted is initiated by an original and basic flaw. The argument rests on the assume existence or commission of a crime as its minor premise. but, under the orderly processes of law, the assumption has yet to be proved, and it is impossible to be proved before it can be of any use to support and clinch the argument. The prosecution is called upon to make the assumption that the goods and properties in question are evidence of a crime. To be valid, the assumption has to presuppose the commission or existence of the crime. That presupposition, in order to be valid, must in turn stand on an authoritative pronouncement which can only be made in a final and executory decision rendered by a court of justice. The prosecution cannot make a conclusive pronouncement, as to the existence or commission of a crime, the basic fact which, under the argument, will entitled the prosecution to retain and use the goods and properties in question. The argument assumes a fact the existence of which still remains to be proved and continues to be enveloped in the mists of the realm of uncertainties, which fact may lead to the disputed right of the prosecution to retain the goods and properties illegally seized as essential evidence of the crime. The line of reasoning the build up the argument can be restated in more abstract terms as follows: justify the means by their necessity to attain an end by starting from the premise that the end was accomplished. Such a reasoning process is fundamentally subversive to logic and is incompatible with the natural workings of the human mind. The rules governing the phenomena of diffusion and osmosis, of permeability and isotonic equilibrium, of assimilation and waste dislodgment, of development and reproduction, like all laws of life, are uniform and universal. Whether in the nuclear chromatin or the cytosome of a single protoplasmic cell of amoeba or in the sinews of the heaviest marsupial, whether in the formation of the smallest bud or in the formation of the smallest bud or in the display of color and aroma by the most beautiful flower, whether in the development of a frog or in the attainment of the perfect curves and velvety skin of a lovely girl, the uniformity and universality of biological laws are manifested unrelentlessly. Any disregard of them is fatal, and will lead to irretrievable disaster and destruction. Moral standards are the laws of social life. In a different plane and order, they are but biological laws, governing the vital processes and functions of social organism. They are and should be uniform and universal and no single unit or organ of human society can disregard them or any one of them without alluring catastrophic consequences. Our decision is to grant all the prayers of the petition, and it was so ever since February 24, 1947, when this Court took the vote for the disposal of this case. In stating this fact we do not want to put any blame on the distinguished member who penned the decision now to be promulgated. In justice to him, we may record that the drafting of the majority decision was transferred and entrusted to him many months after a final vote had been taken on the case, and it did not take him more than a month to have ready the majority opinion. In exposing the fact we mean only to emphasize the crying need of changing a situation or a system of procedure that permits the promulgation of our decisions one year or more after a case has been submitted to us for final action. It is only part of the crusade to curtail judicial delay which we felt our duty to engaged in since it had been our privilege to sit in the Supreme Court, whose vantage in the legal field imposes upon the members thereof the role of leadership in legal thought and practice for the most effective administration of justice. BENGZON, J., dissenting: Sanctity of the home is a by-word anywhere, anytime. The house of man was the first house of God.

In Rome the citizen's dwelling was a safe asylum. Invasion thereof was anathema. Down through the centuries respect for men's abodes has remained a heritage of civilization. In England, the poorest man could in his cottage, defy all the forces of the Crown. "It may be frail; it is roof may shake; the wind may blow through it, the storm may enter; the rain may be enter; but the King England may not enter; all his forces dare not cross the threshold of the ruined tenement." His home was indeed his castle. And in the United States: "The right of the citizen to occupy and enjoy his home, however mean or humble, free from arbitrary invasion and search, has for centuries been protected with the most solicitous care. . . . "The mere fact that a man is an officer, whether of high or low degree, gives him no more right than is possessed by the ordinary private citizen to break in upon the privacy of a home and subject its occupants to the indignity of a search for the evidence of crime, without a legal warrant procured for that purpose. No amount of incriminating evidence, whatever its source, will supply the place of such warrant. At the closed door of the home, be it place or hovel, even bloodhounds must wait till the law, by authoritative process, bids it open. . . . " (McLurg vs. Brenton, 123 Iowa, 368, quoted in 20 Phil., 473.) Logical culmination and practical application of the above principles embodied in our Organic Laws, is the ruling we announced in Alvarez vs. Court of First Instance of Tayabas, 64 Phil., 33, that documents unlawfully seized in a man's home must be returned irrespective of their evidentiary value provided seasonable motions are submitted. We followed the Federal rule in Boyd vs. U.S, 116 U.S., 616 and many others. We had said before that "it is better oftentimes that crimes should be unpunished than that the citizen should be liable to have his premises invaded, his desk broken open, his private books, letters, and papers exposed to prying curiosity, ... under the direction of a mere ministerial officer" ... insensitive perhaps to the rights and feelings of others. (U.S. vs. De los Reyes and Esguerra, 20 Phil., 472, citing Cooley, Constitutional Limitations.) In the Alvarez decision we reflected that "of all rights of a citizen few are of greater importance or more essential to his peace and happiness than the right of personal security, and that involves the exemption of his private affairs, books, and papers from the inspection and scrutiny of others," and while the power to search and seize is necessary to public welfare, still it mist be exercised without transgressing the constitutional rights of citizens, because the enforcement of statutes is never sufficiently important to justify violation of the basic principles of government. It is agreed that the fundamental rights of the individual guaranteed by the Constitution, must be given such a liberal construction of strict construction as will be in his favor, to prevent gradual encroachment or stealthy depreciation of such fundamental rights. (State vs. Custer County, 198 Pac., 362; State vs. McDaniel, 231 Pac., 965; 237 Pac., 373.) Our constitution in its Bill of Rights decrees that "the right of the people to be secure in their persons, houses, papers, and effects against unreasonable searches and seizures shall not be violated, and no warrants shall issue but upon probable cause, to be determined by the judge after examination under oath or affirmation of the complainant and the witnesses he may produce, and particularly describing the place to be searched, and the persons or things to be seized. (Constitution, Article III, section 1 [3].) This is an improvement over the provisions of the Jones Law regarding warrants and seizures. It was designed to make our Constitution "conform entirely" to the Fourth Amendment of the U.S. Constitution. (Aruego, Framing of the Philippine Constitution, Vol., II, p. 1043.) The split between several State Supreme Courts on one side and the Federal Supreme Court on the other, about the admissibility of evidence obtained through illegal searches and seizures, was familiar to this Court (People vs. Carlos, 47 Phil., 626, 630) before it voted to adopt the Federal doctrine in Alvarez vs. Court of First Instance of Tayabas, supra.

This last doctrine, applied in several subsequent cases (People vs. Sy Juco, 64 Phil., 667; Rodriguez vs. Villamil, 37 Off. Gaz., 2416) was probably known to the Constitutional Convention that, the addition, made the Constitutional mandate on the point more complete and explicit, copying exactly the wording of the Federal Constitution, a circumstance which, coupled with the citation of Boyd vs. U.S., showed adherence to the Federal doctrine that debars evidence obtained by illegal search or unlawful seizure. It is significant that the Convention readily adopted the recommendation of the Committee on Bill of Rights after its Chairman had spoken, explaining the meaning and extent of the provision on searches and seizures and specifically invoking the United States decisions of Boyd vs. U.S., 116 U.S., 616 and Gould vs. U.S., 225 U.S., 298, which the majority of this Court would now discard and overrule. (Aruego op. cit. Vol. I, P. 160; Vol., II, pp. 1043, 1044.) Therefore, it is submitted, with tall due respect, that we are not at liberty now to select between two conflicting theories. The selection has been made by the Constitutional Convention when it impliedly chose to abide by the Federal decisions, upholding to the limit the inviolability of man's domicile. Home! The tie that binds, the affection that gives life, the pause that soothes, all nestle there in an atmosphere of security. Remove that security and you destroy the home. Under this new ruling the "King's forces" may now "cross the threshold of the ruined tenement" seize the skeleton from the family closet and rattle it in public, in court, to the vexation or shame of the unhappy occupants. That those forces may be jailed for trespass, is little consolation. That those forces may be pardoned by the King, their master suggests fearful possibilities. The sanctuary, the castle, are gone with the wind. An opinion of Mr. Justice Cardozo in the Court of Appeals of New York is cited as authority for the majority view (People vs. Defore, 150 N.E., 585). Yet it is markworthy that, in New York, protection against unreasonable searches and seizures is not promised by the Constitution of the State but by a mere statute. (Civil Rights of Law.) (See the same case, and 56 C.J., p. 1156.) New York is the only state that denies this privilege the status of a constitutional prerogative. (Supra.) Hence the precedent is obviously inconclusive. Moreover, admitting, for purposes of argument only, that the Alvarez decision is legally erroneous, I maintain that the new doctrine should apply to future cases not to herein petitioner who had relied on it. In Santiago and Flores vs. Valenzuela, No L-670, April 30, 1947 (44 Off. Gaz., 3291, 3296) I argued for that proposition as follows: . . . The reserved right to upset previous decisions is likewise qualified by the proposition that such upsetting shall have prospective not retroactive effect. In Douglass vs. Pike Country, 101 U.S. 677 at p. 687, it was declared, "The true rule (of stare decisis) is to give a change of judicial construction ... the same effect in its operation" ... as to "a legislative amendment,i.e., make it prospective but not retroactive." And in Great Northern R. Co. vs. Sunburst Oil & Ref. Co., 287 U.S., 358, the Supreme Court, through Mr. Justice Cardozo, said: "A state in defining the limits of adherence to precedent may make a choice for itself between the principle of forward operation and that of relation backward. It may say that decisions of its highest court, though later overruled, are law none the less for intermediate transactions. Indeed there are cases intimating, too broadly (cf. Tidal Oil Co. vs. Flanagan, 263 U.S., 444; 68 Law. ed., 382; 44 S Ct., 197, supra), that it must give them that effect; but never has doubt been expressed that it may so treat them if it pleases, whenever injustice or hardship will thereby be averted Gelpcke vs. Dubuque, 1 b.

Wall., 175; 17 Law. ed., 25; Douglassvs. Pike Country, 101 U. S 677, 687; 25 Law. ed., 968, 971; Loeb vs. Columbia Twp. 179 U. S., 472, 492; 45 Law, ed., 280, 290, 21 S. Ct., 174, etc." This view is not unanimous, I know. However, inasmuch as one of the principal arguments of the opposing school of thought is that it makes the overruling decision a mere "declaratory judgment", and since that objection is untenable in this jurisdiction where declaratory relief is permitted (Rule 66), the view herein advocated future operation only future operation only should all the more be acceptable to our system of jurisprudence. More about this in the future, if I should happen to agree to an overruling of previous decisions and the question should hinge on its backward or forward application. For the present, enough to note some of the abundant literature 1 on the point. Stonehill s. Diokno, 20 SCRA 383 Facts: Respondents issued, on different dates, 42 search warrants against petitioners personally, and/or corporations for which they are officers directing peace officers to search the persons of petitioners and premises of their offices, warehouses and/or residences to search for personal properties books of accounts, financial records, vouchers, correspondence, receipts, ledgers, journals, portfolios, credit journals, typewriters, and other documents showing all business transactions including disbursement receipts, balance sheets and profit and loss statements and Bobbins(cigarettes) as the subject of the offense for violations of Central Bank Act, Tariff and Customs Laws, Internal Revenue Code, and Revised Penal Code. Upon effecting the search in the offices of the aforementioned corporations and on the respective residences of the petitioners, there seized documents, papers, money and other records. Petitioners then were subjected to deportation proceedings and were constrained to question the legality of the searches and seizures as well as the admissibility of those seized as evidence against them. On March 20, 1962, the SC issued a writ of preliminary injunction and partially lifted the same on June 29, 1962 with respect to some documents and papers. Held: a. Search warrants issued were violative of the Constitution and the Rules, thus, illegal or being general warrants. There is no probable cause and warrant did not particularly specify the things to be seized. The purpose of the requirement is to avoid placing the sanctity of the domicile and the privacy of communication and correspondence at the mercy of the whims, caprice or passion of peace officers. Document seized from an illegal search warrant is not admissible in court as a fruit of a poisonous tee. However, they could not be returned, except if warranted by the circumstances. c. Petitioners were not the proper party to question the validity and return of those taken from the corporations for which they acted as officers as they are treated as personality different from that of the corporation.

Alih v. Castro, 151 SCRA 279 Facts; Zona was conducted by the men of Maj. Gen Castro in a compound

where petioners reside and conducted illegal search and thereafter seized guns from them. The order was carried on by his Castro's men with the contention that the petitioners are involved in the latest killing of the town's mayor Cesar Climaco. Issue; Is Held; The Supreme Court declared those seized in custodia legis and declared that the operation conducted by Maj. Gen. Castro was ILLEGAL. The respondents have all the time to obtain a search warrant granted that they have about 10 trial courts. The SC also held the protection of the petitioner's human rights as stated in Art IV Sec 3 and 4 of the 1973 Constitution regarding illegal search and seizure. The presumption of innocence of the petitioners should be observed and that they cannot be subjected to self-incriminating instances like paraffin tests, photographing and finger printing. As penned by J. Cruz in this case, "The Constitution is a law for rulers and people, equally in war and in peace, and covers with the shield of its protection all classes of men, at all times and under all circumstances. No doctrine, involving more pernicious consequences, was ever invented by the wit of man than that any of its provisions can be suspended during any of the great exigencies of government." 2. Meaning of search CASE: Katz v. United States, 389 U.S. 347 (1967) Facts: the warrantless search and seizure legal?

Rationale:

The Court stated that both parties had formulated the issue incorrectly. Instead of asking whether a telephone booth can characterized as a constitutionally protected area, the proper question is whether electronically listening to and recording Katzs conversation violated his right to privacy.

The Court reasoned that the Fourth Amendment right to privacy protects people, not places. Previous case law held that in order for the right to privacy to be violated, the government had to infringe upon tangible property. In Warden v. Hayden, another 1967 case, the Court discredited the trespass doctrine, which held that property interests no longer controlled the governments right to search and seizures.

In the instant case, the government contended that Katzs use of a see-through booth meant that he was still just as visible inside the booth as outside of it. However, the Court noted that Katz was not trying to get away from the intruding eye, instead, he was fleeing the uninvited ear. His use of a glass phone booth with a closed door did not publicize his conversation, only his appearance. The FBI agents did not have a warrant and thus did not have to confine their investigation within the confines of a warrant. Even though the agents restrained themselves and did not unnecessarily invade Katzs privacy, they solely based their actions on their belief that Katz would return to the same pay phone and the same time every day to make illegal gambling bets. The Court reiterated that the U.S. Constitution requires the impartial judgment of a judicial officer to stand between citizens and the police. Because searches conducted without judicial process are per se unreasonable, this case was REVERSED.

Katz was arrested after FBI agents overheard him making illegal gambling bets while in a public phone booth. The agents placed electronic listening and recording devices to the outside of the booth and only heard and recorded Katzs end of the conversations. Procedural History:

At trial, Katz objected to the introduction of evidence of the telephone conversation. However, the trial court allowed the evidence. The appellate court upheld the conviction, holding that Katzs Fourth Amendment protection from unreasonable searches and seizures was not infringed upon because the agents never physically entered the phone booth. The U.S. Supreme Court reversed the lower court.

Justice HARLAN concurred, stating that Fourth Amendment questions followed a two-part test: (1) whether there is an actual, subjective expectation of privacy; and (2) whether the expectation is objectively reasonable.

Issue:

Justice BLACK dissented, noting that it was not the Courts role to rewrite the Fourth Amendment to remove the limitation to tangible property. He stated that a warrant cannot be obtained regarding a future conversation, as the police would not be able to describe such a conversation in the warrant application.

Did the government violate Katzs Fourth Amendment rights when its agents attached electronic listening and recording devices to the outside of the phone booth?

Argument:

3.

Katz argued that the telephone booth was a constitutionally protected area and the FBI violated his right to privacy by attaching the bugs to the phone booth. The government argued that it did not violate Katzs right to privacy because none of the agents invaded the booth before performing the search and/or seizure. Facts:

Requisites of a Valid Search Warrant Existence of Probable Cause CASES: Burgos v. Chief of Staff, 133 SCRA 800

Holding:

Two warrants were issued against petitioners for the search on the premises of Metropolitan Mail and We Forum newspapers and the seizure of items alleged to have been used in subversive activities. Petitioners prayed that a writ of preliminary mandatory and prohibitory injunction be issued for the return of the seized articles, and that respondents be enjoined from using the articles thus seized as evidence against petitioner.

Yes, the government violated Katzs right to privacy. Reversed.

Petitioners questioned the warrants for the lack of probable cause and that the two warrants issued indicated only one and the same address. In addition, the items seized subject to the warrant were real properties. Issue: Whether or not the two warrants were valid to justify seizure of the items. Held: The defect in the indication of the same address in the two warrants was held by the court as a typographical error and immaterial in view of the correct determination of the place sought to be searched set forth in the application. The purpose and intent to search two distinct premises was evident in the issuance of the two warrant. As to the issue that the items seized were real properties, the court applied the principle in the case of Davao Sawmill Co. v. Castillo, ruling that machinery which is movable by nature becomes immobilized when placed by the owner of the tenement, property or plant, but not so when placed by a tenant, usufructuary, or any other person having only a temporary right, unless such person acted as the agent of the owner. In the case at bar, petitioners did not claim to be the owners of the land and/or building on which the machineries were placed. This being the case, the machineries in question, while in fact bolted to the ground remain movable property susceptible to seizure under a search warrant. However, the Court declared the two warrants null and void. Probable cause for a search is defined as such facts and circumstances which would lead a reasonably discreet and prudent man to believe that an offense has been committed and that the objects sought in connection with the offense are in the place sought to be searched. The Court ruled that the affidavits submitted for the application of the warrant did not satisfy the requirement of probable cause, the statements of the witnesses having been mere generalizations. Furthermore, jurisprudence tells of the prohibition on the issuance of general warrants. (Stanford vs. State of Texas). The description and enumeration in the warrant of the items to be searched and seized did not indicate with specification the subversive nature of the said items. Stonehill v. Diokno, supra Asian Surety v. Herrera, 54 SCRA 312 I. Facts of the case: 1961 1964 Believed that the four offenses alleged to Asian Surety & Insurance Co., Inc estafa, falsification, tax evasion & insurance fraud took place. October 27, 1965 Judge Herrera, upon the sworn application of NBI agent Celso Zoleta, Jr. supported by the deposition of his witness, Manuel Cuaresma, issued a search warrant against the corporation. October 27-28, 1965 Armed with the search warrant, Zoleta and other agents entered the premises of the Republic Supermarket Building and served the search warrant in the presence of Mr. William Li Yao, president and chairman of the board of directors of the insurance firm. After the search they seized and carried away two carloads of documents, papers and receipt. October 27-28, 1965

The Asian Surety & Insurance Company Inc. filed a petition assailing the validity of the search warrant, claiming that it was issued in contravention of the explicit provisions of the Constitution and the Rules of Court, particularly Section 3, of Art. IV of the New Constitution, and Sections 3, 5, 8 and 10 of Rule 126 of the Rules of Court. End of Issue The criminal charges filed by the NBI have all been dismissed and/or dropped by the court or by the office of the City Fiscal of Manila in 1968, as manifested in the petition filed by the petitioner The search warrant reads in part: x x x property should be seized and brought to the undersigned. The judge made no attempt to determine whether the property he authorized to be searched and seized pertains specifically to any one of the three classes of personal property that may be searched and seized under a search warrant. It appears that the respondent judge intended the search to apply to all the three classes of property. The petitioner also manifested that the seizure of TWO carloads of their papers has paralyzed their business to the grave prejudice of not only the company, its workers, agents, employees but also of its numerous insured and beneficiaries of bonds issued by it, including the government itself, and of the general public. Asian Surety & Insurance Co., Inc. contests the validity of the search warrant on the ground that it authorized the search and seizure of personal properties so vaguely described and not particularized. It clearly violates the Constitutional Law: the right to be secure against unreasonable searches and seizures stating that the right of personal security, and that involves the exemption of his private affairs, books and papers from the inspection and scrutiny of others, is an important factor to peace and happiness. It also assails the non-compliance with the Sec 2 of Rule 126 which provides that a search warrant may be issued for the search and seizure of the enumerated personal properties including the property subject of the offense.

III. Ratio Decidendi A. Lower Court Judge Herrera approved the respondents application and commanded them to make an immediate search at any time to Asian Surety & Insurance Co., Inc. for there are good & sufficient reasons to believe that Mr. Yao or his employees committed offenses using their properties. B. Supreme Court The petition was granted; the search warrant of October 27, 1965, is nullified and set aside, and the respondents were ordered to return immediately all documents, papers and other objects seized or taken therunder. Without costs.

Castro v. Pabalan, 70 SCRA 477 Facts: Judge Pabalan ordered the issuance of a search warrant despite failure of the application of Lumang or the warrant itself to specify the offense, to examine the applicant as well as his witnesses on the part of the Judge, and to describe with particularity the place to be searched and the things to be seized. Judge never refuted the assertions when required to answer. Application alleged that applicants wee informed and claimed that they verified the report that Maria Castro and Co Ling are in possession of narcotics and other contraband in Barrio Padasil, Bangar, La Union without specifying the particular place in the Barrio. No complete description of the goods and inquiry was brief. Upon actual search, it turned out

that it was in Barrio Ma. Cristina and not in Padasil. Issue: Whether or not the search warrant is validly issued. Held: Search warrant issued illegal for violation of the 1935 Constitution and the Rules of Court because the two basic requirements are not complied with: (a) no warrant shall issue but upon probable cause, (b) the warrant shall particularly describe the things to be seized, thus, a general warrant. However, things seized cannot be returned and shall be destroyed, except the liquors, playing cards, distilled water and five bottles of Streptomycin. Pita v. CA, 178 SCRA 362 J. Sarmiento Facts: In 1983, elements of the Special Anti-Narcotics Group, and the Manila Police, seized and confiscated from dealers along Manila sidewalks, magazines believed to be obscene. These were later burned. One of the publications was Pinoy Playboy published by Leo Pita. He filed an injunction case against the mayor of manila to enjoin him from confiscating more copies of his magazine and claimed that this was a violation of freedom of speech. The court ordered him to show cause. He then filed an Urgent Motion for issuance of a temporary restraining order against indiscriminate seizure. Defendant Mayor Bagatsing admitted the confiscation and burning of obscence reading materials but admitted that these were surrendered by the stall owners and the establishments were not raided. The other defendant, WPD Superintendent, Narcisco Cabrera, filed no answer. On January 11, 1984, the trial court issued an Order setting the case for hearing on January 16, 1984 "for the parties to adduce evidence on the question of whether the publication 'Pinoy Playboy Magazine alleged (sic) seized, confiscated and/or burned by the defendants, are obscence per se or not". On February 3, 1984, the trial court promulgated the Order appealed from denying the motion for a writ of preliminary injunction, and dismissing the case for lack of merit The CA also dismissed the appeal due to the argument that freedom of the press is not without restraint. In the SC, the petitioner claimed that: 1. The CA erred in holding that the police officers could without any court warrant or order seize and confiscate petitioner's magazines on the basis simply of their determination that they are obscene. 2. The Court of Appeals erred in affirming the decision of the trial court and, in effect, holding that the trial court could dismiss the case on its merits without any hearing thereon when what was submitted to it for resolution was merely the application of petitioner for the writ of preliminary injunction. Issue: Was the seizure constitutional? Held: No. Petition granted Ratio: Test for obscenity: "whether the tendency of the matter charged as obscene, is to deprave or corrupt those whose minds are open to such immoral influences and into whose hands a publication or other article charged as being obscene may fall Also, "whether a picture is obscene or indecent must depend upon the circumstances of the case, and that ultimately, the question is to be decided by the "judgment of the aggregate sense of the community reached by it." (Kottinger) When does a publication have a corrupting tendency, or when can it be said to be offensive to human sensibilities? The issue is a complicated one, in which the fine lines have neither been drawn nor divided. Katigbak- "Whether to the average person, applying contemporary standards, the dominant theme of the material taken as a whole appeals to prurient interest." Kalaw-Katigbak represented a marked departure from Kottinger in the sense that it measured obscenity in terms of the "dominant theme" of the work, rather than isolated passages, which were central to Kottinger (although both cases are agreed that

"contemporary community standards" are the final arbiters of what is "obscene"). Kalaw-Katigbak undertook moreover to make the determination of obscenity essentially a judicial question and as a consequence, to temper the wide discretion Kottinger had given unto law enforcers. The latest say on American jurisprudence was Miller v. California, which expressly abandoned Massachusettes, and established "basic guidelines," to wit: "(a) whether 'the average person, applying contemporary standards' would find the work, taken as a whole, appeals to the prurient interest . . .; (b) whether the work depicts or describes, in a patently offensive way, sexual conduct specifically defined by the applicable state law; and (c) whether the work, taken as a whole, lacks serious literary, artistic, political, or scientific value. The lack of uniformity in American jurisprudence as to what constitutes "obscenity" has been attributed to the reluctance of the courts to recognize the constitutional dimension of the problem. Apparently, the courts have assumed that "obscenity" is not included in the guaranty of free speech, an assumption that, as we averred, has allowed a climate of opinions among magistrates predicated upon arbitrary, if vague theories of what is acceptable to society. In the case at bar, there is no challenge on the right of the State, in the legitimate exercise of police power, to suppress smut provided it is smut. For obvious reasons, smut is not smut simply because one insists it is smut. So is it equally evident that individual tastes develop, adapt to wide-ranging influences, and keep in step with the rapid advance of civilization. What shocked our forebears, say, five decades ago, is not necessarily repulsive to the present generation. But neither should we say that "obscenity" is a bare (no pun intended) matter of opinion. As we said earlier, it is the divergent perceptions of men and women that have probably compounded the problem rather than resolved it. Undoubtedly, "immoral" lore or literature comes within the ambit of free expression, although not its protection. In free expression cases, this Court has consistently been on the side of the exercise of the right, barring a "clear and present danger" that would warrant State interference and action. But the burden to show this lies with the authorities. "There must be objective and convincing, not subjective or conjectural, proof of the existence of such clear and present danger." As we so strongly stressed in Bagatsing, a case involving the delivery of a political speech, the presumption is that the speech may validly be said. The burden is on the State to demonstrate the existence of a danger, a danger that must not only be: (1) clear but also, (2) present, to justify State action to stop the speech. The Court is not convinced that the private respondents have shown the required proof to justify a ban and to warrant confiscation of the literature for which mandatory injunction had been sought below. First of all, they were not possessed of a lawful court order: (1) finding the said materials to be pornography, and (2) authorizing them to carry out a search and seizure, by way of a search warrant. Has petitioner been found guilty for publishing obscene works under Presidential Decrees Nos. 960 and 969? This not answered, one can conclude that the fact that the former respondent Mayor's act was sanctioned by "police power" is no license to seize property in disregard of due process. The PDs dont give the authorities the permission to execute high-handed acts. It is basic that searches and seizures may be done only through a judicial warrant, otherwise, they become unreasonable and subject to challenge. There is of course provision for warrantless searches under the Rules of Court but as the provision itself suggests, the search must have been an incident to a lawful arrest and it must be on account fo a crime committed. The Court rejected the argument that "[t]here is no constitutional nor legal provision which would free the accused of all criminal responsibility because there had been no warrant, and there is no "accused" here to speak of, who ought to be "punished". Second, to say that the respondent Mayor could have validly ordered the raid (as a result of an anti-smut campaign) without a lawful search warrant because, in his opinion, "violation of penal laws" has been committed, is to make the respondent Mayor judge, jury, and executioner rolled into one.

Pp v. Salanguit, G.R. No. 133254-55, April 19, 2001 Personal Determination of PC by the judge CASES: Placer v. Villanueva, 126 SCRA 463 Facts: Petitioners filed informations in the city court and they certified that Preliminary Investigation and Examination had been conducted and that prima facie cases have been found. Upon receipt of said informations, respondent judge set the hearing of the criminal cases to determine propriety of issuance of warrants of arrest. Afterthe hearing, respondent issued an order requiring petitioners to submit to the court affidavits of prosecution witnesses and other documentary evidence in support of the informations to aid him in the exercise of his power of judicial review of the findings of probable cause by petitioners. Petitioners petitioned for certiorari and mandamus to compel respondent to issue warrants of arrest. They contended that the fiscals certification in the informations of the existence of probable cause constitutes sufficient justification for the judge to issue warrants of arrest.

Panganiban, J. FACTSOn August 8, 1991, the Anti-Graft League of the Philippines, represented by its chief prosecutor Atty. Reynaldo L. Bagatsing, filed with the Office of the Ombudsman a complaint against Doris Teresa Ho, Rolando S. Narciso (petitioners in G.R. Nos. 106632 and 106678, respectively), Anthony Marden, Arsenio Benjamin Santos and Leonardo Odoo. The complaint was for alleged violation of Section 3 (g) of Republic Act 3019 prohibiting a public officer from entering into any contract or transaction on behalf of the government if it is manifestly and grossly disadvantageous to the latter, whether or not the public officer profited or will profit thereby. According to the information, Rolando Narciso, being then the VicePresident of the National Steel Corporation (NSC), a governmentowned or controlled corporation organized and operating under the Philippine laws, and Doris Ho, the President of National Marine Corporation (NMC), a private corporation organized and operating under our Corporation law, was said to have entered without legal justification into a negotiated contract of affreightment disadvantageous to the NSC for the haulage of its products at the rate of P129.50/MT, from Iligan City to Manila. Such contract was entered into despite their full knowledge that the rate they have agreed upon was much higher than those offered by the Loadstar Shipping Company, Inc. (LSCI) and Premier Shipping Lines, Inc. (PSLI), in the amounts of P109.56 and P123.00 per Metric Ton, respectively, in the public bidding, thereby giving unwarranted benefits to the National Marine Corporation. Ho and Narciso alleged that the Sandiganbayan, in determining probable cause for the issuance of the warrant for their arrest, merely relied on the information and the resolution attached thereto, filed by the Ombudsman without other supporting evidence, in violation of the requirements of Section 2, Article III of the Constitution, and settled jurisprudence. They contend that a judge, in personally determining the existence of probable cause, must have before him sufficient evidence submitted by the parties, other than the information filed by the investigating prosecutor, to support his conclusion and justify the issuance of an arrest warrant. Such evidence should not be merely described in a prosecutors resolution. Sandiganbayans Denial: Considering, therefore, that this Court did not rely solely on the certification appearing in the information in this case in the determination of whether probable cause exists to justify the issuance of the warrant of arrest but also on the basis predominantly shown by the facts and evidence appearing in the resolution/memorandum of responsible investigators/ prosecutors, then the recall of the warrant of arrest, or the reconsideration sought for, cannot be granted. More so, when the information, as filed, clearly shows that it is sufficient in form and substance based on the facts and evidence adduced by both parties during the preliminary investigation. To require this Court to have the entire record of the preliminary investigation to be produced before it, including the evidence submitted by the complainant and the accused-respondents, would appear to be an exercise in f utility. ISSUEMay a judge determine probable cause and issue a warrant of arrest solely on the basis of the resolution of the prosecutor (in the instant case, the Office of the Special Prosecutor of the Ombudsman) who conducted the preliminary investigation, without having before him any of the evidence (such as complainants affidavit, respondents counteraffidavit, exhibits, etc.) which may have been submitted at the preliminary investigation? DECISION & RATIO NO. Art III Section 2, 1987 Constitution : The right of the people to be secure in their persons, houses, papers, and effects against unreasonable searches and seizures of whatever nature and for any purpose shall be inviolable, and no search warrant or warrant of

Issue: Whether or Not respondent city judge may, for the purpose of issuing warrants of arrest, compel the fiscal to submit to the court the supporting affidavits and other documentary evidence presented during the preliminary investigation.

Held: Judge may rely upon the fiscals certification for the existence of probable cause and on the basis thereof, issue a warrant of arrest. But, such certification does not bind the judge to come out with the warrant. The issuance of a warrant is not a mere ministerial function; it calls for the exercise of judicial discretion on the part of issuing magistrate. Under Section 6 Rule 112 of the Rules of Court, the judgemust satisfy himself of the existence of probable cause before issuing a warrant of arrest. If on the face of the information, the judge finds no probable cause, he may disregard the fiscals certification and require submission of the affidavits of witnesses to aid him in arriving at the conclusion as to existence of probable cause. Soliven v. Makasiar, supra Beltran is among the petitioners in this case. He together with others was charged for libel by the president. Cory herself filed a complaint-affidavit against him and others. Makasiar averred that Cory cannot file a complaint affidavit because this would defeat her immunity from suit. He grounded his contention on the principle that a president cannot be sued. However, if a president would sue then the president would allow herself to be placed under the courts jurisdiction and conversely she would be consenting to be sued back. Also, considering the functions of a president, the president may not be able to appear in court to be a witness for herself thus she may be liable for contempt. ISSUE: Whether or not such immunity can be invoked by Beltran, a person other than the president. HELD: The rationale for the grant to the President of the privilege of immunity from suit is to assure the exercise of Presidential duties and functions free from any hindrance or distraction, considering that being the Chief Executive of the Government is a job that, aside from requiring all of the officeholders time, also demands undivided attention. But this privilege of immunity from suit, pertains to the President by virtue of the office and may be invoked only by the holder of the office; not by any other person in the Presidents behalf. Thus, an accused like Beltran et al, in a criminal case in which the President is complainant cannot raise the presidential privilege as a defense to prevent the case from proceeding against such accused. Moreover, there is nothing in our laws that would prevent the President from waiving the privilege. Thus, if so minded the President may shed the protection afforded by the privilege and submit to the courts jurisdiction. The choice of whether to exercise the privilege or to waive it is solely the Presidents prerogative. It is a decision that cannot be assumed and imposed by any other person.

Ho v. Pp, 280 SCRA 365

arrest shall issue except upon probable cause to be determined personally by the judge after examination under oath or affirmation of the complainant and the witnesses he may produce and particularly describing the place to be searched and the persons or things to be seized. (Art III Section 2, 1987 Constitution) HThe word personally does not appear in the corresponding provisions of our previous Constitutions. This emphasis shows the present Constitutions intent to place a greater degree of responsibility upon trial judges than that imposed under the previous Charters.

Soliven vs. Makasiar : In satisfying himself of the existence of probable cause for the issuance of a warrant of arrest, the judge is not required to personally examine the complainant and his witnesses. Following established doctrine and procedure, he shall: (1) personally evaluate the report and the supporting documents submitted by the fiscal regarding the existence of probable cause and, on the basis thereof, issue a warrant of arrest; or (2) if on the basis thereof he finds no probable cause, he may disregard the fiscals report and require the submission of supporting affidavits of witnesses to aid him in arriving at a conclusion as to the existence of probable cause. People vs. Inting : There is a difference between the judges goal from that of the prosecutors. First, the determination of probable cause is a function of the Judge. It is not for the Provincial Fiscal or Prosecutor or for the Election Supervisor to ascertain. Second, the preliminary inquiry made by a Prosecutor does not bind the Judge. It merely assists him to make the determination of probable cause. Third, Judges and Prosecutors alike should distinguish the preliminary inquiry which determines probable cause for the issuance of a warrant of arrest from the preliminary investigation proper which ascertains whether the offender should be held for trial or released. Even if the two inquiries are conducted in the course of one and the same proceeding, there should be no confusion about the objectives. The Court, in this case, reiterated and elaborated on the doctrine laid down in People vs. Intingand ruled that :

enable His Honor to make his personal and separate judicial finding on whether to issue a warrant of arrest Lastly, it is not required that the complete or entire records of the case during the preliminary investigation be submitted to and examined by the judge. We do not intend to unduly burden trial courts by obliging them to examine the complete records of every case all the time simply for the purpose of ordering the arrest of an accused. What is required, rather, is that the judge must have sufficient supporting documents (such as the complaint, affidavits, counter-affidavits, sworn statements of witnesses or transcripts of stenographic notes, if any) upon which to make his independent judgment or, at the very least, upon which to verify the findings of the prosecutor as to the existence of probable cause. The point is: he cannot rely solely and entirely on the prosecutors recommendation, as Respondent Court did in this case. Although the prosecutor enjoys the legal presumption of regularity in the performance of his official duties and functions, which in turn gives his report the presumption of accuracy, the Constitution, we repeat, commands the judge to personallydetermine probable cause in the issuance of warrants of arrest. This Court has consistently held that a judge fails in his bounden duty if he relies merely on the certification or the report of the investigating officer. IN THE INSTANT CASE, the public respondent relied fully and completely upon the resolution of the graft investigation officer and the memorandum of the reviewing prosecutor, attached to the information filed before it, and its conjecture that the Ombudsman would not have approved their recommendation without supporting evidence. It had no other documents from either the complainant (the Anti-Graft League of the Philippines) or the People from which to sustain its own conclusion that probable cause exists. Respondent Court palpably committed grave abuse of discretion in ipso facto issuing the challenged warrant of arrest on the sole basis of the prosecutors findings and recommendation, and without determining on its own the issue of probable cause based on evidence other than such bare findings and recommendation. Webb v. de Leon, 247 SCRA 652 FACTS:On June 19, 1994, the National Bureau of I n v e s t i g a t i o n ( N B I ) f i l e d w i t h t h e Department of Justice a lettercomplaintc h a r g i n g p e t i t i o n e r s H u b e r t W e b b , Michael Gatchalian, Antonio J. Lejano ands i x ( 6 ) o t h e r persons with the crime of R a p e a n d H o m i c i d e o f C a r m e l a N . Vizconde, her mother Estrellita NicolasV i z c o n d e , a n d h e r s i s t e r A n n e M a r i e Jennifer i n their home at Number 80 W.Vinzons, St., BF Homes Paranaque, MetroManila on June 30, 1991.F o r t h w i t h , t h e D e p a r t m e n t o f J u s t i c e formed a panel of prosecutors headed byAssistant Chief State Prosecutor JovencioR . Z u n o t o c o n d u c t t h e p r e l i m i n a r y investigation.ARGUMENTS:P e t i t i o n e r s f a u l t t h e D O J P a n e l f o r i t s finding of probable cause. They assail thecredibility of Jessica Alfaro as inherentlyw e a k a n d u n c o r r o b o r a t e d d u e t o t h e inconsistencies between her April 28,1 9 9 5 a n d M a y 2 2 , 1 9 9 5 s w o r n statements. They criticize the procedurefollowed by the DOJ Panel when it did notexamine witnesses to clarify the allegedinconsistencies.Petitioners charge that respondent JudgeRaul de Leon and, later, respondent JudgeA m e l i t a T o l e n t i n o i s s u e d w a r r a n t s o f arrest against them without conductingthe required preliminary examination.Petitioners complain about the denial of their constitutional right to due processand violation of their right to an impartiali n v e s t i g a t i o n . T h e y a l s o a s s a i l t h e prejudicial publicity that attended theirpreliminary investigation.ISSUES:1. Whether or not the DOJ Panel likewisegravely abused its discretion in holdingt h a t t h e r e

First, as held in Inting, the determination of probable cause by the prosecutor is for a purpose different from that which is to be made by the judge. Whether there is reasonable ground to believe that the accused is guilty of the offense charged and should be held for trial is what the prosecutor passes upon. The judge, on the other hand, determines whether a warrant of arrest should be issued against the accused, i.e. whether there is a necessity for placing him under immediate custody in order not to frustrate the ends of justice. Thus, even if both should base their findings on one and the same proceeding or evidence, there should be no confusion as to their distinct objectives. Second, since their objectives are different, the judge cannot rely solely on the report of the prosecutor in finding probable cause to justify the issuance of a warrant of arrest. Obviously and understandably, the contents of the prosecutors report will support his own conclusion that there is reason to charge the accused of an offense and hold him for trial. However, the judge must decide independently. Hence, he must have supporting evidence, other than the prosecutors bare report, upon which to legally sustain his own findings on the existence (or nonexistence) of probable cause to issue an arrest order. This responsibility of determining personally and independently the existence or nonexistence of probable cause is lodged in him by no less than the most basic law of the land. Parenthetically, the prosecutor could ease the burden of the judge and speed up the litigation process by forwarding to the latter not only the information and his bare resolution finding probable cause, but also so much of the records and the evidence on hand as to

i s p r o b a b l e c a u s e t o c h a r g e them with the crime of rape and homicide2. Whether or not respondent Judges deLeon and Tolentino gravely abused theirdiscretion when they failed to conduct apreliminary examination before issuingwarrants of arrest against them3. Whether or not the DOJ Panel deniedt h e m t h e i r c o n s t i t u t i o n a l r i g h t t o d u e p r o c e s s d u r i n g t h e i r p r e l i m i n a r y inve stigation4 . W h e t h e r o r n o t t h e D O J P a n e l u n l a w f u l l y i n t r u d e d i n t o j u d i c i a l prerogative when it failed t o c h a r g e Jessica Alfaro in the information as anaccused.HELD:1. NO.2. NO.3. NO. There is no merit in this contentionb e c a u s e p e t i t i o n e r s w e r e g i v e n a l l t h e opportuni ties to be heard.4. NO.REASONS:1. The Court ruled that the DOJ Panel didnot gravely abuse its discretion when itf o u n d p r o b a b l e c a u s e a g a i n s t t h e p etitioners. A probable cause needs onlyt o r e s t o n e v i d e n c e showing that morel i k e l y t h a n n o t , a c r i m e h a s b e e n committed and was committed by thesu s p e c t s . P r o b a b l e c a u s e n e e d n o t b e based on clear and convincing evidence of g u i l t , n e i t h e r o n e v i d e n c e e s t a b l i s h i n g g u i l t b e y o n d r e a s o n a b l e d o u b t a n d definitely, not on evidence establishingabsolute certainty of guilt.2. The Court ruled that respondent judgesdid not gravely abuse their discretion. Ina r r e s t c a s e s , t h e r e m u s t b e a p r o b a b l e cause that a crime has been committeda n d t h a t t h e p e r s o n t o b e a r r e s t e d committed it. Section 6 of Rule 112 simplyp r o v i d e s t h a t u p o n f i l i n g o f a n information, the Regional Trial Court mayissue a warrant for the accused. Clearlythe, our laws repudiate the submission of petitioners that respondent Judges shouldhave conducted searching examinationof witnesses before issuing warrants of arrest against them.3. The DOJ Panel precisely ed the partiesto adduce more evidence in their behalf and for the panel to study the evidence submitted more fully.4. Petitioners argument lacks appeal for itl i e s o n t h e f a u l t y a s s u m p t i o n t h a t t h e decision whom to prosecute is a judicialf u n c t i o n , t h e s o l e p r e r o g a t i v e o f t h e c o u r t s a n d b e y o n d e x e c u t i v e a n d l e g i s l a t i v e i n t e r f e r e n c e . I n t r u t h , t h e prosecuti on of crimes appertains to thee x e c u t i v e d e p a r t m e n t o f g o v e r n m e n t whose principal power and responsibilityi s t o s e e t h a t o u r l a w s a r e f a i t h f u l l y executed. A necessary component of thisp o w e r i s t h e r i g h t t o p r o s e c u t e t h e i r violat ors (See R.A. No. 6981 and section 9of Rule 119 for legal basis).With regard to the inconsistencies of thes w o r n s t a t e m e n t s o f J e s s i c a A l f a r o , t h e C o u r t b e l i e v e s t h a t t h e s e h a v e b e e n sufficiently ex p l a i n e d a n d t h e r e i s n o showing that the inconsis t e n c i e s w e r e deliberately made to distort the truth.With regard to the petitioners complainta b o u t t h e p r e j u d i c i a l p u b l i c i t y t h a t attended their preliminary investigation,the Court finds nothing in the records thatwill prove that the tone and content of thepublicity that attended the investigationof petitioners fatally infected the fairnessa n d i m p a r t i a l i t y o f t h e D O J P a n el .P e t it io ne r s c an no t j u st r e ly o n t h e subliminal effects of publicity on thesen se of fairness of the DOJ Panel, fort h e s e a r e b a s i c a l l y u n b e k n o w n a n d beyond knowing. Roberts v. CA, 254 SCRA 307 DECISION DAVIDE, JR., J.: I. THE FACTS

Petitioners, who are corporate officers and members of the Board of Pepsi Cola Products Phils., Inc. were prosecuted in connection with the Pepsi Number Fever promotion by handlers of the supposedly winning 349 Pepsi crowns. Of the four cases filed against the petitioners, probable cause was found by the investigating prosecutor only for the crime of estafa, but not for the other alleged offenses. On 12 April 1993, the information was filed with the trial court without anything accompanying it. A copy of the investigating prosecutors Joint Resolution was forwarded to and received by the trial court only on 22 April 1993. However, no affidavits of the witnesses, transcripts of stenographic notes of the proceedings during the preliminary investigation, or other documents submitted in the course thereof were found in the records of the case as of 19 May 1993. On 15 April 1993, petitioners Roberts, et al. filed a petition for review to the Department of Justice seeking the reversal of the finding of probable cause by the investigating prosecutor. They also moved for the suspension of the proceedings and the holding in abeyance of the issuance of warrants of arrest against them. Meanwhile, the public prosecutor also moved to defer the arraignment of the accused-appellants pending the final disposition of the appeal to the Department of Justice. On 17 May 1993, respondent Judge Asuncion issued the challenged order (1) denying, on the basis of Crespovs. Mogul, the foregoing motions respectively filed by the petitioners and the public prosecutor, and directing the issuance of the warrants of arrest after June 1993 and setting the arraignment on 28 June 1993. In part, respondent judge stated in his order that since the case is already pending in this Court for trial, following whatever opinion the Secretary of Justice may have on the matter would undermine the independence and integrity his court. To justify his order, he quoted the ruling of the Supreme Court in Crespo, which stated: In order therefor to avoid such a situation whereby the opinion of the Secretary of Justice who reviewed the action of the fiscal may be disregarded by the trial court, the Secretary of Justice should, as far as practicable, refrain from entertaining a petition for review or appeal from the action of the fiscal, when the complaint or information has already been filed in Court. The matter should be left entirely for the determination of the Court. Petitioners went to the Court of Appeals (CA), arguing that the respondent judge had not the slightest basis at all for determining probable cause when he ordered the issuance of warrants of arrest. After finding that a copy of the public prosecutors Joint Resolution had in fact been forwarded to, and received by, the trial court on 22 April 1993, the CA denied petitioners application for writ of preliminary injunction. The CA ruled that the Joint Resolution was sufficient in itself to have been relied upon by respondent Judge in convincing himself that probable cause indeed exists for the purpose of issuing the corresponding warrants of arrest and that the mere silence of the records or the absence of any express declaration in the questioned order as to the basis of such finding does not give rise to an adverse inference, for the respondent Judge enjoys in his favor the presumption of regularity in the performance of his official duty. Roberts, et al. sought reconsideration, but meanwhile, the DOJ affirmed the finding of probable cause by the investigating prosecutor. The CA therefore dismissed the petition for mootness. II. THE ISSUES 1. Did Judge Asuncion commit grave abuse of discretion in denying, on the basis of Crespo vs. Mogul, the motions to suspend proceedings and hold in abeyance the issuance of warrants

of arrest and to defer arraignment until after the petition for review filed with the DOJ shall have been resolved? 2. Did Judge Asuncion commit grave abuse of discretion in ordering the issuance of warrants of arrest without examining the records of the preliminary investigation? 3. May the Supreme Court determine in this [sic] proceedings the existence of probable cause either for the issuance of warrants of arrest against the petitioners or for their prosecution for the crime of estafa? III. THE RULING [The Court, in a 7-5-2 vote, GRANTED the petition. It SET ASIDE the decision and resolution of the CA, the resolutions of the DOJ 349 Committee, and the order of respondent judge.] 1. YES, Judge Asuncion committed grave abuse of discretion in denying, on the basis of Crespo vs. Mogul, the motions to suspend proceedings and hold in abeyance the issuance of warrants of arrest and to defer arraignment until after the petition for review filed with the DOJ shall have been resolved. There is nothing in Crespo vs. Mogul which bars the DOJ from taking cognizance of an appeal, by way of a petition for review, by an accused in a criminal case from an unfavorable ruling of the investigating prosecutor. It merely advised the DOJ to, as far as practicable, refrain from entertaining a petition for review or appeal from the action of the fiscal, when the complaint or information has already been filed in Court. Whether the DOJ would affirm or reverse the challenged Joint Resolution is still a matter of guesswork. Accordingly, it was premature for respondent Judge Asuncion to deny the motions to suspend proceedings and to defer arraignment on the following grounds: This case is already pending in this Court for trial. To follow whatever opinion the Secretary of Justice may have on the matter would undermine the independence and integrity of this Court. This Court is still capable of administering justice. The real and ultimate test of the independence and integrity of this court is not the filing of the aforementioned motions [to suspend proceedings and issuance of warrants of arrest and to defer arraignment] at that stage but the filing of a motion to dismiss or to withdraw the information on the basis of a resolution of the petition for review reversing the Joint Resolution of the investigating prosecutor. However, once a motion to dismiss or withdraw the information is filed the trial judge may grant or deny it, not out of subservience to the Secretary of Justice, but in faithful exercise of judicial prerogative. 2. YES, Judge Asuncion committed grave abuse of discretion in ordering the issuance of warrants of arrest without examining the records of the preliminary investigation. The teachings then of Soliven, Inting, Lim, Allado, and Webb reject the proposition that the investigating prosecutors certification in an information or his resolution which is made the basis for the filing of the information, or both, would suffice in the judicial determination of probable cause for the issuance of a warrant of arrest. In Webb, this Court assumed that since the respondent Judges had before them not only the 26-page resolution of the investigating panel but also the affidavits of the prosecution witnesses and even the counter-affidavits of the respondents, they (judges) made personal evaluation of the evidence attached to the records of the case.

In this case, nothing accompanied the information upon its filing on 12 April 1993 with the trial court. A copy of the Joint Resolution was forwarded to, and received by, the trial court only on 22 April 1993. And as revealed by the certification of respondent judges clerk of court, no affidavits of the witnesses, transcripts of stenographic notes of the proceedings during the preliminary investigation, or other documents submitted in the course thereof were found in the records of this case as of 19 May 1993. Clearly, when respondent Judge Asuncion issued the assailed order of 17 May 1993 directing, among other things, the issuance of warrants of arrest, he had only the information, amended information, and Joint Resolution as bases thereof. He did not have the records or evidence supporting the prosecutor's finding of probable cause. And strangely enough, he made no specific finding of probable cause; he merely directed the issuance of warrants of arrest after June 21, 1993. It may, however, be argued that the directive presupposes a finding of probable cause. But then compliance with a constitutional requirement for the protection of individual liberty cannot be left to presupposition, conjecture, or even convincing logic. 3. NO, the Supreme Court MAY NOT determine in this [sic] proceedings the existence of probable cause either for the issuance of warrants of arrest against the petitioners or for their prosecution for the crime of estafa. Ordinarily, the determination of probable cause is not lodged with this Court. Its duty in an appropriate case is confined to the issue of whether the executive or judicial determination, as the case may be, of probable cause was done without or in excess of jurisdiction or with grave abuse of discretion amounting to want of jurisdiction. This is consistent with the general rule that criminal prosecutions may not be restrained or stayed by injunction, preliminary or final. There are, however, exceptions to the foregoing rule. But the Court refused to reevaluate the evidence to determine if indeed there is probable cause for the issuance of warrants of arrest in this case. For the respondent judge did not, in fact, find that probable cause exists, and if he did he did not have the basis therefor. Moreover, the records of the preliminary investigation in this case are not with the Court. They were forwarded by the Office of the City Prosecutor of Quezon City to the DOJ in compliance with the latter's 1st Indorsement of 21 April 1993. The trial court and the DOJ must be required to perform their duty EXC: determination by administrative officials CASES: Morano v. Vivo, 20 SCRA

562 Facts: Petitioners are Chinese nationals (Chan Sau Wah from Fukein, with a minor child from prior marriage, Fu Yan Fun) who were granted a temporary visitors visa as mom-immigrant for 2 months upon posting P4k cash bond to visit a cousin in the Philippines. She soon married to Esteban Morano, a Filipino Citizen, on January 24, 1962 and gave birth to a child, Esteban Morano, Jr. After several extensions to prolong stay in Philippines, their visas expired on Sept. 10, 1962 and were ordered by Commissioner of Immigration (COI) on Aug. 31, 1962 thru a letter, to leave the country on or before Sept. 10, 1962 with warning of issuance of warrant of arrest for failure to leave and confiscation of bond. Petitioners then filed with the CFI of Manila for Mandamus to compel COI to cancel their ACR, to stop issuing arrest warrant, and preliminary injunction from confiscating their bond. They argue that Chan Sau Wah became a Filipino Citizen upon marriage to Esteban Morano by virtue of Section 15 of Commonwealth Act No. 473 (Revised Naturalization Act). Likewise, it argues that Section 37 of the Naturalization Law is unconstitutional for allowing the COI to issue warrant of arrest and effecting deportation without judicial

intervention enshrined in the Constitution. CFI decided partly against petitioners, thus, COI and petitioners both appealed to SC. Issues:

Whether or not the marriage of Chan Sau Wah to Esteban Morano makes her a Filipino citizen. Whether or not Section 37 of the Naturalization Law empowering the COI to issue a warrant of arrest, and deport upon a warrant on deportation cases is unconstitutional for are covered by the Constitutional mandate on searches and seizures without judicial intervention required under the Constitution on searches and seizures.

Petitioners were among the 22 suspected alien pedophiles. They were apprehended 17 February1988 after close surveillance for 3 month of the CID in Pagsanjan, Laguna. 17 of the arrested aliens opted for self-deportation. One released for lack of evidence, another charged not for pedophile but working with NO VISA, the 3 petitioners chose to face deportation proceedings. On 4 March1988, deportation proceedings were instituted against aliens for being undesirable aliens under Sec.69 of Revised Administrative Code. Warrants of Arrest were issued 7March1988 against petitioners for violation of Sec37, 45 and 46 of Immigration Act and sec69 of Revised Administrative Code. Trial by the Board of Special Inquiry III commenced the same date. Petition for bail was filed 11March 1988 but was not granted by the Commissioner of Immigration. 4 April1988 Petitioners filed a petition for Writ of Habeas Corpus. The court heard the case on oral argument on 20 April 1988. Issue: Whether or Not the Commissioner has the power to arrest and detain petitioners pending determination of existence of probable cause. Whether or Not there was unreasonable searches and seizures by CID agents. Whether or Not the writ of Habeas Corpus may be granted to petitioners. Held: While pedophilia is not a crime under the Revised Penal Code, it violates the declared policy of the state to promote and protect the physical, moral, spiritual and social well being of the youth. The arrest of petitioners was based on the probable cause determined after close surveillance of 3 months. The existence of probable cause justified the arrest and seizure of articles linked to the offense. The articles were seized as an incident to a lawful arrest; therefore the articles are admissible evidences (Rule 126, Section12 of Rules on Criminal Procedure). The rule that search and seizures must be supported by a valid warrant of arrest is not an absolute rule. There are at least three exceptions to this rule. 1.) Search is incidental to the arrest. 2.) Search in a moving vehicle. 3.) Seizure of evidence in plain view. In view of the foregoing, the search done was incidental to the arrest. The filing of the petitioners for bail is considered as a waiver of any irregularity attending their arrest and estops them from questioning its validity. Furthermore, the deportation charges and the hearing presently conducted by the Board of Special Inquiry made their detention legal. It is a fundamental rule that habeas corpus will not be granted when confinement is or has become legal, although such confinement was illegal at the beginning. The deportation charges instituted by the Commissioner of Immigration are in accordance with Sec37 (a) of the Philippine Immigration Act of 1940 in relation to sec69 of the Revised Administrative code. Section 37 (a) provides that aliens shall be arrested and deported upon warrant of the Commissioner of Immigration and Deportation after a determination by the Board of Commissioners of the existence of a ground for deportation against them. Deportation proceedings are administrative in character and never construed as a punishment but a preventive measure. Therefore, it need not be conducted strictly in accordance with ordinary Court proceedings. What is essential is that there should be a specific charge against the alien intended to be arrested and deported. A fair hearing must also be conducted with assistance of a counsel if desired. Lastly, the power to deport aliens is an act of the State and done under the authority of the sovereign power. It a police measure against the undesirable aliens whose continued presence in the

Ruling: Citizenship. Marriage to a Filipino citizen does not ipso facto make her a Filipino citizen. She must show that she possess all the qualifications, and none of the disqualifications required by the Naturalization Law requiring as follows: Alien woman herself might be lawfully naturalized In the additional stipulation of facts of July 3, 1963, petitioners admit that Chan Sau Wah is not possessed of all the qualifications required by the Naturalization Law. Thus, she did not become a Filipino citizen. Searches and seizures. Power to deport aliens is an attribute of sovereignty planted on the accepted maxim of international law, that every sovereign nation has the power, as inherent in sovereignty, and essential to self-preservation, to forbid the entrance of foreigners within its dominions. Section 1 (3), Article III of the Constitution, does not require judicial intervention in the execution of a final order of deportation issued in accordance with law. The constitutional limitation contemplates an order of arrest in the exercise of judicial power as a step preliminary or incidental to prosecution or proceeding for a given offense or administrative action, not as a measure indispensable to carry out a valid decision by a competent official, such as a legal order of deportation, issued by the Commissioner of Immigration, in pursuance of a valid legislation. Petition for mandamus and prohibition with respect to petitioners Chan Sau Wah is hereby denied; and judgment declaring her a citizen of the Philippines, directing COI to cancel her Alien Certificate of Registration and other immigration papers, and declaring the preliminary injunction with respect to her permanent, are all hereby set aside; and in all other respects, the decision appealed is hereby affirmed. Harvey v. Santiago, 162 SCRA 840 Facts: This is a petition for Habeas Corpus. Petitioners are the following: American nationals Andrew Harvey, 52 and Jonh Sherman 72. Dutch Citizen Adriaan Van Den Elshout, 58. All reside at Pagsanjan Laguna respondent Commissioner Miriam Defensor Santiago issued Mission Orders to the Commission of Immigration and Deportation (CID) to apprehended petitioners at their residences. The Operation Report read that Andrew Harvey was found together with two young boys. Richard Sherman was found with two naked boys inside his room. While Van Den Elshout in the after Mission Report read that two children of ages 14 and 16 has been under his care and subjects confirmed being live-in for sometime now. Seized during the petitioners apprehension were rolls of photo negatives and photos of suspected child prostitutes shown in scandalous poses as well as boys and girls engaged in sex. Posters and other literature advertising the child prostitutes were also found.

Valid marriage; and

country is found to be injurious to the public good and tranquility of the people. o EXC to EXC CASE: Salazar v. Achacoso, 183 SCRA 145 Facts: Rosalie Tesoro of Pasay City in a sworn statement filed with the POEA, charged petitioner with illegal recruitment. Public respondent Atty. Ferdinand Marquez sent petitioner a telegramdirecting him to appear to the POEA regarding the complaint against him. On the same day, after knowing that petitioner had no license to operate a recruitment agency, public respondent Administrator TomasAchacoso issued a Closure and Seizure Order No. 1205 to petitioner. It stated that there will a seizure of the documents and paraphernalia being used or intended to be used as the means of committing illegal recruitment, it having verified that petitioner has (1) No valid license or authority from the Department of Labor and Employment to recruit and deploy workers for overseas employment; (2) Committed/are committing acts prohibited under Article 34 of the New Labor Code in relation to Article 38 of the same code. A team was then tasked to implement the said Order. The group, accompanied by mediamen and Mandaluyong policemen, went to petitioners residence. They served the order to a certain Mrs. For a Salazar, who let them in. The team confiscated assorted costumes. Petitioner filed with POEA a letter requesting for the return of the seized properties, because she was not given prior notice and hearing. The said Order violated dueprocess. She also alleged that it violated sec 2 of the Bill of Rights, and the properties were confiscated against her will and were done with unreasonable force and intimidation.

as a money lender charging usurious rates of interest in violation of the law. In his oath the chief of the secret service did not swear to the truth of his statements upon his knowledge of the facts but the information received by him from a reliable person. Upon this questioned affidavit, the judge issued the search warrant, ordering the search of the petitioners house at any time of the day or night, the seizure of the books and documents and the immediate delivery of such to him (judge). With said warrant, several agents of the AntiUsury Board entered the petitioner's store and residence at 7 o'clock of the night and seized and took possession of various articles belonging to the petitioner. The petitioner asks that the warrant of issued by the Court of First Instance of Tayabas, ordering the search of his house and the seizure, at anytime of the day or night, of certain accounting books, documents, and papers belonging to him in his residence situated in Infanta, Tayabas, as well as the order of a later date, authorizing the agents of the Anti-Usury board to retain the articles seized, be declared illegal and set aside, and prays that all the articles in question be returned to him. Issues: 1.) What is the nature of searchers and seizures as contemplated in the law?

Issue: Whether or Not the Philippine Overseas Employment Administration (or the Secretary of Labor) can validly issue warrants ofsearch and seizure (or arrest) under Article 38 of the Labor Code

Held: Under the new Constitution, . . . no search warrant or warrant of arrest shall issue except upon probable cause to be determined personally by the judge after examination under oath or affirmation of the complainant and the witnesses he may produce, and particularly describing the place to be searched and the persons or things to be seized. Mayors and prosecuting officers cannot issue warrants of seizure or arrest. The Closure and Seizure Order was based on Article 38 of the Labor Code. The Supreme Court held, We reiterate that the Secretary of Labor, not being a judge, may no longer issue search or arrest warrants. Hence, the authorities must go through the judicial process. To that extent, we declare Article 38, paragraph (c), of the Labor Code, unconstitutional and of no force and effect The power of the President to order the arrest of aliens for deportation is, obviously, exceptional. It (the power to order arrests) cannot be made to extend to other cases, like the one at bar. Under the Constitution, it is the sole domain of the courts. Furthermore, the search and seizure order was in the nature of a general warrant. The court held that the warrant is null and void, because it must identify specifically the things to be seized. WHEREFORE, the petition is GRANTED. Article 38, paragraph (c) of the Labor Code is declared UNCONSTITUTIONAL and null and void. The respondents are ORDERED to return all materials seized as a result of the implementation of Search and Seizure Order No. 1205.

2.) What is required of the oath in the issuance of search warrant? 3.) What is the purpose of the disposition in addition to the affidavit? 4.) Whether or not the search warrant could be serve at night? 5.) Whether or not the seizure of evidence to use in an investigation is constitutional? 6.) Whether or not there was a waiver of constitutional guarantees? Held: A search warrant is an order in writing, issued in the name of the People of the Philippine Islands, signed by a judge or a justice of the peace, and directed to a peace officer, commanding him to search for personal property and bring it before the court (section 95, General Orders. No. 58, as amended by section 6 of Act No. 2886). Of all the rights of a citizen, few are of greater importance or more essential to his peace and happiness than the right of personal security, and that involves the exemption of his private affairs, books, and papers from the inspection and scrutiny of others ( In re Pacific Railways Commission, 32 Fed., 241; Interstate Commerce Commission vs Brimson, 38 Law. ed., 1047; Broyd vs. U. S., 29 Law. ed., 746; Caroll vs. U. S., 69 Law. ed., 543, 549). While the power to search and seize is necessary to the public welfare, still it must be exercised and the law enforced without transgressing the constitutional rights or citizen, for the enforcement of no statue is of sufficient importance to justify indifference to the basis principles of government (People vs. Elias, 147 N. E., 472).

Examination of Applicant by the judge CASES: J. Imperial Alvarez v. CFI, 64 Phil 3

Facts: On June 3, 1936, the chief of of the secret service of the Anti-Usury Board presented to Judge David, presiding judge of CFI of Tayabas, alleging that according to reliable information, the petitioner is keeping in his house in Infanta, Tayabas documents, receipts, lists, chits and other papers used by him in connection with his activities

equally well founded and that the search could not legally be made As the protection of the citizen and the maintenance of his constitutional right is one of the highest duties and privileges of the court, these constitutional guaranties should be given a liberal construction or a strict construction in favor of the individual, to prevent stealthy encroachment upon, or gradual depreciation on, the rights secured by them(State vs.Custer County, 198 Pac., 362; State vs. McDaniel, 231 Pac., 965; 237 Pac., 373). Since the proceeding is a drastic one, it is the general rule that statutes authorizing searches and seizure or search warrants must be strictly construed (Rose vs. St. Clair, 28 Fed., [2d], 189; Leonard vs.U. S., 6 Fed. [2d], 353; Perry vs. U. S. 14 Fed. [2d],88; Cofer vs.State, 118 So., 613). Unreasonable searches and seizures are a menace against which the constitutional guarantee afford full protection. The term At the hearing of the incidents of the case raised before the court it clearly appeared that the books and documents had really been seized to enable the Anti-Usury Board to conduct an investigation and later use all or some of the articles in question as evidence against the petitioner in the criminal cases that may be filed against him. The seizure of books and documents by means of a search warrant, for the purpose of using them as evidence in a criminal case against the person in whose possession they were found, is unconstitutional because it makes the warrant unreasonable, and it is equivalent to a violation of the constitutional provision prohibiting the compulsion of an accused to testify against himself (Uy Kheytin vs. Villareal, 42 Phil,, 886; Brady vs.U. S., 266 U. S., 620; Temperani vs. U. S., 299 Fed., 365; U. S. vs.Madden, 297 Fed., 679; Boyd vs. U. S.,116 U. S., 116; Caroll vs. U. S., 267 U. S., 132). Therefore, it appearing that at least nineteen of the documents in question were seized for the purpose of using them as evidence Neither the Constitution nor General Orders. No. 58 provides that it is of imperative necessity to take the deposition of the witnesses to be presented by the applicant or complainant in addition to the affidavit of the latter. The purpose of both in requiring the presentation of depositions is nothing more than to satisfy the committing magistrate of the existence of probable cause. Therefore, if the affidavit of the applicant or complainant is sufficient, the judge may dispense with that of other witnesses. Inasmuch as the affidavit of the agent in this case was insufficient because his knowledge of the facts was not personal but merely hearsay, it is the duty of the judge to require the affidavit of one or more witnesses for the purpose of determining the existence of probable cause to warrant the issuance of the search warrant. When the affidavit of the applicant of the complaint contains sufficient facts within his personal and direct knowledge, it is sufficient if the judge is satisfied that there exist probable cause; when the applicant's knowledge of the facts is mere hearsay, the affidavit of one or more witnesses having a personal knowledge of the fact is necessary. We conclude, therefore, that the warrant issued is likewise illegal because it was based only on the affidavit of the agent who had no personal knowledge of the facts. Section 101 of General Orders, No. 58 authorizes that the search be made at night when it is positively asserted in the affidavits that the property is on the person or in the place ordered to be searched. As we have declared the affidavits insufficient and the warrant issued exclusively upon it illegal, our conclusion is that the contention is The Anti-Usury Board insinuates in its answer that the petitioner cannot now question the validity of the search warrant or the proceedings had subsequent to the issuance thereof, because he has waived his constitutional rights in proposing a compromise whereby he agreed to pay a fine of P200 for the purpose of evading the criminal proceeding or proceedings. We are of the opinion that there was no such waiver, first, because the petitioner has emphatically denied the offer of compromise and, second, because if there was a compromise it reffered but to the institution of criminal proceedings fro violation of the Anti-Usury Law. The waiver would have been a good defense for the respondents had the petitioner voluntarily consented to the search and seizure of the articles in question, but such was not the case because the petitioner protested from the beginning and stated his protest in writing in the insufficient inventory furnished him by the agents. Burgos v. Chief of Staff, supra Mata v. Bayona, 128 SCRA 388 FACTS: The contention is that the search warrant issued by respondent Judge was based merely on application for Search Warrant and a joint affidavit of private respondents which were wrongfully allegedly subscribed, and sworn to before the Clerk of Court. Furthermore, there was allegedly a failure on the part of the respondent Judge to attach the necessary papers pertinently under against the petitioner in the criminal proceeding or proceedings for violation against him, we hold that the search warrant issued is illegal and that the documents should be returned to him. "unreasonable search and seizure" is not defined in the Constitution or in General Orders No. 58, and it is said to have no fixed, absolute or unchangeable meaning, although the term has been defined in general language. All illegal searches and seizure are unreasonable while lawful ones are reasonable. What constitutes a reasonable or unreasonable search or seizure in any particular case is purely a judicial question, determinable from a consideration of the circumstances involved, including the purpose of the search, the presence or absence or probable cause, the manner in which the search and seizure was made, the place or thing searched, and the character of the articles procured (Go-Bart Importing Co. vs. U. S. 75 Law. ed., 374; Peru vs. U. S., 4 Fed., [2d], 881;U. S.vs. Vatune, 292 Fed., 497; Angelo vs. U. S. 70 Law, ed., 145; Lambertvs. U. S. 282 Fed., 413; U. S. vs. Bateman, 278 Fed., 231; Mason vs.Rollins, 16 Fed. Cas. [No. 9252], 2 Biss., 99). The only description of the articles given in the affidavit presented to the judge was as follows: "that there are being kept in said premises books, documents, receipts, lists, chits and other papers used by him in connection with his activities as money-lender, charging a usurious rate of interest, in violation of the law." Taking into consideration the nature of the article so described, it is clear that no other more adequate and detailed description could have been given, particularly because it is difficult to give a particular description of the contents thereof. The description so made substantially complies with the legal provisions because the officer of the law who executed the warrant was thereby placed in a position enabling him to identify the articles, which he did. at night.

PD 810, as amended by PD1306, the information against him alleging that Soriano Mata offered, took, and arrangedvbets on the Jai Alai game by selling illegal tickets knows as Masiao tickets without any authority from the Philippine Jai Alai and Corporation or from the government authorities concerned. ISSUE: Whether or not Search Warrant is valid? HELD: No. The Search Warrant is declared as illegal. RATIONALE: Deposition, sometimes used in a broad sense to describe any written statement verified by oath; but in its more technical and appropriate sense the meaning of the word is limited to written testimony of a witness given in the course of a judicial proceeding in advance of the trial or hearing upon oral examination. Mere affidavits of the complainant and his witnesses are thus not sufficient. The examining Judge has to take depositions in writing of the complainant and the witnesses he may produce and to attach them to the record. Such written deposition is necessary in order that the Judge may be able to property determine the existence or non-existence of the probable cause, to hold liable for perjury the person giving if it will be found later his declarations are false. Ponsica v. Ignalaga, 152 SCRA 647 The chief issue raised by the petitioners in this case is whether or 1 not Section 143 of the Local Government Code granting power to the municipal mayor to conduct preliminary investigations and order the arrest of the accused, was repealed by the 1985 Rules on Criminal Procedure promulgated by this Court; and is, in addition, unconstitutional as vesting the power to conduct preliminary investigations in an official who cannot be deemed a "neutral and detached magistrate" within the contemplation of Section 3, Article IV of the 1973 Constitution. The issue is hereby resolved adversely to the petitioners, with the stressed qualification that the mayor's power to order arrest ceased to exist as of February 2, 1987 when the new Constitution was ratified by the Filipino people, and that, in any event, the investigation actually conducted by respondent mayor in the case at bar was fatally defective. Shortly after noon on September 20, 1985, an attempt was made by firemen and soldiers to disperse a crowd of demonstrators massed in front of the Municipal Building of Escalante, Negros Occidental, with the use first, of water spewed from fire hoses, and later, tear gas. Eventually there was gunfire. Within moments, rallyists lay dead on and by the National Road. The fatalities numbered fifteen (15), according to the military officers; twenty-nine (29), according to the demonstrators. In the afternoon of that day, Escalante Town Mayor Braulio Lumayno, in view of the absence of the Municipal Circuit Court Judge (Emilio Ignalaga), took cognizance of a complaint filed by the Military Station Commander charging some of the rallyists with the felony of inciting to sedition, and after avowedly conducting an investigation of the witnesses presented by the complainant, issued an order for the arrest of certain of the demonstrators. His order reads as follows: It appearing that the Presiding Municipal Circuit Court Judge, Hon. Emilio M. Ignalaga, is on official leave of absence and, in the interest of justice, the undersigned has to urgently act on the complaint filed by the Station Commander, against the above-named accused for "INCITING TO SEDITION" and, on the basis of the evidence submitted after a searching question and answer were conducted and, being satisfied that said crime has been committed, in order not to frustrate the ends of justice, it is necessary that the above-named accused be placed under custody. Let therefore, a warrant of arrest be issued for said above-named accused. Bail recommended: P12,000.00.
2

In the record of the Court a quo appear the following inter alia: 1) Complaint for "Inciting to Sedition" (RPC 142, as and by PD 183 and PD 1974), signed by a Capt. Jugan, and sworn to before Mayor Lumayno on Sept. 20, 1985, bearing the stamped notation of filing with the MTC: "9/24/85, 4:00 3 PM:" 2) 2-page Affidavit of Capt. Sanson, dated, and sworn to before Actg.. City Fiscal Abros (Cadiz City) on Sept. 20, 4 1985; 3) 3-page document, "Searching Questions and Answers" signed by Capt. Sanson, dated. and sworn to before Mayor 5 Lumayno on, September 20, 1985; 4) 3-page sworn statement of Godofredo Hoyo-A y Jayme, General Manager of the Balintawak-Escalante Water 6 District; 5) Affidavit of Leopoldo Villalon;
7

6) Affidavit of Elpidio Carbajosa; and 7) Affidavit of Eduardo Flores.


9

The gist of the testimony of Capt. Sanson and the other affiants is that on September 20, 1985, the demonstrators, numbering "about 1,000, " had blockaded the main highway in front of the Escalante Municipal Building, by massing themselves on the road as well as by piling stones, coconut trunks and pieces of wood in the middle of the highway. They were also "shouting invectives, seditious and scurrilous words against the government." Negotiations with Ponsica, Chairman of the Escalante Chapter of "BAYAN" (Bagong Alyansang Makabayan), to have the road cleared having been unavailing, firemen on firetrucks began to train a "torrent of water" from their fire hoses on the demonstrators. The rallyists retaliated by hurling stones at the firemen. One of them "approached the security of the firetruck and stabbed him." Others climbed aboard the trucks and tried to grab the firehoses and firearms of the officers. At this point, on Capt. Sanson's orders, his "back-up teams" of soldiers commenced to throw tear gas at the crowd. One of the demonstrators picked up a tear gas canister and hurled it back at the soldiers. At the same time gunfire from "different assorted firearms" emanated from the rallyists; and some of the shots hit the blinker of a firetruck and the headlight of another. The soldiers shot back. This exchange of gunfire resulted in "fourteen (14) demonstrators killed on the spot. " Recovered at the scene were a rifle; a U.S. 45 cal. pistol; 2 "homemade" pistols; 14 steel arrows and 4 assorted slings; 30 assorted knives; a "pogakhang" with 2 live cartridges; 2 grenades; and several empty shells of different caliber. Hours later, another corpse, Identified as that of a demonstrator, too, was brought to the PC Headquarters. The petitioners however give a different version of the facts. What happened, according to them, was that at 9 o'clock in the morning on that day, a group of demonstrators, "composed mostly of laid-off sugar field workers," gathered in front of Escalante Municipal Hall "in the exercise of their constitutionally guaranteed right to freedom of expression and to assemble peacefully to petition the 10 government for redress of grievances." About an hour afterwards, fire trucks arrived one after another, as well as jeeploads of soldiers and CHDF members, in full combat gear, Shortly after noon, after "going thru the motions of negotiating with the demonstrators," the military officers ordered the crowd to disperse; but without warning, fire hoses were trained on and sprayed water at the demonstrators. When the rallyists did not budge, tear gas canisters were thrown at them. A demonstrator picked up a canister and threw it at an "empty space in the plaza" The soldier and CHDF members thereupon fired indiscriminately at the crowd, killing 29 and injuring 11 at least 30 persons. After the Mayor had referred the case to Municipal Trial Court Judge 12 Ignalaga on September 24, 1985, an "URGENT MOTION TO QUASH WARRANT OF ARREST" was filed on September 26, 1985 by

petitioners' counsel on the ground that a mayor no longer has authority to conduct preliminary investigations or issue warrants of arrest that authority having been "withheld in the 1985 New Rules 13 on Criminal Procedure." This was opposed by the Station 14 Commander who invoked Section 143 of the Local Government Code providing that "(i)n case of temporary absence of the Judge assigned to the municipality, the mayor may conduct the preliminary examination in criminal cases when, in his opinion the investigation cannot be delayed without prejudice to the interest of justice." A reply was filed by the petitioners after their receipt of the opposition 15 "only last October 8, 1985." They contended that the "power of the municipal mayor to conduct preliminary investigation and issue a warrant of arrest under the 1964 Revised Rules of Court** (had been) impliedly repealed by the 1985 New Rules on Criminal Procedure;" that "the 1985 New Rules on Criminal Procedure being a special law, controls over provisions of the Local Government Code (BP 337, 1983), which is a general law;" and in any case, "subject t warrants of arrest should be reviewed and revoked as done without observance of legal requisites." By Resolution dated Oct. 11, 1985, the Judge confirmed the mayor's arrest order. He opined that in the absence of the judge, the mayor still has authority to conduct preliminary investigations and issue arrest warrants, since Rule 112, Sec. 2 (d), of the 1985 Rules, includes as among those authorized to conduct preliminary investigations, "Such other officers as may be authorized by law;" and the Local G government Code of 1983, Section 143, grants a town mayor authority to conduct preliminary examinations in case of the temporary absence of the judge when such investigation cannot be delayed without prejudice to the interest of justice. The Judge declared that in the case at bar, the mayor had conducted the examination personally, and having in the exercise of his discretion found probable cause, issued the warrants of arrest in question; and conceding arguendo irregularity in that the preliminary examination was conducted without according the parties the assistance of lawyers (contrary to par. 2, Sec. 143, Chap. 3, Title 2, Book 11, Local Government Code), this does not render the proceedings void because at any rate, the mayor had duly observed the uniform procedure under PD 91 (citing: People v. Paran, 52 Phil. 712; Hashim v. Boncan, 71 Phil. 216; Lino v. Fugoso, 77 Phil. 933). It is Mayor Lumayno's order for the petitioners' arrest of September 20, 1985, and Jude Ignalaga's Order of October 11, 1985 "validating that order of arrest" that the petitioners would have this Court 17 nullify and perpetually enjoin. In their petition for certiorari and prohibition filed on October 15, 18 1985, and their Memorandum of January 21, 1986, they assert that: 1) while Section 3,Rule 112 of the l964 Rules of Court,empowers the municipal mayor, "in case of temporary absence of both the municipal and the auxiliary municipal judges from the municipality, town or place wherein they exercise their jurisdiction, to make the preliminary examination in criminal cases when such examination cannot be delayed without prejudice to the interest of justice," that power has been removed from him by the 1985 New Rules on Criminal Procedure which "do not mention the Municipal Mayor as among the officers authorized to conduct preliminary investigation (Section 2, Rule 112), much less to issue an order of arrest 19 (Section 6, Ibid.) 2) Section 143 of BP Blg. 337 (Local Government Code) under which the order of arrest in question is presumably made to rest-cannot withstand the "constitutional test" of 20 Section 3, Article IV, which safeguards the right against unreasonable searches and seizures, and requires the interposition of a "judge, or such other responsible officer as may be authorized by law," meaning "a neutral and detached magistrate competent to determine probable cause (Shadwick v. City of Tampa, 40 LW 4758; Castillo v. Jias, 62 SCRA 124; Ang Tibay v. CIR, 69 Phil. 635; Zambales Chromite, etc. v. C.A., 934 SCRA 2617);" and since a mayor is obviously not such a magistrate (U.S. v. Chadwick, 433 U.S. 197), the orders of arrest at bar are constitutionally
16

infirm; moreover, said Section 143 of the Local Government Code is "merely a rule of procedure** (and is thus) deemed to have been superseded by the New Rules 22 of Criminal Procedure; 3) the Mayor's examination "falls short of the requirements of "searching questions and answers;" the statements of the witnesses supposedly interrogated by the mayor, are either merely conclusions of law or sterile as regards seditious utterances, hence, probable cause 23 was non-existent and 4) Article 142 of the Revised Penal Code, as amendedunder which the petitioners are charged is based on the US Sedition Act of 1978, which has been declared by the US Supreme Court as "repugnant to the constitutional guarantee of freedom of speech and expression (New York Times Co. v. Sullivan, 376 U.S. 254);" hence, said Article 142 is also fatally flawed and therefore, the warrant. of arrest in question was in effect issued for a "crime which in the context of the constitutionally guaranteed freedom of 24 speech and expression does not exist." In his comment filed with this Court, Judge Ignalaga argues that 1) The validity of the preliminary examination on the basis of which the arrest warrants were issued, may not be raised for the first time in the Supreme Court, but should first be ventilated before the RTC having cognizance of the 26 crime. In any case, upon the considerations set out in his resolution of October 11, 1985, the preliminary 27 examination in question is valid. 2) The petitioners raise factual issues which are best left for determination by the RTC, the Supreme Court not 28 being a trier of facts. 3) The Mayor in fact followed the procedure in the 29 constitution prior to issuing the arrest warrants. 4) The constitutionality of PD 1974 should be assailed by 30 separate petition. The Solicitor General at the time, Estelito Mendoza, also filed a 31 comment on December 6, 1985. The comment addressed itself only to the issue of "the validity of the law authorizing municipal mayors to issue warrants of arrest and the law punishing inciting to sedition," in view of the fact that the respondents had presented 32 their own separate comments on the petition. The comment points out that: 1) Section 3, Article IV, of the 1973 Constitution, which mentions a "judge, or such other responsible officer as may be authorized by law" as vested with competence to conduct preliminary investigations, is an innovation. In implementation thereof, BP Blg. 337 was enacted on February 10, 1983, empowering mayors to conduct preliminary investigations (Sec. 143). This is a recognition of the truism that the determination of probable cause is but a quasi judicial function Ocampo v. US, 58 LED 33 1231). Petitioners' American authorities are not applicable: the mayor is the highest official in the municipality; he exercises only general supervision over the police but is not directly involved in police work; the old rules precisely expressed a recognition of the capability of i mayors to determine probable cause, and the omission of mayors in the 1985 Rules simply means that the determination of officers who may be authorized to conduct preliminary investigations was deemed best 34 left to legislation. 2) The Philippine sedition law is not akin to the US Sedition Law; in any event our own sedition law has passed the test 35 of constitutionality (Espuelas v. People, 90 Phil. 524).
25

21

Solicitor General Sedfrey A. Ordonez subsequently declared that he "stands by the constitutionality of the statutes the petitioners question and, therefore, sustains the position taken by his 36 predecessor in office;" this, in response to this Court's resolution dated May 15, 1986, requiring the parties to state whether 37 supervening events had transpired materially affecting the case. It is clear from the outset that the issue before this Court does not involved the adjudgment of the guilt or innocence of the soldiers in the tragic and regrettable killings in front of the Escalante Town Hall in the early afternoon of that day, the twentieth of September, 1985, an event that caused a great outcry of lamentation and condemnation throughout the land. This is a matter that should be and is in fact now subject of a separate criminal proceeding. Neither is the propriety of the victims' exercise of their constitutional rights of free speech and free assembly for redress of grievances in the premises at issue here. The basic question before the Court is divorced of the drama and the passion of those issues; it deals mainly with the dry, unexciting, but nonetheless important matter of whether or not the municipal mayor has the power to conduct preliminary investigations in the light of the 1985 amendments of the rules governing criminal procedure in the Rules of Court. The answer to the question entails a re-examination and analysis of the relevant legal provisions. The 1964 Rules of Court explicitly gave the mayor authority to conduct preliminary investigations. SEC. 3. Preliminary examination by the municipal mayor. In case of temporary absence of both the municipal and the auxiliary municipal judges from the municipality, town, or place wherein they exercise their jurisdiction, the municipal mayor shall make the preliminary examination in criminal cases when such examination cannot be delayed without prejudice to the interest of justice. He shall make a report of any preliminary examination so made to the municipal or to the auxiliary municipal judge immediately upon the return of one or the other. He shall have authority in such cases to order the arrest of the defendant and to grant him bail in the manner and cases 38 provided for in Rule 114. The Rules of 1940 contained an Identical provision, in Section 3, Rule 108. The 1985 Rules on Criminal Procedure did not reproduce this provision, and did not include the mayor in the enumeration of the officers authorized to conduct preliminary investigations, those listed being judges of municipal trial courts and municipal circuit trial courts; city or provincial fiscals and their assistants; national and regional state prosecutors; and "such other officers as may be 39 authorized by law." SEC. 6. When warrant of arrest may issue. (a) By the Regional Trial Court. Upon the filing of an information, the Regional Trial Court may issue a warrant for the arrest of the accused (b) By the Municipal Trial Court. If the municipal trial judge conducting the preliminary investigation is satisfied after an examination in writing and under oath of the complainant and his witnesses in the form of searching questions and answers, that a probable cause exists and that there is a necessity of placing the respondent under immediate custody in order not to frustrate the ends of 40 justice, he shall issue a warrant of arrest. (n). It is of course the deletion in the 1985 rules of reference to the municipal mayor as among those authorized to conduct preliminary investigations and order arrests upon which the petitioners chiefly rely as basis for their impugnation of Mayor Lumayno's preliminary investigation and order of arrest based thereon.

The matter is however treated of not solely by the Rules of Court but also by the 1973 Constitution, Section 3, Article IV whereof reads: SEC. 3. The right of the people to be secure in their persons, houses, papers, and effects against unreasonable searches and seizures of whatever nature and for any purpose shall not be violated, and no search warrant or warrant of arrest shall issue except upon probable cause to be determined by the judge, or such other responsible officer as may be authorized by law, after examination under oath or affirmation of the complainant and the witnesses he may produce, and particularly describing the place to be searched, and the persons or things to be 41 seized. Parenthetically, it may be noted that Section 3, Rule III of the 1935 Constitution mentioned only "the judge" as having power to determine probable cause and issue search and arrest warrants. The matter is further dealt with in Section 143 of Batas Pambansa Bilang 337, otherwise known as the Local Government Code, which took effect on February 10, 1983. This section evidently deems the mayor a "responsible officer" in contemplation of the cited constitutional provision, and explicitly authorizes him to conduct preliminary examination in criminal cases and order the arrest of the accused upon probable cause. SEC. 143. Authority of the Mayor to Conduct Preliminary Examination. (1) In case of temporary absence of the judge assigned to the municipalities the mayor may conduct the preliminary examination in criminal cases when, in his opinion the investigation cannot be delayed without prejudice to the interest of justice. (2) No examination shall be conducted unless the parties are assisted by lawyers. (3) In cases where he may conduct preliminary examination, the mayor shall upon probable cause after examination of witnesses, have authority order the arrest of the accused and to grant him bail in the manner and cases provided in the Revised Rules of Court and order his provisional release. (4) The mayor shall make a report of any preliminary examination so made immediately after the return of the judge assigned in the area, or upon the designation of his 42 replacement. The 1973 Constitution plainly and unmistakably grants to the legislature the power to determine which "responsible officers, " aside from judges, may issue warrants of arrest after examination under oath or affirmation of the complainant and the witnesses he may produce. And as plainly and unmistakably, the legislature, the Batas Pambansa, has in the Local Government Code made the determination that the mayor is such a "responsible officer "and has in consequence authorized him to conduct preliminary investigations in criminal cases and order the arrest of the accused upon a finding of probable cause. The first issue raised by the petitioners that in September ,1985 the mayor no longer had power to conduct preliminary 43 investigations and issue arrest warrants must therefore be resolved against them. The argument that Section 143 of the Local Government Code is just "a rule of procedure merely having incorporated Rule 112, Section 2 of the old rules of Criminal Procedure," and should therefore be deemed "superseded by the New Rules of Criminal Procedure pursuant to the power of this ** Court to promulgate rules of procedure (Article X, Section 5 151, 44 Constitution **)" cannot be sustained. However superior the Supreme Court may be to the other branches of government in the realm of adjudication, its power to ordain rules of court was at the

time inferior to the lawmaking power of the legislature. It is true that the 1935 Constitution repealed all procedural laws then in force "as statutes" and declared them to be "rules of court;" but this was only so that they could be subject to repeal or modification by the Supreme Court, which was given the power to promulgate rules of 46 procedure, and has since been exercising such power by promulgating the Rules of Court of 1940 and of 1964, and the 1985 Rules on Criminal Procedure, etc. But the 1935 and 1973 Constitutions explicitly conferred on the legislature the power to repeal, alter or supplement those rules of court, although it would appear that that power is no longer granted to it by the 1987 47 Charter. Now, the conditions under which the mayor could conduct preliminary investigations are also clearly indicated by the law, to wit: 1) in case of the temporary absence of the judge assigned to the municipality; and 2) in his (the mayor's) opinion, the investigation cannot be delayed without prejudice to the interests of justice. On these occasions, the mayor may order the arrest of the accused upon a finding of probable cause after searching examination of the complainant and the witnesses the latter may present. In the case at bar, there is no showing that these conditions have not been met. Indeed, quite apart from the presumption that official 48 duty has been regularly performed. and the affirmative statements of respondent official that on that fateful 20th day of September, 49 1985, Judge Ignalaga was in Manila" on official leave or 50 absence," the petitioners have not put the fulfillment of those conditions at issue at all, or the matter of whether or not the accused were "assisted by lawyers" in the course of the 51 investigation. The petitioners also challenge the constitutionality of Section 143 of the Local Government Code, it being claimed that the mayor has been thereby authorized to conduct pre investigations and issue warrants of arrest, although he can not be deemed a "neutral and detached magistrate" within the contemplation of Section 3, Article 52 IV of the 1973 Constitution. The competence of the Batasang Pambansa to decide and declare by statute which "responsible officers," aside from judges, should be entrusted with the authority to conduct preliminary investigations and issue arrest warrants cannot be doubted in view of the clear 53 language of the Constitution. And the determination by the Batasan that a municipal mayor is one such "responsible officer" who may properly be entrusted with the function of conducting preliminary investigations and ordering arrests of suspects upon probable cause, can not be subject of judicial review, absent any indication that the legislative proceedings leading to that statutory determination are void on account of some grave cause. Certainly, the wisdom of the statute, or the validity of the reasons underlying it, or the adequacy of the statistics, facts and circumstances considered by the legislature in its enactment, are beyond the 54 sphere of inquiry of the courts The Batasan was apparently of the belief that a municipal mayor could be sufficiently objective and impartial as to be relied upon to conduct preliminary investigations and issue orders of arrest in the exceptional situation when the judge assigned in the municipality was absent. The petitioners disagree. They contend that the mayor cannot in the very nature of things be "neutral and detached." The disagreement, and the fact that plausible reasons may be adduced by one side or the other on the proposition does not make the question a justiciable one. The theory advocated by the petitioners that the mayor's "deep involvement in law enforcement functions is likely to color his 55 judgment as a trier of probable cause," does not induce persuasion. In the first place the premise cannot be conceded. While it is true that the mayors do "exercise general supervision over units and elements of the INP stationed or assigned in their respective jurisdictions," they are not themselves directly involved in police work and cannot in any sense be described, as the petitioners do, as being deeply involved in law enforcement functions. And even if that

45

"deep involvement" be conceded, it does not follow that this would necessarily preclude their assuming "the cold neutrality of an impartial judge" in conducting preliminary investigations of persons suspected of crimes. But it must be emphasized here and now that what has just been described is the state of the law as it was in September, 1985. The law has since been alter. No longer does the mayor have at this time the power to conduct preliminary investigations, much less issue orders of arrest. Section 143 of the Local Government Code, conferring this power on the mayor has been abrogated, rendered functus officio by the 1987 Constitution which took effect on February 2, 1987, the date of its ratification by the Filipino people. Section 2, Article III of the 1987 Constitution pertinently provides that "no search warrant or warrant of arrest shall issue except upon probable cause to be determined personally by the judge after examination under oath or affirmation of the complainant and the witnesses he may produce, and particularly describing the place to be searched and the person or things to be seized. " The constitutional proscription has thereby been manifested that thenceforth, the function of determining probable cause and issuing, on the basis thereof, warrants of arrest or search warrants, may be validly exercised only by judges, this being evidenced by the elimination in the present Constitution of the phrase, "such other responsible officer as may be authorized by law" 56 found in the counterpart provision of said 1973 Constitution, who, aside from judges, might conduct preliminary investigations and issue warrants of arrest or search warrants. As the law now stands, the mayor may no longer conduct preliminary investigation, the authority to do so being limited under Section 2, Rule 1 1 2 of the Rules of Court to (1) provincial or city fiscals and their assistants; (2) judges of the Municipal Trial Courts and Municipal Circuit Trial Courts; (3) national and regional state prosecutors; and (d) such other officers as may be authorized by 57 law. But only "the judge" may issue search and arrest warrants 58 after due determination of probable cause. The petitioners' assault, on the other hand, upon the constitutionality of Article 141 of the Revised Penal Code, defining and penalizing the felony of inciting to sedition, upon the claim that it was "borrowed" from the U.S. Sedition Act of 1798 which in turn has been struck down as inconsistent with the First Amendment of 59 the American Constitution, is repelled by the Solicitor General's 60 arguments. In the first place, ** our law on inciting to sedition is not akin to the US Sedition Act of 1798 which was imposed on the American colonies by their British ruler. With the success of the American revolution, the 1798 Sedition Act naturally ceased to have effect as it would be utterly incongrous to punish those who sought the overthrow of the British government in America. xxx xxx xxx xxx xxx xxx

To annul our law on sedition is to give license to those who seek the application of lawless methods in the advancement of their political views. Our constitution surely does not contemplate this. Finally, the petitioners postulate that in the determination of the existence of probable cause from the constitutional aspect, it is required that: "(1) The judge (or) officer must examine the ** witnesses personally; (2) The examination must be under oath; and (3) The examination must be reduced to writing in the form of searching questions and answers. (Marinas v. Siochio, 104 SCRA 403)." In the light of these principles they contend: firstly, that the mayor's questioning of the witnesses was not "searching" enough; and secondly, that the witnesses' testimony does not establish prima facie the commission of the felony of inciting to sedition.

The fact is that is shown by the record, questions about the material events were in truth propounded by Mayor Lumayno to the chief 61 witness. Capt. Sanson; and no proof to the contrary has been submitted. The circumstance that the answers given by Capt. Sanson to the mayor's questions are closely reflective of the contends of his affidavit should not come as a surprise and cannot, without more, be taken as debilitating or nullifying the interrogation. The petitioner's second point is much more substantial and is decisive of the controversy. The petitioners are correct in their claim 62 that Capt. Sanson's testimony does not in truth contain any facts demonstrating the actuality of the crime of inciting to sedition, which is the crime for which arrest warrants were issued by Mayor Lumayno. The facts recited by Capt. Sanson may perhaps warrant a charge of traffic obstruction, or grave coercion, or malicious mischief, or illegal possession of firearms or deadly weapons, or maybe, attempted murder or homicide. But it is barren of facts to support an accusation of inciting to sedition. The other evidence on record exhibits the same barreness. Two witnesses, Godofredo Hoyo-A and Elpidio Carbajosa, advert in 63 their affidavits to the shouting of "seditious words," etc. Hoyo-A is quoted as deposing that "the demonstrators were already unruly ** (and kept) on shouting anti-government slogans, ** invectiues and 64 seditious words against the government," that certain of the petitioners were Identified as being "against the government, ** and using the (Mt. Carmel) School in doctrining (sic) the students against 65 the government," that petitioner Rogelio Arnaiz had delivered a speech in which he had said, "Rumpagon ang dictatorial na regimen ni Pres. Marcos, pamatukan ang NSL, ibagsak ang military kagwalaon ang CHDF, suklan nato ang gobyerno," a woman speaker had said the same thing, and a third speaker, petitioner Daniel Gempesala, also stated: Lumpagon and dictatorial na regimen ni 66 Marcos, kag ibagsak ang Gobyerno;" and they had elicited a response from the people there who had raised their right hands 67 and repeated the shouted "slogan." Carbajosa in his turn declared that the demonstrators ** were shouting for trouble and ready to 68 make revolution with the use of arms." Another affiant, Eduardo Flores, stated that the demonstrators were unruly, "tumultuously shouting seditious words against the 69 government and shouting for revolution." The sworn statement of the only other witness, Leopoldo 70 Villalon, is totally innocuous as far as proving the elements of inciting to sedition is concerned. The evidence can not justify the action taken by the respondent Mayor and Judge. The Court thus declares as sorely inadequate and mortally defective the avowed evidentiary foundation for Mayor Lumayno's finding of probable cause respecting the commission by the petitioners of the crime of inciting to sedition. The affiants declarations in their sworn statements which might otherwise be pertinent to the offense, are generalities, mere conclusions of their, not positive averments of particular facts within their personal knowledge. 'They do not Identify the specific persons supposed to have perpetrated the crime charged, except two. But even the Identification of these two is of no moment. For except as regards Capt. Sanson, whose testimony, to repeat, is in any case ineffectual to prove the precise offense ascribed to the petitioners, there had been no searching interrogation by Mayor Lumayno of the witnesses as required by the Constitution. Hence, whatever credit could possibly have been accorded to the affidavit of Hoyo-A which ventures to quote the exact words allegedly shouted by petitioners Arnaiz and Gempesala, and an unidentified woman was thereby effectively foreclosed. WHEREFORE, the writs of certiorari and prohibition are granted. The order of respondent Mayor Lumayno issued on September 20, 1985 and the resolution promulgated by respondent Judge Ignalaga on October 11, 1985 are annulled and set aside, and the respondents are perpetually forbidden to enforce or in any way implement the orders for the arrest of any of the petitioners. No costs.

Teehankee, C.J, Yap, Fernan, Melencio-Herrera, Gutierrez, Jr., Cruz, Paras, Feliciano, Gancayco, Padilla, Bidin and Cortes, JJ., concur. Sarmiento, J., took no part.

Particularity of Description CASES: Stonehill v. Diokno, supra Alvarez v. CFI, 64 SCRA 33 Burgos v. Chief of Staff, supra Nolasco v. Pano, 139 SCRA 152 FACTS: Milagros Aguilar-Roque was arrested together with Cynthia Nolasco by the Constabulary Security Group (CSG). Milagros had been wanted as a high ranking officer of the CPP. The arrest took place at 11:30 a.m. of August 6, 1984. At noon of the same day, her premises were searched and 428 documents, a portable typewriter and 2 boxes were seized. Earlier that day, Judge Cruz Pao issued a search warrant to be served at Aguilar-Roques leased residence allegedly an underground house of the CPP/NPA. On the basis of the documents seized, charges of subversion and rebellion by the CSG were filed by but the fiscals office merely charged her and Nolasco with illegal possession of subversive materials. Aguilar-Roque asked for suppression of the evidence on the ground that it was illegally obtained and that the search warrant is void because it is a general warrant since it does not sufficiently describe with particularity the things subject of the search and seizure, and that probable cause has not been properly established for lack of searching questions propounded to the applicants witness. ISSUE: WON the search warrant was valid? HELD: NO. Section 3, Article IV of the Constitution, guarantees the right of the people to be secure in their persons, houses, papers and effects against unreasonable searches and seizures of whatever nature and for any purpose. It also specifically provides that no Search Warrant shall issue except upon probable cause to be determined by the Judge or such other responsible officer as may be authorized by law, after examination under oath or affirmation of the complainant and the witnesses he may produce, and particularly describing the place to be searched and the things to be seized. It is at once evident that the foregoing Search Warrant authorizes the seizure of personal properties vaguely described and not particularized. It is an all- embracing description which includes everything conceivable regarding the Communist Party of the Philippines and the National Democratic Front. It does not specify what the subversive books and instructions are; what the manuals not otherwise available to the public contain to make them subversive or to enable them to be used for the crime of rebellion. There is absent a definite guideline to the searching team as to what items might be lawfully seized thus giving the officers of the law discretion regarding what articles they should seize as, in fact, taken also were a portable typewriter and 2 wooden boxes. It is thus in the nature of a general warrant and infringes on the constitutional mandate requiring particular description of the things to be seized. In the recent rulings of this Court, search warrants of similar description were considered null and void for being too general. 20 Century Fox v. CA, 164 SCRA 655 I. THE FACTS Petitioner 20 Century Fox Film Corporation sought the assistance of the NBI in conducting searches and seizures in connection with the NBIs anti-film piracy campaign. Petitioner alleged that certain videotape outlets all over Metro Manila are
th th

engaged in the unauthorized sale and renting out of copyrighted films in violation of PD No. 49 (the old Intellectual Property Law). The NBI conducted surveillance and investigation of the outlets pinpointed by the petitioner and subsequently filed three (3) applications for search warrants against the video outlets owned by the private respondents. The lower court issued the desired search warrants. The NBI, accompanied by the petitioner's agents, raided the video outlets and seized the items described in the three warrants. Private respondents later filed a motion to lift the search warrants and release the seized properties, which was granted by the lower court. Petitioners motion for reconsideration was denied by the lower court. The CA affirmed the trial court. II. THE ISSUE Did the judge properly lift the search warrants he issued earlier? III. THE RULING [The Court DISMISSED the petition and AFFIRMED the questioned decision and resolution of the CA.] YES, the judge properly lifted the search warrants he issued earlier. The lower court lifted the three (3) questioned search warrants in the absence of probable cause that the private respondents violated P.D. 49. NBI agents who acted as witnesses during the application for search warrant did not have personal knowledge of the subject matter of their testimony, which was the alleged commission of the offense of piracy by the private respondents. Only the petitioners counsel who was also a witness during the application stated that he had personal knowledge that the confiscated tapes owned by the private respondents were pirated tapes taken from master tapes belonging to the petitioner. The lower court lifted the warrants, declaring that the testimony of petitioners counsel did not have much credence because the master tapes of the allegedly pirated tapes were not shown to the court during the application. The presentation of the master tapes of the copyrighted films, from which the pirated films were allegedly copied, was necessary for the validity of search warrants against those who have in their possession the pirated films. The petitioner's argument to the effect that the presentation of the master tapes at the time of application may not be necessary as these would be merely evidentiary in nature and not determinative of whether or not a probable cause exists to justify the issuance of the search warrants is not meritorious. The court cannot presume that duplicate or copied tapes were necessarily reproduced from master tapes that it owns. The essence of a copyright infringement is the similarity or at least substantial similarity of the purported pirated works to the copyrighted work. Hence, the applicant must present to the court the copyrighted films to compare them with the purchased evidence of the video tapes allegedly pirated to determine whether the latter is an unauthorized reproduction of the former. This linkage of the copyrighted films to the pirated films must be established to satisfy the requirements of probable cause. Mere allegations as to the existence of the copyrighted films cannot serve as basis for the issuance of a search warrant. Columbia Pictures v. Flores, 223 SCRA 761 I. THE FACTS As a consequence of a complaint filed by the Motion Association of America, Inc., NBI agents

conducted surveillance operations on certain video establishments, among them respondent FGT Video Network, Inc. (FGT), forunauthorized sale, rental, reproduction and/or disposition of copyrighted film," a violation of PD 49 (the old Intellectual Property Law). After an NBI agent was able to havecopyrighted motion pictures Cleopatra (owned by 20th Century Fox) and The Ten Commandments (owned by Paramount) reproduced in video format in FGT, the NBI applied for and was able to obtain from the respondent judge the subject Search Warrant No. 45 which reads: TO ANY PEACE OFFICER: GREETINGS: It appearing to the satisfaction of the Undersigned after examining under oath NBI Senior Agent Lauro C. Reyes and his witnesses Mr. Danilo Manalang and Ms. Rebecca Benitez-Cruz, that there is a probable cause to believe that Violation of Section 56 P.D. No. 49 as amended by P.D. No. 1988 (otherwise known as the Decree on Protection of Intellectual Property) has been committed and that there are good and sufficient reasons to believe that FGT Video Network, Inc., Manuel Mendoza, Alfredo C. Ongyanco, Eric Apolonio, Susan Yang and Eduardo Yotoko are responsible and have in control/possession at No. 4 Epifanio de los Santos corner Connecticut, Greenhills, San Juan, Metro Manila (per attached sketch and list of MPAA member Company Titles) the following properties to wit: (a) Pirated video tapes of the copyrighted motion pictures/films the titles of which are mentioned in the attached list; (b) Posters, advertising leaflets, flyers, brochures, invoices, lists of titles being reproduced or retaped, journals, ledgers, jon (sic) order slips, delivery slips and books of accounts bearing and/or mentioning the pirated films with titles (as per attached list), or otherwise used in the reproduction/retaping business of the defendants; (c) Television sets, video cassette recorders, rewinders, tape head cleaners, accessories, equipment and other machines and paraphernalia or materials used or intended to be used in the unlawful sale, lease, distribution, or possession for purpose of sale, lease, distribution, circulation or public exhibition of the abovementioned pirated video tapes which they are keeping and concealing in the premises above-described, which should be seized and brought to the Undersigned. You are hereby commanded to make an immediate search at any time in the day between 8:00 A.M. to 5:00 P.M. of the premises above-described and forthwith seize and take possession of the above-enumerated personal properties, and bring said properties to the undersigned immediately upon implementation to be dealt with as the law directs. In the course of the implementation of the search warrant in the premises of FGT, the NBI agents found and seized various video tapes of copyrighted films owned and exclusively distributed by petitioners. Also seized were machines and equipment, television sets, paraphernalia, materials, accessories, rewinders, tape head cleaners, statements of order, return slips, video prints, flyers, production orders, and posters. FGT moved for the release of the seized television sets, video cassette recorders, rewinders, tape head cleaners, accessories, equipment and other machines or paraphernalia seized by virtue of the subject warrant. It argued that as a licensed video reproducer, it had the right possess the seized reproduction equipment, which are not illegal per se, but are rather exclusively used and intended to be used for reproduction and not in the sale, lease, distribution or possession for purposes of sale, lease distribution, circulation or public exhibition of pirated video tapes. Finding that FGT was a registered and duly licensed distributor and in certain instances and under special instructions

Picture

and conditions reproducer of videograms and that, therefore, its right to possess and use the seized equipment had been placed in serious doubt, the lower court ordered the return of the television sets, video cassette recorders, rewinders, tape head cleaners, accessories, equipment and other machines or paraphernalia to FGT.

pirated video tapes of the copyrighted motion pictures/films were reproduced. As already stated, FGT Video Network, Inc. is a registered and duly licensed distributor and in certain instances and under special instructions . . . reproducer of videograms, and as such, it has the right to keep in its possession, maintain and operate reproduction equipment(s) and paraphernalia(s). Pp v. Tee, G.R. No. 140546-47, January 20, 2003 Facts The case involves an automatic review of judgment made against Tee who was convicted for illegal possession of marijuana and sentenced to death. The defense assailed the decision of the court for taking admissible as evidence the marijuana seized from the accused by virtue of allegedly general search warrant. They further contend that the accused was deprived of his right to speedy trial by failure of the prosecution to produce their witness who failed to appear during the 20 hearing dates thereby slowing down the trial procedure.

II. THE ISSUE Did the respondent judge act with grave abuse of discretion amounting to lack of jurisdiction in ordering the immediate return of some of the items seized by virtue of the search warrant?

III. THE RULING [The High Tribunal DISMISSED the petition and AFFIRMED the order of the respondent Judge Flores.] NO, the respondent judge DID NOT act with grave abuse of discretion amounting to lack of jurisdiction in ordering the immediate return of some of the items seized by virtue of the search warrant. Search Warrant No. 45 fails to satisfy the test of legality. This is more so because the Court has previously decided a case dealing with virtually the same kind of search warrant. In 20th Century Fox vs. CA, the Court upheld the legality of the order of the lower court lifting the search warrant issued under circumstances similar to those obtaining in the case at bar. A striking similarity between this case and 20th Century Fox is the fact that Search Warrant No. 45, specifically paragraph (c) thereof describing the articles to be seized, contains an almost identical description as the warrant issued in the 20th Century Fox case, to wit: (c) Television sets, Video Cassettes Recorders, rewinders, tape head cleaners, accessories, equipments and other machines used or intended to be used in the unlawful reproduction, sale, rental/lease, distribution of the above-mentioned video tapes which she is keeping and concealing in the premises above-described. On the propriety of the seizure of the articles abovedescribed, the Court held in20th Century Fox: Television sets, video cassette recorders, rewinders and tape cleaners are articles which can be found in a video tape store engaged in the legitimate business of lending or renting out betamax tapes. In short,these articles and appliances are generally connected with, or related to a legitimate business not necessarily involving piracy of intellectual property or infringement of copyright laws. Hence, including these articles without specification and/or particularity that they were really instruments in violating an AntiPiracy law makes the search warrant too general which could result in the confiscation of all items found in any video store. The language used in paragraph (c) of Search Warrant No. 45 is thus too all-embracing as to include all the paraphernalia of FGT in the operation of its business. As the search warrant is in the nature of a general one, it is constitutionally objectionable. The Court concluded that the respondent judge did not gravely abuse his discretion in ordering the immediate release of the enumerated items, but that he was merely correcting his own erroneous conclusions in issuing Search Warrant No. 45. This can be gleaned from his statement that . . . the machines and equipment could have been used or intended to be used in the illegal reproduction of tapes of the copyrighted motion pictures/films, yet, it cannot be said with moral certainty that the machines or equipment(s) were used in violating the law by the mere fact that

Issue Whether or not the substantive right of the accused for a speedy trial prejudiced during the hearing of the case. Held The court ruled that the substantive right of the accused for a fair and speedy trial was not violated. It held that the Speedy Trial Act of 1998 provides that the trial period for the criminal cases should be in general 180 days. However, in determining the right of an accused to speedy trial, courts should do more than a mathematical computation of the number of postponements of the scheduled hearings of the case.The right to a speedy trial is deemed violated only when: (1) the proceedings are attended by vexatious, capricious, and oppressive delays; or (2) when unjustified postponements are asked for and secured; or (3) when without cause or justifiable motive a long period of time is allowed to elapse without the party having his case tried. It was shown by the records that the prosecution exerted efforts in obtaining a warrant to compel the witness to testify. The concept of speedy trial is necessarily relative where several factors are weighed such as the length of time of delay, the reason of such delay, and conduct of prosecution and the accused and the prejudice and damaged caused to the accused of such delay. The court did not find the 20 days of delayed hearing unreasonable length of time as to constitute deprivation of the constitutional rights of the accused for a speedy trial in addition to the fact that court trial may be always subjected to postponement for reasonable cause of delay. In the absence of showing that the reason for delay was capricious or oppressive, the State must not be deprived of reasonable opportunity in prosecuting the accused. Microsoft Corp v. Maxicorp, Inc., G.R. No. 140946, Sept 13, 2004 Intellectual Property Law on Copyright Probable Cause in Issuing Search Warrant In 1996, Dominador Samiano, an agent of the National Bureau of Investigation (NBI) conducted a surveillance against Maxicorp Inc. He observed that Microsoft Software (Windows Operating Systems) were being produced and packaged withinthe premises of Maxicorp. Samiano, together with a civilian witness (John Benedict Sacriz) then bought a computer unit from Maxicorp. The unit was pre-installed with a pirated copy of Windows. For their purchase, they were issued a receipt, however, the receipt was in the name of a certain Joel Diaz. Subsequently, Samiano applied for a search warrant before the RTC. He brought with him Sacriz as witness. He also brought the computer unit they bought as evidence as well as the receipt. He even added an additional witness (Felixberto Pante), a computer technician, who showed the judge that the software in the computer unit bought by Samiano from Maxicorp was pirated. The RTC judge, convinced that there is a probable cause fora case of copyright infringement and unfair competition committed by Maxicorp, issued the corresponding warrant. Maxicorp assailed the legality of the warrant before the Court of Appeals. The Court of

Appeals ruled in favor of Maxicorp and in its decision it highlighted the fact that the receipt issued was not in Samianos or Sacriz name hence the proceeding in the trial court was infirm from the onset. ISSUE: Whether or not the Court of Appeals is correct. HELD: No. The testimonies of the two witnesses, coupled with the object and documentary evidence they presented, are sufficient to establish the existence of probable cause. From what they have witnessed, there is reason to believe that Maxicorp engaged in copyright infringement and unfair competition to the prejudice of Microsoft. Both NBI Agent Samiano and Sacriz were clear and insistent that the counterfeit software were not only displayed and sold within Maxicorps premises, they were also produced, packaged and in some cases, installed there. The fact that the receipt issued was not in Samianos name nor was it in Sacriz name does not render the issuance of the warrant void. No law or rule states that probable cause requires a specific kind of evidence. No formula or fixed rule for its determination exists. Probable cause is determined in the light of conditions obtaining in a given situation.Thus, it was improper for the Court of Appeals to reverse the RTCs findings simply because the sales receipt evidencing NBI Agent Samianos purchase of counterfeit goods is not in his name.

merely reported by a person whom one considers to be reliable. Petitioner also assails the validity of the search warrant on the ground that it failed to particularly describe the place to be searched, contending that there were several rooms at the ground floor and the second floor of the PUP. In the present case, however, the application for search warrant was captioned: "For Violation of PD No. 1866 (Illegal Possession of Firearms, etc.) While the said decree punishes several offenses, the alleged violation in this case was, qualified by the phrase "illegal possession of firearms, etc." As explained by respondent Judge, the term "etc." referred to ammunitions and explosives. In other words, the search warrant was issued for the specific offense of illegal possession of firearms and explosives. Probable Cause

Manifestly, in the case at bar, the evidence failed to show the existence of probable cause to justify the issuance of the search warrant. The Court also notes post facto that the search in question yielded, no armalites, handguns, pistols, assorted weapons or ammunitions as stated in the application for search warrant, the supporting deposition, and the search warrant the supporting hand grenades were itself only three (3) live fragmentation found in the searched premises of the PUP, according to the affidavit of an alleged member of the searching party. The Court avails of this decision to reiterate the strict requirements for determination of "probable cause" in the valid issuance of a search warrant, as enunciated in earlier cases. True, these requirements are stringent but the purpose is to assure that the constitutional right of the individual against unreasonable search and seizure shall remain both meaningful and effective. Under Oath

In connection with one particular offense CASE: Prudente v. Dayrit, G.R. No. 82870 December 14, 1989, 180 SCRA 69

Facts: The case is a petition for certiorari to annul and set aside the order of respondent Judge dated 9 March 1988 which denied Dr. Nemesis E. Prudentes (PUP President) motion to quash Search Warrant No. 87-14, as well as his order dated 20 April 1988 denying petitioner's motion for reconsideration of the earlier order. It appears that on 31 October 1987, P/Major Alladin Dimagmaliw, Chief of the Intelligence Special Action Division (ISAD) of the Western Police District (WPD) filed with the Regional Trial Court (RTC) of Manila, Branch 33, presided over by respondent Judge Abelardo Dayrit, now Associate Justice of the Court of Appeals, an application for the issuance of a search warrant, docketed therein as SEARCH WARRANT NO. 87-14, for VIOLATION OF PD NO. 1866 (Illegal Possession of Firearms, etc.) entitled "People of the Philippines, Plaintiff, versus Dr. Nemesis E. Prudente, Defendant." In his application for search warrant, P/Major Alladin Dimagmaliw alleged the following: that in PUP he has in his control or possession firearms, explosives handgrenades and ammunition which are illegally possessed or intended to be used as the means of committing an offense. That the undersigned has verified the report and found it to be a fact, and therefore, believes that a Search Warrant should be issued to enable the undersigned or any agent of the law to take possession and bring to the Honorable Court. Issue: Whether or not the application for Search Warrant No. 87-14 filed is legal against the defendant in violation of PD No. 1866? Held: No. The questioned orders dated 9 March 1988 and 20 April 1988 as well as Search Warrant No. 87-14 were annulled and set aside. Search Warrant

In Alvarez vs. Court of First Instance, the Court laid the following test in determining whether the allegations in an application for search warrant or in a supporting deposition, are based on personal knowledge or not, stating that true test of sufficiency of a deposition or affidavit to warrant issuance of a search warrant is whether it has been drawn in a manner that perjury could be charged thereon and the affiant be held liable for damage caused. The oath required must refer to the truth of the facts within the personal knowledge of the applicant for search warrant, and/or his witnesses, not of the facts

Finally, in connection with the petitioner's contention that the failure of the applicant to state, under oath, the urgent need for the issuance of the search warrant, his application having been filed on a Saturday, rendered the questioned warrant invalid for being violative of this Court's Circular No. 19, dated 14 August 1987, which reads: That applications filed after office hours, during Saturdays, Sundays and holidays shall likewise be taken cognizance of and acted upon by any judge of the court having jurisdiction of the place to be searched, but in such cases the applicant shall certify and state the facts under oath, to the satisfaction of the judge, that the issuance is urgent. 4. Other forms of search Drug testing as search CASE: SJS v. Dangerous Drugs Board, G.R. No. 157870, November 3, 2008 Facts: In its Petition for Prohibition under Rule 65, petitioner Social Justice Society (SJS), a registered political party, seeks to prohibit the Dangerous Drugs Board (DDB) and the Philippine Drug Enforcement Agency (PDEA)from enforcing paragraphs (c), (d), (f), and (g) of Sec. 36 of RA 9165 on the ground that they are constitutionallyinfirm. For one, the provisions constitute undue delegation of legislative power when they give unbridled discretionto schools and employers to determine the manner of drug testing. For another, the provisions trench in the equal protection clause inasmuch as they can be used to harass a student or an employee deemed undesirable. And for athird, a person's constitutional right against unreasonable searches is also breached by said provisions. Issues: (1) Do Sec. 36(g) of RA 9165 and COMELEC Resolution No. 6486 impose an additional qualification for candidates for senator? Corollarily, can Congress enact a law prescribing qualifications for candidates for senator inaddition to those laid down by the Constitution? and(2) Are paragraphs (c), (d), (f), and (g) of Sec. 36, RA 9165 unconstitutional? Specifically, do these paragraphsviolate the right to privacy, the right against unreasonable searches and seizure, and the equal protection clause? Or do they constitute undue delegation of legislative power? Held: WHEREFORE

, the Court resolves to GRANT the petition in G.R. No. 161658 and declares Sec. 36(g) of RA 9165 and COMELEC Resolution No. 6486 as UNCONSTITUTIONAL ; and to PARTIALLY GRANT the petition in G.R. Nos. 157870 and 158633 by declaring Sec. 36(c) and (d) of RA 9165 CONSTITUTIONAL , butdeclaring its Sec. 36(f) UNCONSTITUTIONAL . All concerned agencies are, accordingly, permanently enjoinedfrom implementing Sec. 36(f) and (g) of RA 9165 . No costs. Ratio/Doctrine: Sec. 36(g) of RA 9165 should be, as it is hereby declared as, unconstitutional. It is basic that if alaw or an administrative rule violates any norm of the Constitution, that issuance is null and void and has no effect.The Constitution is the basic law to which all laws must conform; no act shall be valid if it conflicts with theConstitution. [8] In the discharge of their defined functions, the three departments of government have no choice butto yield obedience to the commands of the Constitution. Whatever limits it imposes must be observed. [9] It ought to be made abundantly clear, however, that the unconstitutionality of Sec. 36(g) of RA 9165 is rooted on itshaving infringed the constitutional provision defining the qualification or eligibility requirements for one aspiring torun for and serve as senator.Sec. 36(c) and (d) of RA 9165, the Court finds no valid justification for mandatory drug testing for persons accusedof crimes. In the case of students, the constitutional viability of the mandatory, random, and suspicionless drugtesting for students emanates primarily from the waiver by the students of their right to privacy when they seek entryto the school, and from their voluntarily submitting their persons to the parental authority of school authorities. Inthe case of private and public employees, the constitutional soundness of the mandatory, random, and suspicionlessdrug testing proceeds from the reasonableness of the drug test policy and requirement.We find the situation entirely different in the case of persons charged before the public prosecutor's office withcriminal offenses punishable with six (6) years and one (1) day imprisonment. The operative concepts in themandatory drug testing are "randomness" and "suspicionless." In the case of persons charged with a crime beforethe prosecutor's office, a mandatory drug testing can never be random or suspicionless. The ideas of randomnessand being suspicionless are antithetical to their being made defendants in a criminal complaint. They are notrandomly picked; neither are they beyond suspicion. When persons suspected of committing a crime are charged,they are singled out and are impleaded against their will. The persons thus charged, by the bare fact of being haled before the prosecutor's office and peaceably submitting themselves to drug testing, if that be the case, do notnecessarily consent to the procedure, let alone waive their right to privacy.

[40] To impose mandatory drug testing onthe accused is a blatant attempt to harness a medical test as a tool for criminal prosecution, contrary to the statedobjectives of RA 9165. Drug testing in this case would violate a persons' right to privacy guaranteed under Sec. 2,Art. III of the Constitution. Worse still, the accused persons are veritably forced to incriminate themselves

Properties Subject to Seizure (See Sec. 3 Rule 126, Revised Rules of Criminal Procedure) CASES: Pp v. Marcos, 117 SCRA 999 Accused was convicted of robbery with homicide. He questions sufficiency of evidence to warrant conviction. HELD: As a rule, the trial courts assessment of the credibility of witnesses and their testimonies is binding on appellate courts, absent any fact or circumstance of weight and substance that may have been overlooked, misapprehended or misapplied. In this case, the court a quo committed serious lapses which warrant the acquittal of the appellant. Uy Khetin vs. Villareal, 42 Phil 886 Facts: -On April 30, 1919, one Ramon Gayanilo, corporal of the Philippine Constabulary, presented to the judge of the Court of First Instance of Iloilo an application for search warrant, the said Ramon Gayanilo stating in hisapplication; "That in the house of Chino Uy Kheytin, Sto. Nio St., No. 20, Iloilo, under the writing desk inhis store, there is kept a certain amount of opium."- Armed with that search warrant, the respondent M. S. Torralba, on the same day (April 30th) searchedthe house of the petitioner Uy Kheytin and found therein 60 small cans of opium. They wanted to searchalso the bodega on the ground-floor of the house, but Uy Kheytin positively denied that it was his or thathe rented it. Lt. Torralba wanted to be sure, and for this reason, he placed a guard in the premises to seethat nothing was removed therefrom, and then went away to find out who the owner of the bodega was. The next morning he learned from the owner of the house, one Segovia, of the town of Molo, that theChinaman Uy Kheytin was the one who was renting the bodega. Thereupon Lt. Torralba and hissubordinates resumed the search and then and there found and seized articles which were all withconnection to the using of opium.- A criminal complaint was filed in the court of the justice of the peace of Iloilo against all the petitionersherein, charging them with a violation of the Opium Law. They were duly arrested. 7 -Defendants urged: (1) that the search warrant of April 30 was illegal because the requisites prescribed bythe General Orders No. 58 had not been complied with in its issuance (specifically (a) because it was not issued upon either of the grounds mentioned in section 96 of General Orders No. 58, and (b) because the judge who issued it did not determine the probable cause by examining witnesses under oath) ; (2) that thesearches and seizures made on May 1st had been made without any semblance of authority and henceillegal; and (3) that the seizure of the defendants' books and letters was a violation of the provisions of the Jones Law providing that no person shall be compelled to testify against himself, and protecting himagainst unreasonable searches and seizures. Issue: WON the defendants positions are with merit. Ruling (s): 1. That although in the issuance of the search warrant in question the judge did not comply with therequirements of section 98 of General Orders No. 58, the petitioners are not entitled to the return of theopium and its paraphernalia which were found and seized under said warrant, and much less are theyentitled to be exonerated because of such omission of the judge.2. That the search made on May 1st was a continuation of the search begun on the previous day, and,therefore, did not require another search warrant.3. That the seizure of the petitioner's books, letters, telegrams, and other articles which have no inherentrelation with opium and the

5.

possession of which is not forbidden by law, was illegal and in violation of thepetitioners' constitutional rights. RD: (Contention # 1)-SEC. 96. of General Orders No. 58 provide: It (a search warrant) may be issued upon either of thefollowing grounds: 1. When the property was stolen or embezzled. ; 2. When it was used or when the intentexists to use it as the means of committing a felony.-Suffice it to say that, whatever may be the technical common-law meaning of the word "felony", which isused in paragraph 2 of sec. 96 above quoted, the Court believes it would be the height of absurdity tohold, upon technical grounds, that a search warrant is illegal which is issued to search for and seizeproperty the very possession of which is forbidden by law and constitutes a crime. Opium is such property.- Search-warrants have heretofore been allowed to search for material so kept as to endanger the publicsafety.- A search warrant may be likened to a warrant of arrest. The issuance of both is restricted by the sameprovision of the Jones Law (sec. 3) which is as follows: That no warrant shall issue but upon probablecause, supported by oath or affirmation, and particularly describing the place to be searched and the person or thing to be seized. - In the present case there was an irregularity in the issuance of the search warrant in question in that the judge did not first examine the complainant or any witnesses under oath. But the property sought to besearched for and seized having been actually found in the place described by the complainant, reasoningby analogy from the case of an improper arrest, we are of the opinion that that irregularity is not sufficientcause for ordering the return of the opium found and seized under said warrant, to the petitioners, andexonerating the latter.(Contention # 2) 8 - While it is true that a warrant is good for 10 days after the date of issuance, this cannot be interpreted tomean that a search warrant can be used every day for 10 days, and for a different purpose each day. Thiswould be absurd.-It appears from the oral evidence adduced during the hearing of the petitioners' motion in the court belowthat the search for opium, the property mentioned in the warrant, was not completed on April 30th; it wasinterrupted by the necessity to ascertain who the owner of the bodega on the ground-floor was, becausethe petitioner Uy Kheytin falsely disclaimed ownership thereof. In other words, the search of May 1st wasnot made "for a different purpose," nor could it be considered "another search," but was really acontinuation of the search begun on April 30th. This is shown by the fact that during the interval betweenthe two searches the premises in question were guarded by Constabulary soldiers, and the petitionerswere made to understand on April 30th that the authorities were not yet through with the search andwould continue the same as soon as they found out that the bodega was also occupied by the petitionerUy Kheytin.(Contention # 3)- In order to comply with the constitutional provisions regulating the issuance of search warrants, theproperty to be seized under a warrant must be particularly described therein and no other property can betaken thereunder.- That the officers of the law believed that the books, papers, etc., which they seized might be used asevidence against the petitioners herein a criminal action against them for a violation of the Opium Law, isno reason or justification under the law for the seizure: First, because they were not "particularlydescribed" or even mentioned in the search warrant; second, because, even if they had been mentioned inthe search warrant, they could not be legally seized, for a search warrant cannot be used for the purposeof obtaining evidence; and third, because to compel a person to produce his private papers to be used inevidence against him would be equivalent to compelling him to be a witness against himself. Burgos v. Chief of Staff, supra 6. Admissibility of Illegally-Seized Evidence CASES: Stonehill v. Diokno, supra Alih v. Castro, supra Pp v. Exala, 221 SCRA 494 THE FACTS A private jeep driven by accused-appellant Bocalan was stopped at a police checkpoint in Cavite City for routine inspection. With Bocalan were his co-accused Fernandez and Exala. Pfc. Galang, a member of the inspection team, went near the jeep and asked the

occupants if there were firearms inside. They answered in the negative. Pfc. Galang proceeded to inspect the vehicle by beaming a flashlight inside. He then noticed a black leather bag measuring about 1 foot wide and 2 feet long with its sides bulging. When he asked what it contained, there was deadening silence from the 3 accused. Nobody answered. Instead, they suddenly became fidgety. Suspicious, Pfc. Galang ordered the bag opened, which was found out to contain marijuana. The 3 accused were thereafter prosecuted and convicted of illegal transportation of marijuana. Accused Bocalan appealed and questioned the legality of the admission of the marijuana as evidence against him since it was seized without a valid search warrant. II. THE ISSUE Was the marijuana seized without warrant during the checkpoint admissible in evidence against the accused? III. THE RULING [The 1 Division voted 3-1 to AFFIRM the conviction of the accused. Justices Grio-Aquino and Quiason concurred with Justice Bellosillos ponencia. Justice Cruz, by his lonesome, dissented from the majority.] The Court held that Bocalan is deemed to have waived his objection to the admission of the seized marijuana because he neither raised this issue before the trial court nor objected to the admissibility of the marijuana when it was offered in evidence. And even assuming that there was no such waiver, the Court held that still Bocalans contention deserves scant consideration because there are instances where search and seizure can be made without necessarily being preceded by an arrest. An illustration would be the stop-and-search without a warrant at military or police checkpoints, the constitutionality of which has already been upheld by this Court [inValmonte vs. De Villa]. Vehicles are generally allowed to pass through these checkpoints after a routine inspection and answering a few questions. If vehicles are stopped and extensively searched it is because of some probable cause which justifies a reasonable belief of those manning the checkpoints that either the motorist is a law-offender or thecontents of the vehicle are or have been instruments in the commission of an offense. According to the Court, lest it be misunderstood, the foregoing doctrine is not intended to do away with the general rule that no person shall be subjected to search of his person, personal effects and belongings, or his residence except of virtue of a search warrant or on the occasion of a lawful arrest. This case, however, is an incident to or an offshoot of a lawful stop-and-search at a military or police checkpoint. The checkpoint in the instant case was established in line with Operational Bakal, the main object of which was to search for unlicensed firearms and other prohibited items in the possession of unauthorized persons passing through it. When the jeep carrying the contraband passed through the checkpoint, it was flagged down and the occupants were asked routine questions. In the course thereof, Pfc. Galang noticed a black leather bag the sides of which were bulging. He asked what the contents of the bag were. None of the accused answered. At that moment, the demeanor of the accused changed; they became suspiciously quiet and nervous as if they were concealing something from Pfc. Galang.The accused clearly appeared to be in abject fear of being discovered. Such peculiar apprehensiveness if not restrained reaction of the accused, which did not appear normal, provided the probable cause justifying a more extensive search that led to the opening of the bag and the discovery of the prohibited stuff.
st

I.

[NOTE: Incidentally, one of the co-counsels for accusedappellant Bocalan in his appeal to the Supreme Court was then-Atty. and now Supreme Court Senior Associate Justice Presbitero Velasco Jr.]

Culled from the findings of the trial judge, as well as the [2] decision of the Court of Appeals, promulgated on 09 November 1998 and penned by Associate Justice Delilah Vidallon-Magtolis, the facts could be gathered, thusly: At about two o'clock in the morning of 29 April 1994, Chief of Police Rosauro Francisco of Tarragona, Davao Oriental, received a tip from a police informer that Rolando Zaspa and a companion were bringing dried marijuana leaves bound for Mati, somewhere at Crossing Banhawan, Tarragona, Davao Oriental. The police chief promptly organized and dispatched to the area a team composed of SPO2 Honorio Carasca (the team leader), PO1 Letecio Rafael and SPO1 Cesar Travelegio. The group immediately proceeded to Crossing Banhawan, arriving thereat at about five o'clock in the morning. There, the team saw Zaspa and his companion standing by the side of the road with a big black "loalde" bag in front of them. Just as SPO2 Carasca and PO1 Rafael, who were both in uniform, proceeded to approach the two men, Zaspa tried to flee. He was intercepted by the policemen. Zaspa claimed that the contents of the bag did not belong to them. When the bag was opened, Zaspa told the policemen that the dried marijuana leaves were owned by one Bito Mangandan. Zaspa and his companion, who turned out to be Julius Galvan, were arrested and brought to the Tarragona police station for investigation. Samples of the leaves taken from the bag were sent to the PNP Crime Laboratory in Ecoland, Davao City, where the specimen were tested and confirmed to be marijuana leaves. Chemistry Report No. 035-94, submitted by Police Senior Inspector Noem; Austero, a forensic expert, contained the following findings: "Qualitative examination conducted on the above-mentioned specimen gave positive result to the tests for marijuana, a [3] prohibited drug." Zaspa, taking the witness stand in his defense, testified that sometime in January 1994 he was hired by Maturino Masanguid to cut an Antipolo round timber for the amount of P5,000.00. He was paid P3,500.00 and was about to get the balance of P1,500.00 on 29 April 1994 when the incident transpired. He stated that he was walking towards Barrio Sambarangay when an armed man in civilian outfit pointed a gun at him and proceeded to examine the brown bag he was holding. He was thereafter dragged to the side of the road and questioned whether a black bag also belonged to him. He denied either ownership or possession of the bag. He was brought to the police station in Tarragona with another man whom he later learned to be Julius Galvan. At the police station, a certain Francisco slapped him and made him and Galvan crawl on the floor. The man also tried to smash the face of Galvan. He was detained in Tarragona Municipal jail from 29 April until he was brought, on 02 May 1994, to the PC Barracks at Menzi, Mati, Davao Oriental. Galvan corroborated the testimony of Zaspa. He asserted that on 12 April 1994, he was contracted by Algin Divinagracia to spray their mango trees in Banhawan. The first spray was made on 15 April; he returned to Mati on the same day. The second spray was made on 28 April but, unable to finish the work that day, he stayed overnight with Divinagracia. At about four o'clock in the morning, he was on his way to Banhawan crossing when he was stopped by a man holding a gun. He was brought to a place where some police officers were questioning a man about the ownership of a black bag. The two were brought to the police station where they were investigated and detained until they were brought the following day to the P.C. Barracks at Mati. After trial, the court a quo found the two accused guilty of the crime with which they were charged; the trial court held: "WHEREFORE, the Court finds the accused Rolando Zaspa alias `Tata' and Julius Galvan both GUILTY beyond reasonable doubt of violation of Section 8 in relation to Section 20 of the Republic Act 6425, as amended by R.A. 7659, and hereby imposes upon each of them the penalty of RECLUSION PERPETUA and to pay a fine of P500,000.00, with the accessory penalties provided by law, and to pay the costs of the proceedings. "The marijuana leaves subject hereof (Exhs. `D,' `D-1' and `D-2') are hereby ordered turned over to the Dangerous Drugs Board, thru the

CRUZ, J., dissenting: Justice Cruz maintained the proposition in his dissent in Valmonte vs. De Villathat checkpoints and the searches and seizures incident thereto are unconstitutional. InPeople vs. Exala, he expounded on this thesis: I am opposed to checkpoints as regular police measures aimed at reducing criminality in general. I do not agree that in the interest of peace and order, any or every vehicle may be stopped at any time by the authorities and searched without warrant on the chance that it may be carrying prohibited articles. That possibility is not the probable cause envisioned in the Bill of Rights. In the case of the ordinary checkpoint, there is not even any suspicion to justify the search. The search is made as a matter of course, either of all vehicles or at random. There is no showing that a crime is about to be committed, is actually being committed, or has just been committed and the searching officer has personal knowledge that the person being searched or arrested is the culprit. I will concede that checkpoints may be established at borders of states or at constructive borders near the boundary for the purpose of preventing violations of immigration and customs laws. But in the interior of the territory, the requirements of a valid search and seizure must be strictly observed. The only permissible exemption is where a crime like a bank robbery has just been committed or a jailbreak has just occurred, and the authorities have to seal off all possible avenues of escape in the area. In all other cases, I submit that the checkpoint should not be allowed. xxx. [W]e cannot retroactively validate an illegal search on the justification that, after all, the articles seized are illegal. That is putting the cart before the horse. I would rather see some criminals go unpunished now and then than agree to the Bill of Rights being systematically ignored in the oppressive checkpoint. Respect for the Constitution is more important than securing a conviction based on a violation of the rights of the accused. (Emphasis supplied.) Pp v. Zaspa, 340 SCRA 752 DECISION VITUG, J.: Elevated and certified by the Court of Appeals to this Court for review is the decision, dated 08 January 1996, of the Regional Trial Court of Mati, Davao Oriental, Branch 5, penned by Judge Ricardo M. Berba. The case, docketed Criminal Case No. 2621 before the trial court, has charged Rolando Zaspa, a.k.a. "Tata," and Julius Galvan with violation of Section 8 of Republic Act No. 6425, otherwise also known as the Dangerous Drugs Act of 1972, as amended. The case originated from an Information, dated 13 October 1994, which read: "That on or about April 29, 1994, in the Municipality of Tarragona, Province of Davao Oriental, Philippines and within the jurisdiction of this Honorable Court, the abovenamed accused, with intent to use did then and there wilfully, unlawfully, feloniously own and possess five point six (5.6) kilos of marijuana dried leaves with stalks, a prohibited dangerous drugs, without proper license or permit from [1] the authorities." Upon arraignment, both accused pled "not guilty" to the charge.

National Bureau of Investigation (NBI), for disposition in accordance with law. "SO ORDERED."
[4]

Zaspa and Galvan appealed their conviction, albeit the penalty imposed, to the Court of Appeals for review. The appellate court upheld the conviction; it said: "Regarding the first issue, the appellee asseverates that there is entirely no merit in appellants' claim that they were not in possession of marijuana when the police authorities apprehended them at Banhawan Crossing. The testimonial evidence submitted by the prosecution clearly pointed out that the two appellants were standing by the side of the road, with the black bag containing the prohibited marijuana just 1/2 foot away. The absence of any other person within the vicinity indicates that the contraband belonged to the appellants and to no one else. Now, the appellants could not sufficiently explain the presence of the bag in their possession. Neither could they explain why the police would `plant' the same as evidence. Likewise, they did not substantiate the alleged `maltreatment' suffered in the hands of the law enforcers. Moreover, no improper motive was attributed to the police as to why they would testify falsely against the appellants -- if such was the fact. "With respect to the alleged unlawful and warrantless arrest, the People manifests that Section 5, Rule 113 of the Revised Rules of Court authorizes an arrest without a warrant when the person to be arrested has committed a crime, is actually committing or about to commit a crime in the presence of the police officers. As the appellants were found to be in possession of the prohibited drug at the time of their arrest, the same is admissible as evidence. "We agree with the appellee. Well-settled is the rule that peace officers may pursue and arrest without a warrant any person under circumstances reasonably tending to show that such person has committed or is about to commit any crime or breach of the peace (People vs. Bautista, 227 SCRA 152). In the case at bench, the facts and circumstances leading to the arrest of the accused at dawn of April 29, 1994 would show that the arresting officers have proper and justifiable reasons to arrest the two (2) suspects. First, they received a confidential information from a police informer that a certain Rolando Zaspa with a companion were bringing dried marijuana leaves bound for Mati. Second, when the police arrived at the crime scene, the two (2) suspects were suspiciously at the side of the road with a big black bag in front of them. Third, there were no other people in sight and it is therefore safe to conclude that the bag containing the contraband belonged to no one else but the suspects. Lastly, when the police officers were approaching, the appellant Zaspa attempted to escape. "A warrantless arrest and seizure was valid where it was done by the police team dispatched to look for persons responsible for the crime (People vs. Acol, 232 SCRA 406). In any case, in accordance with settled jurisprudence, any objection, defect or irregularity attending an arrest must be made before the accused enters his plea (Padilla vs. Court of Appeals, 269 SCRA 402). Thus, any irregularity attendant to the arrest of the accused was cured when they voluntarily submitted to the jurisdiction of the trial court by entering a plea of not guilty and by participating in the trial (People vs. De Guzman, 224 SCRA 93). "As to the seized marijuana, the same is admissible in evidence, for trite is the jurisprudence that the search of the appellant's person and the seizure of the marijuana in his possession were valid because they were incident to a lawful warrantless arrest (People vs. Gerente, 219 SCRA 756). As the appellant Zaspa opened the black bag containing the prohibited drug, then it is a consented search. Settled is the rule that drugs discovered as a result of a consented search is admissible in evidence (People vs. Cuizon, 256 SCRA 325). "In the case at bench, it is clear that the appellants were really in possession of the seized marijuana. Prosecution witness SPO2 Honorio Carasca's testimony that the black bag containing the

contraband was just 1/2 foot from infront of the appellants (TSN, 4/25/95, p. 10), apart from the fact that there were no other people in the vicinity, would clearly show that the two (2) appellants were the possessors of the prohibited item. The appellants' respective defenses denying the ownership and possession of the marijuana cannot topple the evidence proffered by the prosecution. In the first place, denial, like an alibi, is a weak defense which becomes even weaker in the face of positive identification of the accused by prosecution witnesses (People vs. Ompad, 233 SCRA 62). Second, the appellants failed to present any receipts proving that on different occasions, they were at the crime scene merely by chance. It must be noted that the appellant Zaspa failed to present any evidence to show that he collected the sum of P3,500.00, or that he was to collect the balance of P1,500.00 from the witness, Mr. Maturino Masanguid. In the same manner, the appellant Galvan failed to present the receipts for the medicine he purportedly bought for the spraying of the mango trees (TSN, 7/19/95, p. 22). Third, if the appellants were really maltreated in order to confess to the crime charged, then why is it that they failed to submit to a medical examination or treatment? (TSN, 6/8/95, p. 48 and TSN, 7/19/95, p. 36). Why is it that the appellants filed a case before the Ombudsman and the Commission on Human Rights only after almost a year from the time of the incident (Exhibit `1' and `2')? In any case, there is nothing on record which indicates that the police operatives were actuated by improper motive against the appellants. Credence can be given to the narration of the incident by the prosecution witnesses, who as police officers are presumed to have performed their duties in a regular manner in the absence of proof to the contrary (People vs. Bautista, 227 SCRA 152). Lastly, as the seized items were positive for marijuana, then, the corpus delicti of the crime had been fully proved with certainty and conclusiveness (People vs. Simon, 234 SCRA 555). "In sum, the guilt of the accused-appellants has been established by proof beyond reasonable doubt; hence, the affirmance of the appealed judgment is in order. However, considering that the applicable penalty isreclusion perpetua, the second paragraph of Section 13, Rule 124 of the Rules of Court shall apply. "WHEREFORE, the judgment appealed from is AFFIRMED in toto. Let this case be CERTIFIED and ELEVATED to the Honorable Supreme Court for review. "SO ORDERED."
[5]

Although the appeal from the decision of the trial court imposing reclusion perpetua should have been directly appealed to this Court, considering the penalty involved, the Court has decided to ignore this breach of technicality and to nevertheless consider the appeal and evaluate the case. The testimony given by the witnesses for the prosecution and that of the defense are diametrically opposed to each other on almost every point. In resolving such a conflict, so dealing as it does on the credibility of the witnesses, the Court relies heavily on findings of the trial court being in the best position, certainly more than the appellate court, in making that judgment. Thus, often repeated, is the standing rule that absent any showing that the trial judge has overlooked, misunderstood or misappreciated any evidence that could otherwise alter the result of the case, the Court would adhere to the assessment made by the trial court on the question. Parenthetically, the Court of Appeals has basically done likewise in finding no reason to alter the conclusion of the trial judge. In almost every case involving a buy-bust operation, the accused would put up the defense of frame-up. The Court views such a claim with disfavor for, like alibi, the frame-up theory can [6] easily be concocted. In the instant case, the police informer has particularly mentioned the name of Rolando Zaspa as being one of those who would be bringing the bag containing the [7] marijuana, thus paving the way for the authorities to conduct their operation. When Zaspa, indeed, has made an attempt to run away upon seeing the police officers, he inadvertently has also confirmed the information given to the police. It bears to repeat that absent any convincing proof of an intent on the part of police authorities to falsely impute a serious crime against an accused, the presumption

of regularity in the performance of official duty will ordinarily have [8] to prevail. On the validity of the warrantless arrest, along with the corresponding search and seizure, suffice it to say that any objection regarding the regularity of an arrest must be made before the [9] accused enters his plea; otherwise, the defect shall be deemed cured by the voluntary submission by the accused to the jurisdiction [10] of the trial court. No significant value could be given to the allegations of accused-appellants that they were maltreated. Zaspa only decided to file charges against the apprehending police officers after almost a year following the incident. For his part, Galvan chose to remain silent. Neither one of the two accused-appellants submitted himself to medical examination. The attendant circumstances scarcely augur well to support the asseveration of maltreatment allegedly suffered by accused-appellants from the police authorities. WHEREFORE, the decision of the trial court is AFFIRMED in toto. Costs against accused-appellants. SO ORDERED. Melo, (Chairman), Panganiban, Purisima, and Gonzaga-Reyes, JJ., concur. 7. Warrantless Searches and Seizures See Sec. 5, Rule 113, Revised Rules of Criminal Procedure Other cases of valid warrantless searches CASE: Malacat v. CA, 283 SCRA 159

overt physical act, on the part of petitioner,indicating that a crime had just been committed, was being committed or wasgoing to be committed. Having thus shown the invalidity of the warrantless arrest in this case, plainly, thesearch conducted on petitioner could not have been one incidental to a lawfularrest.We now proceed to the justification for and allowable scope of a "stop-and-frisk" as a "limited protective search of outer clothing for weapons," as laid downin T erry , thus:We merely hold today that where a police officer observes unusualconduct which leads him reasonably to conclude in light of hisexperience that criminal activity may be afoot and that the personswith whom he is dealing may be armed and presently dangerous,where in the course of investigating this behavior he identifieshimself as a policeman and makes reasonable inquiries, and wherenothing in the initial stages of the encounter serves to dispel hisreasonable fear for his own or others' safety, he is entitled for theprotection of himself and others in the area to conduct a carefullylimited search of the outer clothing of such persons in an attempt todiscover weapons which might be used to assault him. Such asearch is a reasonable search under the Fourth Amendment . . .Other notable points of Terry are that while probable cause is not requiredto conduct a "stop and frisk," it nevertheless holds that mere suspicion or ahunch will not validate a "stop and frisk." A genuine reason must exist, inlight of the police officer's experience and surrounding conditions, towarrant the belief that the person detained has weapons concealedabout him. Finally, a "stop-andfrisk" serves a two-fold interest: (1) thegeneral interest of effective crime prevention and detection, whichunderlies the recognition that a police officer may, under appropriatecircumstances and in an appropriate manner, approach a person for purposes of investigating possible criminal behavior even withoutprobable cause; and (2) the more pressing interest of safety and selfpreservation which permit the police officer to take steps to assure himselfthat the person with whom he deals is not armed with a deadly weaponthat could unexpectedly and fatally be used against the police officer.Here, there are at least three (3) reasons why the "stop-and-frisk" was invalid:First, we harbor grave doubts as to Yu's claim that petitioner was a member ofthe group which attempted to bomb Plaza Miranda two days earlier. This claimis neither supported by any police report or record nor corroborated by anyother police officer who allegedly chased that group. Aside from impairing Yu'scredibility as a witness, this likewise diminishes the probability that a genuinereason existed so as to arrest and search petitioner. If only to further tarnish the credibility of Yu's testimony, contrary to his claim that petitioner and hiscompanions had to be chased before being apprehended, the affidavit ofarrest (Exh. "A") expressly declares otherwise, i . e ., upon arrival of five (5) other police officers, petitioner and his companions were "immediately collared."Second, there was nothing in petitioner's behavior or conduct which could havereasonably elicited even mere suspicion other than that his eyes were "movingvery fast" an observation which leaves us incredulous since Yu and histeammates were nowhere near petitioner and it was already 6:30 p.m., thuspresumably dusk. Petitioner and his companions were merely standing at thecorner and were not creating any commotion or trouble, as Yu explicitlydeclared on cross-examination:Q And what were they doing?A They were merely standing.Q You are sure of that?A Yes, sir.Q And when you saw them standing, there werenothing or they did not create any commotion.A None, sir.Q Neither did you see them create commotion?A None, sir.Third, there was at all no ground, probable or otherwise, to believe thatpetitioner was armed with a deadly weapon. None was visible to Yu, for as headmitted, the alleged grenade was "discovered" " inside the front waistline " ofpetitioner, and from all indications as to the distance between Yu andpetitioner, any telltale bulge, assuming that petitioner was indeed hiding agrenade, could not have been visible to Yu. In fact, as noted by the trial court:When the policemen approached the

(b) When an offense has in fact just been committed,and he has personal knowledge of facts indicating thatthe person to be arrested has committed it; and(c) When the person to be arrested is a prisoner whohas escaped . . .A warrantless arrest under the circumstances contemplated under Section 5(a) has been denominated as one " in flagrante delicto ," whilethat under Section 5(b) has been described as a "hot pursuit" arrest.Turning to valid warrantless searches, they are limited to the following: (1)customs searches; (2) search of moving vehicles; (3) seizure of evidence in plainview; (4) consent searches; 33 (5) a search incidental to a lawful arrest; 34 and (6)a "stop and frisk." 35 In the instant petition, the trial court validated the warrantless search as a "stopand frisk" with "the seizure of the grenade from the accused [as an appropriateincident to his arrest," hence necessitating a brief discussion on the nature ofthese exceptions to the warrant requirement.At the outset, we note that the trial court confused the concepts of a "stop-and-frisk" and of a search incidental to a lawful arrest. These two types of warrantlesssearches differ in terms of the requisite quantum of proof before they may bevalidly effected and in their allowable scope.In a search incidental to a lawful arrest, as the precedent arrest determines thevalidity of the incidental search, the legality of the arrest is questioned in a largemajority of these cases, e . g ., whether an arrest was merely used as a pretext for conducting a search. In this instance, the law requires that there first be a lawfularrest before a search can be made the process cannot be reversed. Atbottom, assuming a valid arrest, the arresting officer may search the person ofthe arrestee and the area within which the latter may reach for a weapon or for evidence to destroy, and seize any money or property found which was used inthe commission of the crime, or the fruit of the crime, or that which may be usedas evidence, or which might furnish the arrestee with the means of escaping or committing violence.Here, there could have been no valid in flagrante delicto or hot pursuit arrestpreceding the search in light of the lack of personal knowledge on the part ofYu, the arresting officer, or an

accused and hiscompanions, they were not yet aware that a hand grenade wastucked inside his waistline. They did not see any bulging object in[ sic ] his person.What is unequivocal then in this case are blatant violations of petitioner's rightssolemnly guaranteed in Sections 2 and 12(1) of Article III of the Constitution 679 Paras (Edgardo), J.: This is an unlawful detainer case originating from the Municipal Court of the City of Manila which rendered a decision in favor of the plaintiff. Upon appeal by the defendant, and after a trial de novo, the Court of First Instance of Manila absolved him from the complaint, at the same time decreeing that the rent which should be paid by the defendant to the plaintiff beginning August, 1945, is P50. The case now before us on appeal by the plaintiff. The latter erroneously contends, under his first assignment of error, that the Court of First Instance should have ordered the dismissal of defendants appeal from the decision of the Municipal Court in view of his failure to file an appeal bond for sum of P25. As the purpose of the appeal bond is to insure the payment of all costs which the Court of First Instance may award against the appellant, and as the defendant herein had admittedly filed a supersedeas bond in the sum of P80 which is also answerable for costs down to the time of the final judgment, the filing of the appeal bond was unnecessary. (Fernando vs. De la Cruz, 61 Phil. 435.) The defendant does not controvert the plaintiffs contention that the former is holding the premises located at No. 1608 Oroquieta, Manila, under a month-to-month contract of lease; and the only question raised by the plaintiffs second assignment of error is one of fact, namely: Whether the defendant had defaulted in the payment of the stipulated rents for March, April, May, and June, 1945, in a sense warranting his ejectment. The lessor may, under article 1569 of the Civil Code, judicially dispossess the lease for default in the payment of the price agreed upon. But, after a careful perusal of the entire record, we have come to the belief that the defendant had in fact refused to pay said rents because the plaintiff sought to collect an amount (P50) beyond that agreed upon. Indeed, the rental charged by the plaintiff and actually paid by defendant for December, 1944, and January, 1945, was P40, the attempt of plaintiff to collect P50 without defendants previous conformity having commenced in February, 1945, with the further warning by the plaintiff to the defendant that said rent would be increased to P60 beginning March, 1945. The defendant had purposely stopped paying the increased rent for the reason that, so the answer alleges, plaintiff had formed the habit of raising and raising it until lately he demanded exorbitant rates. At this point, it need only be pointed out that the rents in question steadily rose from the initial amount of P20 to P25, P30, P40 and P50, with the further threat by the plaintiff to make the same P60. It is not necessary to decide whether said rates exceed the authorized limits, since our finding that the alleged unpaid rents had not been previously stipulated by the parties is sufficient to dispose of the search incident to a lawful arrest CASES: Pp v. Figueroa, 248 SCRA

present appeal adversely to plaintiffs pretense that the Court of First Instance erred in not sentencing the defendant to vacate the premises involved herein to pay P50 a month beginning March, 1945. Our view is not calculated to curtail or violate plaintiffs proprietary rights or his freedom of contract, as safeguarded and delimited by the Constitution and the law. But we cannot be so insensitive to the present housing situation as to fail to make an appeal, however faint it may prove to be, for owners to be more human and less opportunistic by retaining, at least during the emergency, their old tenants, provided of course they are willing and able to pay a reasonable rental which will in all likelihood be lower than what would be offered by contenders. As a matter of fact, the idea involved in our appeal has been crystallized in Commonwealth Act No. 689, enacted on October 15, 1945, penalizing speculation on rents of buildings destined for dwelling purposes and providing that the fact that the rents are unjust and unreasonable shall constitute a valid defense in a suit for ejection or for the collection of rents. Whether the plaintiff may dispossess the defendant through an action based on the expiration or termination of the lease (article 1569, Civil Code) is a matter not herein ventilated. This is true, notwithstanding plaintiffs allegation that he needs the premises for his law office and to accommodate his brother who is a fire victim, since this allegation, though expressive of his projected use, is not the basis of the cause of action. Even so, there is evidence to the effect that the plaintiff owns several houses in Manila and his brother has been living with him at his residence on Quiricada Street, Manila, wherein the plaintiff also has his law office. The plaintiff likewise charges the defendant with having cooked inside the house with firewood, a complaint which is obviously flimsy, if not puerile, because it is of common knowledge that firewood is in general use as a fuel, in the total absence of gas and in view of the very limited supply of electric stoves. Besides, there is absolutely no indication in the proof that, under the lease, the defendant is prohibited either from cooking inside the premises or from employing firewood for the purpose. The appealed judgment is affirmed, with costs against the appellant. So ordered. Moran, C.J., Jaranilla, Pablo, and Briones, JJ., concur. Feria, J. concurs in the result.

Pp v. Salazar, 266 SCRA 607 PANGANIBAN, J.: Although homicide (a crime against persons) is independently a graver offense than robbery (a crime against property), it is treated in the special complex crime of robbery with homicide as a mere incident committed by reason or on the occasion of the robbery. Unless the prosecution convincingly proves that the main purpose of the culprit(s) was the asportation of personal property and that the death was merely incidental to such asportation, there can be no conviction for this special complex crime.

Statement of the Case

22). Armed with the dagger, Gotangugan suddenly started stabbing Gatmen (Ibid., p. 5, 15, 16). At that precise moment, Pedro Soriano, who was only about 10 to 15 meters from Lindas Supermarket heard moans coming from the guardhouse in front of Lindas Supermarket. He turned his head towards the place where the moans were coming from and saw Gatmen inside the guardhouse being repeatedly stabbed by Gotangugan (TSN, November 6, 1989, pp. 2-6, 15-16). Both Miranda and Soriano were able to witness and identify the malefactors because the place where the incident happened was well-lighted (TSN, October 31, 1989, p. 22; November 6, 1989, pp. 45). While Gotangugan was stabbing Gatmen, Salazar stood close to Gotangugan, while their unidentified companion acted as a lookout (TSN, October 31, 1989, pp. 8, 15; November 6, 1989, pp. 16). Out of fear, Miranda and Arriola ran towards Tandang Sora. While running, however, they saw Salazar and Gotangugan get the revolver of Gatmen (TSN, October 31, 1989, p. 7-8). Soriano, on the other hand, left slowly but saw Salazar get the gun of Gatmen (TSN, November 6, 1989, p. 6-7). After getting the gun, Salazar, Gotangugan, and their unidentified companion left the scene of the crime (TSN, October 31, 1989, p. 8; November 6, 1989, p. 7). Gatmen died as a consequence of the following stab wounds, to wit: (1) Hacked wound, frontal extending to the right preauricular region, measuring 10 by 1 cm, 8 cm from the anterior midline, fracturing the frontal bone. (2) Stab wound, right zygomatic region, measuring 3 by 0.7 cm, 10 cm from the anterior midline, directed posteriorwards and medialwards, fracturing the right zygomatic bone. (3) Incised wound, chin, measuring 1.2 by 0.3 cm, 2 cm right of the anterior midline. (4) Stab wound, neck measuring 5 by 1.2 cm, crossing the anterior midline, 4 cm to the right and 1 cm to the posteriorwards and medialwards, lacerating the trachea, larynx and esophagus. (5) Stab wound, right clavicular region, measuring 8 by 2 cm, 7 cm from the anterior midline, 3 cm deep, directed posteriorwards and medialwards, fracturing the right clavicle. (6) Stab wound, interclavicular region, measuring 3 by 0.1 cm, 6 cm from the anterior midline.

This principle is stressed by the Court as it rules on this appeal [1] from the Judgment dated April 1, 1991 of the Regional Trial Court [2] of Quezon City, Branch 104 which, acting as a special criminal court, convicted Appellants Domingo Salazar y Seroma alias Inggo and Monchito Gotangugan y Sevilla alias Monching of robbery with homicide. In an Information dated July 31, 1989, Asst. Quezon City Prosecutor Perpetuo L.B. Alonzo accused Appellants Salazar and Gotangugan, together with one John Doe, of robbery with [3] homicide committed as follows: That on or about the 10th day of March 1989, in Quezon City, Metro-Manila, Philippines and within the jurisdiction of this Honorable Court, the above-named accused, conspiring together, confederating with and mutually helping one another, did then and there, wilfully, unlawfully and feloniously, with intent to gain and by means of violence upon person, rob one CRISPIN GATMEN Y CEYNAS of his service firearm, a Squires Bingham Cal. 38 Revolver with Serial No. 1096012 valued at P6,000.00, Philippine Currency, to the damage and prejudice of the said offended party thereof in the aforementioned amount; and that by reason of or on the occasion of the said robbery, said accused with intent to kill and without any justifiable cause, did then and there, wilfully, unlawfully and feloniously attack, and assault the person of said CRISPIN GATMEN Y CEYNAS, by stabbing the latter, hitting him on the different parts of his body by the use of bladed weapon, thereby inflicting upon him serious and mortal wounds which were the direct and immediate cause of his untimely death, to the damage and prejudice of the heirs of said victim in such amount as may be awarded to them under the provisions of the Civil Code. Upon arraignment, appellants pleaded not guilty. After trial proceeded in due course, the court a quo rendered the assailed [4] Judgment, the dispositive portion of which reads: WHEREFORE, judgment is hereby rendered, finding both accused, Domingo Salazar y Seroma and Monchito Gotangugan y Sevilla, guilty of the crime of Robbery with Homicide as charged in the information. They are both sentenced to suffer the penalty of RECLUSION PERPETUA, to pay the heirs of the deceased damages in the amount of P30,000.00, plus the sum of P6,500.00 representing the value of the revolver taken by both accused, plus all the accessory penalties provided for by law, without subsidiary imprisonment in case of insolvency, and to pay the costs.

The Facts Evidence for the Prosecution

The prosecution presented the following witnesses: Pfc. Jose Antonio of the Quezon City Police, Eyewitnesses Vicente Miranda, Jr. and Pedro Soriano, Dr. Dario L. Gajardo of the PC/INP Crime Laboratory and Ben Felipe Dangza, Consultant/Manager of PUMA Security Agency. The Solicitor General, on behalf of the People, [5] summarized the facts as viewed by the prosecution: On March 10, 1989, at or about 3:30 (a.m.), Vicente Miranda, Jr. and his friend Nestor Arriola were standing in the corner of Road 1 and Visayas Avenue, Quezon City, about 12 meters from Lindas Supermarket (TSN, October 31, 1989, pp. 3 and 21). At about the same time, Pedro Soriano, who was himself waiting for a ride, was standing in front of the Kambingan Restaurant along Visayas Avenue and beside Lindas Supermarket (TSN, November 6, 1989, p. 23). Moments later, they saw appellants Domingo Salazar and Monchito Gotangugan together with an unidentified companion approach the security guard of Lindas Supermarket (TSN, October 31, 1989, pp. 3-4, 27). Salazar, Gotangugan and their companion talked to the security guard, who was later identified as Crispin Gatmen. Thereafter, Miranda saw Salazar pull out a 9-10 inches long dagger from his pocket, and pass the same to Gotangugan (Ibid., pp. 5, 15,

(7) Linear abrasion, left mammary region, measuring 3 by 0.1 cm, 6 cm from the anterior midline. (8) Lacerated wound, palmar aspect of the left hand, measuring 5 by 2 cm, 3 cm lateral to its anterior midline. (9) Lacerated wound, palmar aspect of the right hand, measuring 4 by 2 cm, along its anterior midline. (10) Lacerated wound, middle phalange of the left small finger, measuring 1 by 0.1 cm. (11) Stab wound, proximal phalange of the left index finger, measuring 2.2 by 1 cm. (12) Incised wound, middle phalange of the right middle finger, measuring 1 by 0.5 cm. Exhibit E)

Evidence for the Defense

Appellants, testifying for themselves, set up the defense of [6] alibi. The court a quo summarized their testimonies as follows: DOMINGO SALAZAR, 30 years old, and residing at c/o Bureau of Animal Industry, Visayas Avenue, Quezon City, testified that he had been in that place for ten (10) years and that during the early morning of July 27, 1989, he was at their house sleeping, when all of a sudden he was awakened because two persons kicked him and with guns pointed at him, he was ordered to stand up. The two persons searched his things inside the house and asked him where he hid his gun. Then, they dragged him out of his house. Outside the house, he was kicked, boxed and hit with the butt of the gun. There were about five (5) people who went to his house, all armed. He denied any participation in the killing of the deceased because according to him, he was at his house sleeping. He slept at 7:00 (p.m. on) March 9 and woke up at dawn, March 10. He was with his wife Juanita Salazar, and his father-in-law. They were sleeping side by side. He did not see Vicente Miranda and Pedro Soriano before he was arrested on July 27, 1989. However, he admitted having seen Miranda and Soriano at the police headquarters at Sikatuna, peeping at their cell. He often saw them at City Hall with policemen accompanying newly arrested persons. MONCHITO GOTANGUGAN, 23 years old, married, vendor, and with residence at Visayas Avenue, Quezon City testified that he was not at the scene of the crime when the said incident took place and that on March 10, 1989, he was at Lucena City. On July 27, 1989, he was at the house of his sister located at Baex Compound, Visayas Avenue, Quezon City, arriving in that place during the first week of May 1989. In the morning of July 27, 1989, policemen forcibly entered his house and dragged him outside, ransacking his belongings and bringing him at Sikatuna Police Headquarters. He was accused of being a Sparrow, hold-upper and akyat bahay, and ordered to bring out firearms, but they did not find anything. They mauled him at Sikatuna headquarters. While still at their house, he was kicked and hit with the butt of the armalite. At the Sikatuna headquarters, he was never informed by the police that he participated in the killing of the security guard in front of Lindas Supermarket in the morning. He had never seen the witnesses presented by the prosecution, however, he saw them at the City hall together with policemen and other detainees.

show that they are highly incredible and consistently contradicting and improbable. II The lower court erred in convicting both the accused-appellants since the evidence presented by the prosecution failed to prove beyond reasonable doubt the offense charged. III The lower court erred in refusing and failing to find that the herein accused-appellants were arrested without warrant and therefore all evidence obtained after such illegal arrest are inadmissible. IV The lower court committed serious error amounting to grave abuse of discretion in finding that the testimonies of the prosecution witnesses Vicente Miranda and Pedro Soriano were not refuted because the defense interposed by the accused is alibi. These alleged errors will be discussed by the Court under the general heading Credibility of Witnesses and Sufficiency of Evidence. In addition, the Court will tackle, motu proprio, the issue of whether appellants may be held liable for the special complex crime of robbery with homicide, in the light of the proven facts.

The Courts Ruling

The Court finds appellants guilty of two separate felonies; namely, homicide and theft, but not of the special complex crime of robbery with homicide.

Credibility of Witnesses and Sufficiency of Evidence

Ruling of the Trial Court

The trial court brushed aside the alibi interposed by appellants, branding it as an inherently weak defense. It gave full credence to the accounts of the eyewitnesses for the prosecution as no evidence was adduced to refute them or to show why said eyewitnesses would testify falsely against appellants. Quite the contrary, the prosecution evidence (was) clear and convincing. The bare allegation of the defense that they were assets and informants of the apprehending policemen was disbelieved. Thus, it rendered the aforementioned six-page Judgment of conviction. Hence, this appeal direct to this Court, the penalty being reclusin perpetua.

As in most criminal cases, appellants contend that the court a quo erred in bestowing credence on the testimony of prosecution witnesses. Appellants assail the credibility of the eyewitnesses by pointing out several inconsistencies in their testimonies which render them highly improbable and consistently impossible.

Two Different Persons Produced the Dagger and Stabbed the Victim per Mirandas Testimony

Assignment of Errors
[7]

In their Brief, appellants attack the prosecution evidence for its lack of probative value to outweigh their alibi and to sustain their conviction. They allege that the eyewitnesses testimonies suffer from material inconsistencies and contradictions that cast serious doubt on their credibility. Specifically, the following errors were [8] imputed to the court a quo: I The lower court erred in giving full faith and credence to the testimonies of prosecution witnesses Vicente Miranda and Pedro Soriano since a more conscientious scrutiny of their testimonies will

We disagree with appellants contention that, during the direct examination, Prosecution Witness Miranda wobbled particularly on who between the appellants stabbed the victim. The defense segregated Mirandas testimony, quoting and highlighting separate portions thereof to show alleged inconsistencies. According to the defense, Miranda was thus not credible as he was not clear as to who between the two appellants actually pulled out the dagger and stabbed the victim. But when questioned specifically on this point, the witness clarified the sequence of events from the act of pulling out the dagger to the [9] actual stabbing. Said the witness: Q. You said one of the three men pulled out a bladed instrument. Will you tell the Honorable Court how far were you from these three persons who were then in these conversation with the security guard? A. From the witness stand up to that corner, sir. FISCAL BELTRAN: About a distance of 12 meters. I am proposing that the distance pointed to by the witness is about 12 meters.

xxx xxx FISCAL BELTRAN:

xxx

Q. You said that one of the three persons pulled out a bladed instrument. What did you do after pulling out this bladed instrument? A. I saw him, sir, stab the guard. Q. The same person who pulled out this bladed instrument was the same one who stabbed the security guard? A. No, sir. He handed the bladed instrument to the other person, sir. xxx xxx Q. Were you able to see the face of this person whom you said pulled out this bladed instrument? A. Yes, sir. Nor are we persuaded by appellants contention that the witness sworn statement to the police shows that the one who pulled out the dagger was the same person who stabbed the [10] victim. The pertinent portion of his statement reads: 06.TAnong kinalaman mo sa pangyayaring yon, kung mayroon? SMangyari po, ay napadaan ho ako noon, kasama ko yong mga kaibigan ko, sa may harap ng Lindas Grocery sa may kanto ng Road 1, at Visayas Avenue. Noong mga oras na yon (3:30AM) ay nakita ko na yong guardya na nakabantay doon sa may grocery ay parang may sinita na tatlong lalaki, tapos mamaya-maya, yong isang lalaki ay tinapik niya sa puwit yong kasama at bigla na lamang bumunot ng dagger at inundayan ng saksak yong guwardya, mga tatlong sunud-sunod, tapos atras ng atras naman yong gwardya hanggang napasandal doon sa may guard house at doon siya natumba tapos hinablot niya yong baril ng gwardya tapos nag-takbohan na sila patungo sa squatteros (sic) area sa may likoran ng Agriculture building, yong BAEX ho. 07.TBale ilan ka-tao ang sumaksak sa guardia, noong makita mo? SYong isa lang na maliit, na medyo kalbo, at yong isa naman medyo pa-pilay-pilay ay siya tumapik sa kalbo na parang nag-uutos na saksakin yong guardya, yong isa naman ay doon sa may pinto ng Lindas Grocery na tinitingnan naman niya yong kandadado (sic) ng pinto. While there may have been some vagueness in the answer to Question 06, the clarification in the succeeding statement, i.e., the reply to Question 07, sufficiently explains the witness story. It must be remembered that ex parte affidavits are generally considered incomplete and inaccurate and will not prevail over the [11] witness statements on the stand. That the defense labels Mirandas answers as inconsistencies appears to this Court to be merely a strained interpretation of the witness testimony. xxx

not impair a witness credibility. The ambivalence of a witness on the exact number of stabs inflicted on the victim does not detract from the obvious fact that the victim was killed by Gotangugan, as clearly and positively testified to by Miranda. Indeed, in a startling event like a killing, it is difficult for a witness to keep tab of the exact number of strokes the killer made. It is enough that the witness gives a fair estimate. The important thing is that the stabbing took place, the victim died and the witness identified the culprit(s).

[12]

Distance Did Not Necessarily Hinder Perception

The contention that Miranda did not really see the culprits faces deserves scant consideration. The distance of the witness from the stabbing incident was only 12 meters. That the crime happened before dawn (about 3:30 a.m.) is immaterial because the [13] place was lighted. The witness remark that he was far from the situs of the crime merely meant that he was not near enough to hear the culprits whispered conversation, but he was near enough [14] to see their faces and their felonious deed. Appellants denigrate the witness story that he ran closer to the locus criminis while witnessing the stabbing. They claim that the natural tendency would be to run farther away. However, under the circumstances, the witness did not really intend to come closer to the scene of the crime. What he did was to go to the area of Tandang Sora where there was a public market and where, expectably, there would be people even at such an early hour. The area towards the city hall, on the other hand, was unlit and deserted, as the government buildings there were still closed. Besides, since the witness was on his way to his house in Teachers Village, he had to go to Tandang Sora, at that time, to get a ride. Hence, the witness reaction was not unnatural. In fact, it was most prudent under the circumstances.

No Two Versions of the Event in Sorianos Testimony

The defense contends that the other eyewitness, Soriano, did not actually see the stabbing incident, much less the person who stabbed the victim. Appellants claim that Soriano presented two versions of the incident. According to one version given during direct examination, the witness allegedly stated that the security guard was seated outside the guardhouse and that one of the accused urinated behind him. Thereafter, the latter stabbed the [15] victim. In the other version given during cross-examination, the victim was allegedly inside the guardhouse when one of the culprits [16] stabbed the victim. We disagree. The guardhouse was actually an outpost, a structure open on all sides. The guard was sitting under its roof and his assailants were standing outside the roof. One of them was behind him and relieving himself. In this light, it is easy to understand the absence of discrepancy in the witness testimony on this point.
[17]

Appellant Gotangugan Stabbed the Victim

The Number of Stabs Inflicted Is a Minor Matter

Appellants make a mountain out of Mirandas admission on cross-examination that he was not sure how many times Gotangugan stabbed the victim vis-a-vis his earlier statement that one of appellants stabbed the victim thrice. There is really no inconsistency here. And even if we grant that there is, the alleged conflict pertains to an insignificant detail that is not material to the question of who killed the deceased. The general rule is that inconsistencies and contradictions in minor and trivial matters do

Appellants claim that the witness was uncertain as to who actually stabbed the victim. While it is true that initially he made a mistake in pinpointing the culprits in the courtroom, this was the result only of his lack of familiarity with appellants names. However, despite his initial confusion, his identification of the culprit who delivered the fatal stab wound was categorical. This [18] is clearly shown in the transcripts of the trial: Q. Earlier, you pointed to two persons inside the courtroom as having participated in the stabbing of Crispin Gatmen, the first one gave his name as Monchito Gotangugan, will you tell the Honorable Court what was the special participation of Monchito Gotangugan in the stabbing? A. The gun was handed over to him.

COURT: Gun? A. Yes, Your Honor. FISCAL BELTRAN: Q. How about the other person, this Domingo Salazar, what did he do? A. He was the one who stabbed the victim. Q. Who stabbed the victim? A. The bald one, sir. INTERPRETER: Witness pointing to a person by the name of Monchito Gotangugan. ATTY. OSORIO: Your Honor, may I reiterate the witness to speak louder. FISCAL BELTRAN: Q Alright. You pointed to Monchito Gotangugan as the one who stabbed Crispin Gatmen. How about the other person you pointed to, what did he do? A. After the bald one stabbed the victim, sir, the big one took the gun from the security guard. Indeed, the foregoing testimony demonstrates that while the witness was confused as to the name of the culprit, he was certain about his identity.

infirmity as fruits of the poisonous tree. Considering that their conviction could be secured on the strength of the testimonial evidence given in open court which are not inadmissible in evidence, the court finds no reason to further belabor the matter.

Elements of Robo con Homicidio

In prosecuting robbery with homicide cases, the government needs to prove the following elements: (1) the taking of personal property is committed with violence or intimidation against persons; (2) the property taken belongs to another; (3) the taking is done with animo lucrandi; and (4) by reason of the robbery or on the occasion thereof, homicide (used in its generic sense) is [21] committed. In this case, the prosecution has convincingly proven that (1) appellants asported a gun with violence and intimidation against the victim; (2) the gun belonged to the deceased; and (3) the security guard was killed. Animus lucrandi is presumed when there is proof [22] of asportation. All of these facts are supported by the testimonies of competent eyewitnesses presented by the prosecution. There is, however, no showing that the death of the security guard occurred merely by reason or on the occasion of the robbery. The prosecution was silent on appellants primary criminal intent. Did they intend to kill the security guard in order to steal the gun? Or did they intend only to kill him, the taking of the gun being merely an afterthought? The prosecution did not prove either of the two propositions, and the court a quo failed to elaborate on this point. Thus, we cannot affirm appellants conviction of the crime charged in the Information. In several cases, the Court has already ruled that a conviction for robbery with homicide requires certitude that the robbery was the main purpose and objective of the criminals and that the killing was merely incidental, resulting merely by reason or on the [23] occasion of the robbery. Article 294 of the Revised Penal Code specifically states: Art. 294. Robbery with violence against or intimidation of persons-Penalties.--Any person guilty of robbery with the use of violence against or intimidation of any person shall suffer: 1. the penalty or reclusin perpetua to death, when by reason or on occasion of the robbery, the crime of homicide shall have been committed, x x x. (Underscoring supplied) The Spanish version of Article 294 (1) of the Revised Penal Code reads: 1.0--Con la pena de reclusion perpetua a muerte, cuando con motivo o con ocasion del robo resultare homicidio. Chief Justice Ramon C. Aquino explains that the use of the words con motivodel robo permits of no interpretation other than that the intent of the actor must supply the connection between the homicide and the robbery in order to constitute the complex offense. If that intent comprehends the robbery, it is immaterial that the homicide may in point of time immediately precede instead of follow the robbery. Where the original design comprehends robbery, and homicide is perpetrated by reason or on the occasion of the consummation of the former, the crime committed is the special complex offense, even if homicide precedes the robbery by an appreciable interval of time. On the other hand, if the original criminal design does not clearly comprehend robbery, but robbery follows the homicide as an afterthought or as a minor incident of the homicide, the criminal acts should be viewed as constitutive of two offenses and not of a single complex offense. Robbery with homicide arises only when there is a direct relation, an intimate connection, between the robbery and the killing, even if the killing is [24] prior to, concurrent with, or subsequent to the robbery. On the other hand, robbery with homicide under Article 294 of the Code is distinguished from the complex crime punished in Article 48, which contemplates a situation where one offense is a necessary means to commit the other or where a single act results in two or more offenses. The homicide in Article 294(1) is not necessary for [25] the accomplishment of the robbery. However, it could be

Eyewitnesses Accounts Are Consistent with Each Other

The defense claims that the testimonies of the two eyewitnesses materially contradicted each other in two ways. First, Miranda allegedly said that the security guard was standing when he was stabbed while Soriano stated that he was sitting down. Second, Miranda testified that one of the culprits produced the dagger and the other stabbed the victim with it. Soriano, on the other hand, said that only one person produced the dagger and thereafter stabbed the victim. We have examined the Appellants Brief and the records of this case and we have found no factual basis for the strained inferences of the defense. From Mirandas sworn statement, the defense deduced that the security guard was standing outside the outpost. But this deduction is not supported by said sworn statement or by the witness testimony in court. Miranda never said that the victim was standing up or was outside the guardhouse when he was stabbed. Both witnesses agreed that the person who stabbed the victim was Gotangugan. Both eyewitnesses corroborated each other in identifying Salazar as the one who drew the dagger from his jacket and then handed it to Gotangugan. With the dagger, Gotangugan stabbed the security guard. The insistence of the defense on this supposed contradiction simply has no basis. Time and again, this Court has ruled that the assessment of the trial court on the credibility of witnesses and their stories is wellnigh conclusive on appeal, provided it is not tainted with arbitrariness or oversight of some fact or circumstance of weight [19] and influence. In this case, the defense has tried but failed to establish any material inconsistency or contradiction which would justify a departure from this rule. Compared with the evidence submitted by the prosecution, appellants denial and alibi cannot possibly be given more probative weight than the clear and positive identification provided by no less [20] than two credible eyewitnesses. Granting arguendo that appellants were illegally arrested, such arrest did not invest these eyewitness accounts with constitutional

committed to avoid future identification of the robbers or as a consequence or incident thereof. Robo con homicidio is an indivisible offense, a special complex crime. The penalty for robbery with homicide is more severe because the law sees, in this crime, that men placed lucre above the value of human life, thus, justifying the imposition of a more severe penalty than that for simple homicide or robbery. In view of said graver penalty, jurisprudence exacts a stricter requirement before convicting the accused of this crime. Where the homicide is not conclusively shown to have been committed for the purpose of robbing the victim, or where the robbery was not proven, there can [26] be no conviction for robo con homicidio. In the case under consideration, appellants primary intent remains an enigma. For this reason, we cannot affirm appellants conviction for robbery with homicide. The fact that appellants took the firearm after shooting the security guard did not prove that their primary intent was to commit robbery. It shows that they committed an unlawful taking of property, but it does not exclude the possibility that this was merely an afterthought. Any conclusion as to their primary criminal intent based on the proven facts is speculative and without adequate basis. In view of the facts established and consistent with jurisprudence, the Court can convict appellants only of the separate offenses of theft and homicide, which were both duly proven. This Court is cognizant of the fact that the Information accused appellants of the crime of robbery with homicide. Nonetheless, it is axiomatic that the nature and character of the crime charged are determined not by the designation of the specific crime but by the [27] facts alleged in the Information. Thus, in People vs. Ponciano, the Court through Mr. Justice Hugo E. Gutierrez, Jr. held: x x x In the case at bar, the direct relation or intimate connection between the robbery and the killing was not established. We therefore, follow the rule laid down in People v. Manalang [28] [170 SCRA 149,163, February 9,1989], to wit: We already had several occasions to hold that if the original design was not to commit robbery but that the idea of taking the personal property of another with intent to gain came to the mind of the offender after the homicide only as an afterthought or as a minor incident in the homicide, the criminal acts should be viewed as constituting two distinct offenses and not as a single complex crime; the crimes would be either homicide or murder, as the case may be, and theft. (People v. Atanacio, et al., No. L-11844, November 29, 1960, 110 Phil. 1032; People v. Elizaga, 86 Phil. 364 [1950]; People v. Glore, 87 Phil. 739 *1950+) (Underscoring supplied) Thus, appellants should be held guilty of homicide under Article 249 of the Revised Penal Code and theft under Article 309 of the same Code. We also hold that treachery aggravated the killing. The attack was sudden and without warning, affording the [29] security guard no chance to defend himself. As it was not alleged in the Information, it cannot be used to qualify the killing to murder. However, treachery can still be considered as a generic [30] aggravating circumstance. While it was proven during the trial that the stolen pistol was [31] worth P6,500.00, the Information placed the value at P6,000.00 only. However, the appellant did not object to the higher valuation and is thus deemed to have waived his right to avail of the lower penalty under paragraph 3 of Article 309 of the Revised Penal Code. Consequently, appellants may be penalized for theft under [32] Article 309 (2) of the said Code. It is scarcely necessary to point out that there was conspiracy between appellants, because they clearly acted in concert and with [33] a unified criminal design. The eyewitness accounts tell us that one of the assailants touched the other appellants behind to signal the start of the attack against the security guard. Salazar then pulled out the dagger which Gotangugan used to stab the victim. WHEREFORE, the assailed Judgment is hereby MODIFIED as follows:

(1) Appellants are hereby found GUILTY of the separate offense of homicide and SENTENCED to the indeterminate sentence of ten years and one day of prisin mayor, as minimum, to seventeen years, four months and one day of reclusin temporal, as maximum. (2) The indemnity ex delicto imposed by the trial court is INCREASED to fifty thousand pesos (P50,000.00) in [34] line with prevailing jurisprudence. (3) Appellants are found also GUILTY of the separate offense of theft in accordance with Article 309 (2) of the Revised Penal Code and SENTENCED to the indeterminate penalty of six months and one day of prision correccional, as minimum, to four years and two months and one day also of prision correccional, as maximum. (4) Costs against appellants. SO ORDERED. Narvasa, C.J., (Chairman), Davide, Jr., Melo, and Francisco, JJ., concur. Espano v. CA, 288 SCRA 588 ROMERO, J.: This is a petition for review of the decision of the Court of [1] Appeals in CA-G.R. CR No. 13976 dated January 16, 1995, which affirmed in toto the judgment of the Regional Trial Court of Manila, Branch 1, convicting petitioner Rodolfo Espano for violation of Article II, Section 8 of Republic Act No. 6425, as amended, otherwise known as the Dangerous Drugs Act. Petitioner was charged under the following information: That on or about July 14, 1991, in the City of Manila, Philippines, the said accused, not being authorized by law to possess or use any prohibited drug, did then and there wilfully, unlawfully and knowingly have in his possession and under his custody and control twelve (12) plastic cellophane (bags) containing crushed flowering tops, marijuana weighing 5.5 grams which is a prohibited drug. Contrary to law.
[2]

The evidence for the prosecution, based on the testimony of Pat. Romeo Pagilagan, shows that on July 14, 1991, at about 12:30 a.m., he and other police officers, namely, Pat. Wilfredo Aquino, Simplicio Rivera, and Erlindo Lumboy of the Western Police District (WPD), Narcotics Division went to Zamora and Pandacan Streets, Manila to confirm reports of drug pushing in the area. They saw petitioner selling something to another person. After the alleged buyer left, they approached petitioner, identified themselves as policemen, and frisked him. The search yielded two plastic cellophane tea bags of marijuana. When asked if he had more marijuana, he replied that there was more in his house. The policemen went to his residence where they found ten more cellophane tea bags of marijuana. Petitioner was brought to the police headquarters where he was charged with possession of [3] prohibited drugs. On July 24, 1991, petitioner posted bail and the [4] trial court issued his order of release on July 29, 1991. Annabelle Alip, forensic chemist of the WPD Criminal Investigation Laboratory Section, testified that the articles sent to her by Pat. Wilfredo Aquino regarding the apprehension of a certain Rodolfo Espano for examination tested positive for marijuana, with a total weight of 5.5 grams. By way of defense, petitioner testified that on said evening, he was sleeping in his house and was awakened only when the policemen handcuffed him. He alleged that the policemen were looking for his brother-in-law Lauro, and when they could not find the latter, he was instead brought to the police station for investigation and later indicted for possession of prohibited drugs. His wife Myrna corroborated his story.

The trial court rejected petitioners defense as a mere afterthought and found the version of the prosecution more credible and trustworthy. Thus, on August 14, 1992, the trial court rendered a decision, convicting petitioner of the crime charged, the dispositive portion of which reads: WHEREFORE there being proof beyond reasonable doubt, the court finds the accused Rodolfo Espano y Valeria guilty of the crime of violation of Section 8, Article II, in relation to Section 2 (e-L) (I) of Republic Act No. 6425 as amended by Batas Pambansa Blg. 179, and pursuant to law hereby sentences him to suffer imprisonment of six (6) years and one (1) day to twelve (12) years and to pay a fine of P6,000.00 with subsidiary imprisonment in case of default plus costs. The marijuana is declared forfeited in favor of government and shall be turned over to the Dangerous Drugs Board without delay. SO ORDERED.
[5]

the scene of the crime at the time of its commission and that it was physically impossible for him to be there. Moreover, the claim of a frame-up, like alibi, is a defense that has been invariably viewed by the Court with disfavor for it can just as easily be concocted but difficult to prove, and is a common and standard line of defense in most prosecutions arising from violations of the Dangerous Drugs [9] Act. No clear and convincing evidence was presented by petitioner to prove his defense of alibi. Second, petitioner contends that the prosecutions failure to present the alleged informant in court cast a reasonable doubt which warrants his acquittal. This is again without merit, since failure of the prosecution to produce the informant in court is of no moment especially when he is not even the best witness to establish the fact that a buy-bust operation had indeed been conducted. In this case, Pat. Pagilagan, one of the policemen who apprehended petitioner, testified on the actual incident of July 14, 1991, and identified him as the one they caught in possession of prohibited drugs. Thus, We find that the prosecution had satisfactorily proved its case against appellants. There is no compelling reason for us to overturn the finding of the trial court that the testimony of Sgt. Gamboa, the lone witness for the prosecution, was straightforward, spontaneous and convincing. The testimony of a sole witness, if credible and positive and satisfies the court beyond reasonable doubt, is [10] sufficient to convict. Thus on the basis of Pat. Pagilagans testimony, the prosecution was able to prove that petitioner indeed committed the crime charged; consequently, the finding of conviction was proper. Lastly, the issue on the admissibility of the marijuana seized should likewise be ruled upon. Rule 113 Section 5(a) of the Rules of Court provides: A peace officer or a private person may, without a warrant, arrest a person: a. when, in his presence, the person to be arrested has committed, is actually committing, or is attempting to commit an offense; xxx xxx x x x.

Petitioner appealed the decision to the Court of Appeals. The appellate court, however, affirmed the decision of the trial court in toto. Hence, this petition. Petitioner contends that the trial and appellate courts erred in convicting him on the basis of the following: (a) the pieces of evidence seized were inadmissible; (b) the superiority of his constitutional right to be presumed innocent over the doctrine of presumption of regularity; (c) he was denied the constitutional right of confrontation and to compulsory process; and (d) his conviction was based on evidence which was irrelevant and not properly identified. After a careful examination of the records of the case, this Court finds no compelling reason sufficient to reverse the decisions of the trial and appellate courts. First, it is a well settled doctrine that findings of trial courts on the credibility of witnesses deserve a high degree of respect. Having observed the deportment of witnesses during the trial, the trial judge is in a better position to determine the issue of credibility and, thus, his findings will not be disturbed during appeal in the absence of any clear showing that he had overlooked, misunderstood or misapplied some facts or circumstances of weight and substance [6] which could have altered the conviction of the appellants. In this case, the findings of the trial court that the prosecution witnesses were more credible than those of the defense must stand. Petitioner failed to show that Pat. Pagilagan, in testifying against him, was motivated by reasons other than his duty to curb drug abuse and had any intent to falsely impute to him such a serious crime as possession of prohibited drugs. In the absence of such ill motive, the presumption of regularity in the performance of his official duty must prevail. In People v. Velasco, this Court reiterated the doctrine of presumption of regularity in the performance of official duty which provides: x x x. Appellant failed to establish that Pat. Godoy and the other members of the buy-bust team are policemen engaged in mulcting or other unscrupulous activities who were motivated either by the desire to extort money or exact personal vengeance, or by sheer whim and caprice, when they entrapped her. And in the absence of proof of any intent on the part of the police authorities to falsely impute such a serious crime against appellant, as in this case, the presumption of regularity in the performance of official duty, . . ., must prevail over the self-serving and uncorroborated claim of [8] appellant that she had been framed. Furthermore, the defense set up by petitioner does not deserve any consideration. He simply contended that he was in his house sleeping at the time of the incident. This Court has consistently held that alibi is the weakest of all defenses; and for it to prosper, the accused has the burden of proving that he was not at
[7]

Petitioners arrest falls squarely under the aforecited rule. He was caught in flagranti as a result of a buy-bust operation conducted by police officers on the basis of information received regarding the illegal trade of drugs within the area of Zamora and Pandacan Streets, Manila. The police officer saw petitioner handing over something to an alleged buyer. After the buyer left, they searched him and discovered two cellophanes of marijuana. His arrest was, therefore, lawful and the two cellophane bags of marijuana seized were admissible in evidence, being the fruits of the crime. As for the ten cellophane bags of marijuana found at petitioners residence, however, the same are inadmissible in evidence. The 1987 Constitution guarantees freedom against unreasonable searches and seizures under Article III, Section 2 which provides: The right of the people to be secure in their persons, houses, papers and effects against unreasonable searches and seizures of whatever nature and for any purpose shall be inviolable, and no search warrant or warrant of arrest shall issue except upon probable cause to be determined personally by the judge after examination under oath or affirmation of the complainant and the witnesses he may produce, and particularly describing the place to be searched and the persons or things to be seized. An exception to the said rule is a warrantless search incidental to a lawful arrest for dangerous weapons or anything which may be [11] used as proof of the commission of an offense. It may extend beyond the person of the one arrested to include the premises or surroundings under his immediate control. In this case, the ten

cellophane bags of marijuana seized at petitioners house after his arrest at Pandacan and Zamora Streets do not fall under the said exceptions. In the case of People v. Lua,
[12]

this Court held:

As regards the brick of marijuana found inside the appellants house, the trial court correctly ignored it apparently in view of its inadmissibility. While initially the arrest as well as the body search was lawful, the warrantless search made inside the appellants house became unlawful since the police operatives were not armed with a search warrant. Such search cannot fall under search made incidental to a lawful arrest, the same being limited to body search and to that point within reach or control of the person arrested, or that which may furnish him with the means of committing violence or of escaping. In the case at bar, appellant was admittedly outside his house when he was arrested. Hence, it can hardly be said that the inner portion of his house was within his reach or control. The articles seized from petitioner during his arrest were valid under the doctrine of search made incidental to a lawful arrest. The warrantless search made in his house, however, which yielded ten cellophane bags of marijuana became unlawful since the police officers were not armed with a search warrant at the time. Moreover, it was beyond the reach and control of petitioner. In sum, this Court finds petitioner Rodolfo Espano guilty beyond reasonable doubt of violating Article II, Section 8, in relation to Section 2 (e-L) (I) of Republic Act No. 6425, as amended. Under the said provision, the penalty imposed is six years and one day to twelve years and a fine ranging from six thousand to twelve thousand pesos. With the passage of Republic Act No. 7659, which took effect on December 31, 1993, the imposable penalty shall now depend on the quantity of drugs recovered. Under the provisions of Republic Act No. 7629, Section 20, and as interpreted in People v. [13] [14] Simon and People v. Lara, if the quantity of marijuana involved is less than 750 grams, the imposable penalty ranges from prision correccional to reclusion temporal.Taking into consideration that petitioner is not a habitual delinquent, the amendatory provision is favorable to him and the quantity of marijuana involved is less than 750 grams, the penalty imposed under Republic Act No. 7659 should be applied. There being no mitigating nor aggravating circumstances, the imposable penalty shall be prision correccional in its medium period. Applying the Indeterminate Sentence Law, the maximum penalty shall be taken from the medium period of prision correccional, which is two (2) years, four (4) months and one (1) day to four (4) years and two (2) months, while the minimum shall be taken from the penalty next lower in degree, which is one (1) month and one (1) day to six (6) months of arresto mayor. WHEREFORE, the instant petition is hereby DENIED. The decision of the Court of Appeals in C.A.-G.R. CR No. 13976 dated January 16, 1995 is AFFIRMED with the MODIFICATION that petitioner Rodolfo Espano is sentenced to suffer an indeterminate penalty of TWO (2) months and ONE (1) day of arresto mayor, as minimum to TWO (2) years, FOUR (4) months and ONE (1) day of prision correccional, as maximum. SO ORDERED. Narvasa, C.J., (Chairman), Kapunan, and Purisima, JJ., concur. Pp v. Chua Ho San, 308 SCRA 432 FACTS OF THE CASE: In response to reports of rampant smuggling of firearms and other contraband, Chief of Police Jim Lagasca Cid of Bacnotan Police Station, La Union began patrolling the Bacnotan coastline with his officers. While monitoring the coastal area of Barangay Bulala, he intercepted a radio call at around 12:45 p.m. from Barangay Captain Juan Almoite of Barangay Tammocalao requesting for police assistance regarding an unfamiliar speedboat the latter had spotted. According to Almoite, the vessel looked different from the boats ordinarily used by fisherfolk of the area and was poised to dock at Tammocalao shores. Cid and six of his men led by SPO1 Reynoso Badua, proceeded immediately to Tammocalao beach and there

conferred with Almoite. Cid then observed that the speedboat ferried a lone male passenger, who was later identified as Chua Ho San. When the speed boat landed, the male passenger alighted, carrying a multicolored strawbag, and walked towards the road. Upon seeing the police officers, the man changed direction. Badua held Chuas right arm to prevent him from fleeing. They then introduced themselves as police officers; however, Chua did not understand what theyre saying. And by resorting of sign language, Cid motioned with his hands for the man to open his bag. The man acceded to the request. The said bag was found to contain several transparent plastics containing yellowish crystalline substances, which was later identified to be methamphetamine hydrochloride or shabu. Chua was then brought to Bacnotan Police Station, where he was provided with an interpreter to inform him of his constitutional rights. ISSUE: Whether or not the warrantless arrest, search and seizure conducted by the Police Officers constitute a valid exemption from the warrant requirement. RULING: The Court held in the negative. The Court explains that the Constitution bars State intrusions to a person's body, personal effects or residence except if conducted by virtue of a valid of a valid search warrant issued in accordance with the Rules. However, warrantless searches may be permitted in the following cases, to wit: (1)search of moving vehicles, (2)seizure in plain view, (3)customs searches, (4)waiver or consent searches, (5)stop and frisk situations (Terry search), and (6)search incidental to a lawful arrest. It is required in cases of in flagrante delicto that the arresting officer must have personal knowledge of such facts or circumstances convincingly indicative or constitutive of probable cause. Probable cause means a reasonable ground of suspicion supported by circumstances sufficiently strong in themselves to warrant a cautious man's belief that the person accused is guilty of the offense with which he is charged. In the case at bar, there are no facts on record reasonably suggestive or demonstrative of CHUA's participation in on going criminal enterprise that could have spurred police officers from conducting the obtrusive search. CHUA was not identified as a drug courier by a police informer or agent. The fact that the vessel that ferried him to shore bore no resemblance to the fishing boats of the area did not automatically mark him as in the process of perpetrating an offense. With these, the Court held that there was no probable cause to justify a search incidental to a lawful arrest. The Court likewise did not appreciate the contention of the Prosecution that there was a waiver or consented search. If CHUA could not understand what was orally articulated to him, how could he understand the police's "sign language?" More importantly, it cannot logically be inferred from his alleged cognizance of the "sign language" that he deliberately, intelligently, and consciously waived his right against such an intrusive search. Finally, being a forbidden fruit, the subject regulated substance was held to be inadmissible in evidence. Hence, the accused was acquitted as the evidence was not sufficient to establish guilt beyond reasonable doubt. stop and frisk (Terry search) CASE: Terry v. Ohio, 392 U.S. 1 Brief Fact Summary. The Petitioner, John W. Terry (the Petitioner), was stopped

and searched by an officer after the officer observed the Petitioner seemingly casing a store for a potential robbery. The officer approached the Petitioner for questioning and decided to search him first. Synopsis of Rule of Law. An officer may perform a search for weapons without a warrant, even without probable cause, when the officer reasonably believes that the person may be armed and dangerous. Facts. The officer noticed the Petitioner talking with another individual on a street corner while repeatedly walking up and down the same street. The men would periodically peer into a store window and then talk some more. The men also spoke to a third man whom they eventually followed up the street. The officer believed that the Petitioner and the other men were casing a store for a potential robbery. The officer decided to approach the men for questioning, and given the nature of the behavior the officer decided to perform a quick search of the men before questioning. A quick frisking of the Petitioner produced a concealed weapon and the Petitioner was charged with carrying a concealed weapon. Issue. Whether a search for weapons without probable cause for arrest is an unreasonable search under the Fourth Amendment to the United States Constitution (Constitution)? Held. The Supreme Court of the United States (Supreme Court) held that it is a reasonable search when an officer performs a quick seizure and a limited search for weapons on a person that the officer reasonably believes could be armed. A typical beat officer would be unduly burdened by being prohibited from searching individuals that the officer suspects to be armed. Dissent. Justice William Douglas (J. Douglas) dissented, reasoning that the majoritys holding would grant powers to officers to authorize a search and seizure that even a magistrate would not possess. Concurrence. Justice John Harlan (J. Harlan) agreed with the majority, but he emphasized an additional necessity of the reasonableness of the stop to investigate the crime. Justice Byron White (J. White) agreed with the majority, but he emphasized that the particular facts of the case, that there was suspicion of a violent act, merit the forcible stop and frisk. Discussion. The facts of the case are important to understand the Supreme Courts willingness to allow the search. The suspicious activity was a violent crime, armed robbery, and if the officers suspicions were correct then he would be in a dangerous position to approach the men for questioning without searching them. The officer also did not detain the men for a long period of time to constitute an arrest without probable cause. consented search CASES: Pp v. Tabar, 222 SCRA 144

Philippines, and within the jurisdiction of this Honorable Court, the said accused, conniving and confederating together and mutually helping each other, with deliberate intent, did then and there sell and deliver, without authority of law, Three (3) sticks of marijuana cigarettes, a (sic) prohibited drugs, to a person who posted himself as a buyer, in Viol. of Sec. 4, Art. 11, of RA 6425, as amended, otherwise known as the 1 Dangerous Act of 1972. The case was docketed as Criminal Case No. CBU-14863 and after it was raffled off to Branch 15 of the said court, the accused were forthwith arraigned. Carmelina entered a plea of not guilty while Rommel, then seventeen (17) years of age, with the conformity of the prosecution, entered a plea of guilty to the lesser offense of possession of marijuana under Section 8, Article II of R.A. No. 6425, 2 as amended. As a consequence of his plea, the trial court handed down on 24 April 1989 an Order which reads in part as follows: Therefore this court being satisfied that the accused herein is the same Ramil Tabar described in Annex 1 (Certificate of Birth) of the said accused, and it appearing that he is still a minor (17 years), he is entitled to a suspended sentence of the penalty for possession of marijuana which is a jail term of six (6) years and one (1) day to twelve (12) years and a fine of Six Thousand (P6,000.00) pesos (Sec. 8 RA 6425 as amended of B.P. 179, March 2, 1982). WHEREFORE, the accused Rommel Tabar y Arriesgado is hereby discharged on probation (Sec. 32 of RA 6425 as amended by B.P. 179) and committed to the custody of the Department of Social Welfare and Development, Cebu Regional Office (No. 7) until he reaches the age of majority, or otherwise finally discharged upon orders of this court pursuant to P.D. 603 and B.P. 179, but to be placed under the Supervision of the Dangerous Drugs Board, the alleged crime being drug related, and for a period of one (1) year from date hereof. The Regional Director of the DSWD is hereby ordered to conduct and submit a case study of the accused minor to this court, within sixty days and to report on his conduct once every four months, to 3 this court. Thereafter, trial proceeded as against Carmelina alone. The prosecution presented Pfc. Josephus Trangia and Myra P. Arreola, a forensic analyst of the PC Crime Laboratory Service, as its witnesses. The testimony of Pfc. Raul Tumakay was ordered stricken out since he could not be cross-examined. The defense had only Carmelina as its witness. On 22 December 1990, the trial court promulagated its decision, 4 dated 17 December 1990, finding Carmelina "guilty, beyond reasonable doubt, for (sic) violation of Section 4, Article II RA 6425, otherwise known as the Dangerous Drug Act of 1972 as amended by PD 1675" and sentencing her to "Reclusion Perpetua and to pay a 5 fine of P20,000.00 for the act of selling and distributing marijuana." The conviction is premised on the following findings of fact: From the evidence which consists of the testimony of Pfc. Josephus Trangia, the court gathered that at about 3:00 P.M. of February 8, 1989, he was with Pfc. Romeo Cortes and Gualberto Gabales on a buy-bust operation for marijuana after receipt of information about marijuana pushers in Punta Princesa, Cebu City and that they had their informant go ahead of them after giving the P5.00 bill for him to purchase marijuana. He continued saying that their informant stood in front of a shanty while they posted themselves at a distance of about 50 meters from the place where their informant was standing. And that they saw a young boy approached their informant and handed cigarettes to him who in turn handed the marked money to the young boy. Then, their informant gave them the pre-arranged signal of scratching his head with his right hand; that after the signal, he and his companions immediately approached the young boy and the informant introduced them as police officers. This young boy was

DAVIDE JR., J.: Carmelina Tabar y Carmilotes and her nephew, Rommel Arriesgado y Tabar, of Tres de Abril, Punta Princesa, Cebu City, were charged with the violation of Section 4, Article II of R.A.No. 6425, as amended, in an Information filed by the Office of the City Fiscal of Cebu City with the Regional Trial Court of Cebu City on 9 February 1989, the accusatory portion of which reads a follows: That on or about the 8th day of February 1989, at about 3:00 PM. in the City of Cebu,

about 16-17 years old, by the name of Rommel Arriesgado y Tabar. He had earlier pleaded guilty to the lesser offense of mere possession of marijuana and was, in fact, already convicted by this court. Upon being shown a P5.00 bill with the initials written thereon as: GDG-89 and bearing SL L F 637396, he identified the same bill as the one given to their informant and marked as Exh. "A" for the prosecution. He explained the initials GDG which stands for Gualberto G. Gabales, his team member. He further declared that after the pre-arranged signal from their informant, they immediately proceeded to the scene and were given three sticks of marijuana by their informant after buying the same from the boy, Rommel Arriesgado and that they proceeded to confiscate the P5.00 bill from the boy. At this juncture, he claimed that he observed that after the transaction, the boy went inside the shanty and the moment he got out, he handed the three sticks of marijuana to the informant. In fact, he claimed that after the proceeding to the shanty, they met Carmelina Tabar, accused herein, and that Carmelina Tabar was holding a white pants from where they found other marijuana sticks in cigarette packs which they confiscated. That they brought Carmelina Tabar to Fuente Police Station for investigation. He claimed that there were 75 sticks of marijuana in the Hope Cigarette pack; 22 sticks of marijuana cigarettes in the Philip Morris pack and 99 sticks of marijuana in the Mark cigarette pack. He said that they also confiscated the pants, but only the marijuana sticks were submitted for testing to the PC Crime Laboratory. That pursuant to this requested analysis, Lieut. Fortunato Quijon of the Police issued a Certification of Field Test, Exh. "B" which showed that three sticks of handrolled cigarettes marked Rommel-89 were positive for marijuana. Shown the three packs of cigarettes distincly marked as Hope, Mark and Philip Morris, he identified the same as the ones confiscated from Carmelina Tabar. So did he identify the creamcolored pants he said they confiscated from Carmelina Tabar and which according to him was used to wrap marijuana sticks inside the pack. He finally told the court that this team was composed of Gualberto D. Gabales, Romeo Cortes, Pfc. Tumakay and himself. On cross-examination, this witness affirmed that when he asked the boy, Rommel where he got the marijuana sticks, he was told he got it from his aunt, the accused herein. He further told the court that they arrested Carmelina Tabar later. When the young boy went inside, they presumed the marijuana came from inside the shanty and that when the accused Carmelina Tabar went out, suspiciouslooking and pale and afraid to face them, they told her to stop from going left towards the houses and asked her to open the pants which revealed the three cigarette packs contaning marijuana. The witness candidly admitted they had no search warrant at the time they effected the arrest and confiscation. From the testimonies of Mrs. Myrna Areola, Police Lieutenant, Forensic Analyst of the PC Crime Laboratory, it was established that the specimens submitted to her were positive of marijuana. She then identified Exh. "C", as her Chemistry Report C-038-89. She also identified her signature, Exh. "C-4" thereon and her findings "All are positive of Marijuana", Exh. "C-3"; the specimens submitted as Exh. "C-2" and the name of the subject, Carmelina Tabar as Exh. "C-1". She confirmed her findings on all handrolled cigarettes in all cigarette packs; Hope, Exh. "E-1" to "E-75"; Philip Morris pack, Exh. "F-1" to "F-22" and Mark cigarette pack, Exh. "G-1" to "G-99". She informed the court that the specimens were submitted to her by Pfc Gabales on February 11, 1989, at about 10:50 A.M., and that she examined the sticks one by one and handrolled them again. She also claimed that the police did not ask for a copy of her report and that this is the first time it is presented in court. She informed the court that she placed the specimens in her evidence store room, with 6 keys, she herself kept. The trial court discredited the bare denials of Carmelina and unfavorably considered against her an admission that she had been arrested before by the CANU for possession of marijuana, was charged for the violation of Section 8, Article II of R.A. No. 6425 in Criminal Case No. CBU-8573, was convicted therein, but is now on 7 probation. It further considered against her an allegedly very damaging admission, thus: She made a very damaging admission to the court when the Presiding Judge asked her whether it is not true that she kept on crying because she was caught again and she said, "Yes" and at

which juncture she admitted to the court that she was serving 8 probation for the same offense. It then concluded that: [A]ccused actually employed her nephew, Rommel Arriesgado to sell marijuana from her store and that she has been in that illicit business for quite sometime now. The evidence notwithstanding, talks in the community where the accused lives is rife with accusations (sic) that she is indeed engaged with members of the family, in the sale and distribution of prohibited drugs such as marijuana. Between the positive testimony of the arresting officers, who appear to be more credible than the accused's worthless and untrustworthy denials, the court gives credence to the evidence of 9 the prosecution. Unable to accept the verdict, Carmelina filed her notice of 10 appeal manifesting therein that she is appealing from the decision 11 to the Court of Appeals. In its Order of 27 February 1991, the trial court gave due course to the appeal and directed the clerk of court "to submit all the records, evidences (sic) and trancripts of this proceeding to the Hon. Court of Appeals, for proper disposition." Considering that the penalty imposed is reclusion perpetua, the Court of Appeals transmitted to this Court the records of the case on 12 12 August 1991. In the Resolution of 11 September 1991, this Court accepted the appeal. In her Appellant's Brief, Carmelina, hereinafter referred to as the appellant, imputes upon the trial court the commission of the following errors in the appealed decision: I. . . . IN CONVICTING ACCUSED-APPELLANTS (sic) OF VIOLATION OF SECTION 4, ARTICLE II, REPUBLIC ACT 6425 AS AMENDED WHEN THE EVIDENCE DOES NOT WARRANT IT. II. . . . IN ADMITTING EVIDENCE SEIZED WITHOUT 14 ANY SEARCH WARRANT. As to the first assigned error, the appellant claims that the prosecution presented no evidence that she sold marijuana and since there exists no convincing, positive and conclusive proof of conspiracy between her and her co-accused, Rommel Arriesgado, she cannot be held liable for violation of Section 4, Article II of R.A. No. 6425, as amended. In support of the second assigned error, the appellant maintains that the marijuana cigarettes seized from her are inadmissible in evidence because they were obtained in violation of the constitutional guarantee against unreasonable search and seizure. After a careful perusal of the records and evaluation of the evidence, this Court is inclined to agree with the appellant that she should not be convicted under Section 4, Article II of R.A. No, 6425. We rule, however, that she is liable under Section 8, Article II of the said Act. Her conviction by the trial court under Section 4 is primarily based on its conclusion that the appellant "actually employed her nephew Rommel Arriesgado to sell marijuana from her store and that she had been in that illicit business for quite sometime now." This conclusion is based on the trial court's sweeping statement that "talks in the community where the accused lives is rife with accusations that she is indeed engaged with members of the family, in the sale and distribution of prohibited drugs such as marijuana." We find no evidence on record to sustain this charge. It may thus be said that such a conclusion is not based on established facts but on "talks in the community." If indeed such was the fact, it would not have been difficult for the prosecution to provide the court with overwhelming evidence. Yet, it presented only Pat. Trangia who, rather unfortunately, did not even testify or volunteer information that the main target of the busy-bust operation was the appellant. He did not also disclose in his testimony that the appellant was
13

among the reported "pushers" in Punta Princesa, Cebu City. It may be recalled that the buy-bust operation on 8 February 1989 was conducted because, as he alleged: A. Before that time we have already received information from the community of Punta Princesa regarding marijuana pushers in that 15 place. Nevertheless, the prosecutor who conducted the direct-examination of Trangia did not ask further as to identity of the pushers such that it was not proven that the appellant was one of them. The Solicitor General, however, maintains that there was conspiracy, established by circumstancial evidence, between accused Rommel Arriesgado who was caught in flagrante selling three (3) sticks of handrolled marijuana to the informant and accepting the marked money. We are not persuaded since the evidence for the prosecution does not show that (a) the appellant was in the mind of the members of the team when they planned the buy-bust operation and when they carried out such plan, (b) the three (3) sticks of handrolled marijuana came from the appellant, and (c) the appellant used Rommel as her agent to sell the three (3) sticks to the informant. Moreover, if indeed the prosecution truly believed that such conspiracy existed, it should not have willingly given its conformity to Rommel's plea to the lesser offense of illegal possession of prohibited drugs under Section 8, Article II of R.A. No. 6425, as amended. Having been caught in flagrante for selling marijuana, it was not difficult to prove Rommel's culpability under Section 4, Article II of the Act. Yet it readily consented to his offer to plead guilty to the said lesser offense. It was, however, established beyond any shadow of doubt and, therefore, with moral certainty, that the appellant kept in her possession handrolled sticks of marijuana placed in empty Hope, 16 Philip Morris and Mark cigarrette packs. She does not have any authority to possess them. She may have acquired them with the intention to sell them for profit; but without proof of sale, she cannot be held liable under Section 4, Article II of the Dangerous Drugs Act. For such possession, her liability is covered by Section 8 of the said Article which penalizes possession or use of prohibited drugs. The last paragraph thereof reads: xxx xxx xxx The penalty of the imprisonment ranging from six years and one day to twelve years and a fine ranging from six thousand to twelve thousand pesos shall be imposed upon any person who, unless authorized by law, shall possess or use of Indian hemp. Indian hemp is otherwise known as Marijuana.
17

of the shanty you only saw the white long pants and not the cigarettes? A. Only the pants. Q. Did you say that in order to find out what was the contents of the pants, you asked her to open the pants. Isn't it? A. Yes. Q. Now, after she opened it, what did you see? A. Three (3) packs of marijuana cigarettes. Q. Who among you in your team approached Carmelina Tabar? A. It was PFC Raul 19 Tumakay. Even assuming ex gratia argumenti that the seach and seizure were without a warrant, the appellant had effectively waived her constitutional right relative thereto by voluntarily submitting to the seach and seizure. 20 In People vs. Malasugui, this Court ruled: When one voluntarily submits to a search and consent to have it made of his person or premises, he is precluded from later complaining thereof (Cooley, Constitutional Limitations, 8th ed., vol. I, pages 631). The right to be secure from unreasonable seach may, like every right, be waived and such waiver may be made either expressly or impliedly. The exclusionary rule relied upon by the appellant does not provide her safe refuge. Before We close this case, a final observation for the guidance of trial judges must be made. For the violation of Section 4, Article II of R. A. No. 6425, as amended, the trial court imposed the penalty ofreclusion perpetua. The penalty provided for therein is "life imprisonment to death and a fine ranging from twenty thousand to thirty thousand pesos." In view of Section 19(1), Article III of the 1987 Constitution which prohibits the imposition of the death penalty, the maximum penalty then imposable thereunder would only be life imprisonment. Life imprisonment, however, is not synonymous with reclusion perpetua. 21 We have reiterated this time and again and admonished judges to employ the proper legal terminology in the imposition of imprisonment penalties because of their different accompanying 22 legal accessories and effects. IN THE LIGHT OF THE FOREGOING, judgment is hereby rendered modifying the challenged Decision of Branch 15 of the Regional Trial Court of Cebu in Criminal Case No. CBU-14863 dated 17 December 1990 and, as modified, finding appellant CARMELINA TABAR y CARMILOTES guilty beyond reasonable doubt of illegal possession of marijuana under Section 8, Article II of R. A. No. 6425, otherwise known as the Dangerous Drugs Act of 1972, as amended, and, applying the Inderterminate Sentence Law, she is sentenced to suffer imprisonment of eight (8) years as minimum to twelve (12) years as maximum and to pay a fine of Ten Thousand Pesos (P10,000.00). Costs against the appellant.

Appellant, therefore, may specifically be penalized under the aforesaid last paragraph of Section 8, Article II of the Act. Applying 18 the Indeterminate Sentence Law, the penalty of eight (8) years as Minimum to twelve (12) years as Maximum and a fine of P10,000.00 may then be imposed upon her. The second assigned error is without merit. The evidence for the prosecution discloses that the appellant placed the packs of marijuana sticks under the rolled pair of pants which she was then carrying at the time she hurriedly left her shanty after noticing the arrest of Rommel. When she was asked to spread it out, which she voluntary did, the package containing the packs of marijuana sticks were thus exposed in plain view to the member of the team. A crime was thus committed in the presence of the policemen. Pursuant to Section 5, Rule 113 and Section 12 Rule 126 of the Revised Rules of Court, she could lawfully be arrested and searched for anything which may be used as proof of the commission of an offense without the corresponding arrest and search warrants. Her own counsel on cross-examination of prosecution witness Josephus Trangia further obtained a affirmation of these facts, thus: Q. You mean to say that when you saw Carmelina Tabar allegely went (sic) out

SO ORDERED. Feliciano, Bidin, Romero and Melo, JJ., concur.

RULE

113,

RULES

OF

COURT

Section 5. Arrest without warrant; when lawful. A peace officer or a private person may, without a warrant, arrest a person: (a) When, in his presence, the person to be arrested has committed, is actually committing, or is attempting to commit an offense; (b) When an offense has just been committed, and he has probable cause to believe based on personal knowledge of facts or circumstances that the person to be arrested has committed it; and (c) When the person to be arrested is a prisoner who has escaped from a penal establishment or place where he is serving final judgment or is temporarily confined while his case is pending, or has escaped while being transferred from one confinement to another. In cases falling under paragraph (a) and (b) above, the person arrested without a warrant shall be forthwith delivered to the nearest police station or jail and shall be proceeded against in accordance with section 7 of Rule 112.

Pp v. Aruta, 288 SCRA 626 Facts: On Dec. 13, 1988, P/Lt. Abello was tipped off by his informant that a certain Aling Rosa will be arriving from Baguio City with a large volume of marijuana and assembled a team. The nextday, at the Victory Liner Bus terminal they waited for the bus coming from Baguio, when the informer pointed out who Aling Rosa was, the team approached her and introduced themselves as NARCOM agents. When Abello asked aling Rosa about the contents of her bag, the latter handed it out to the police. They found dried marijuana leaves packed in a plastic bag marked cash katutak. Instead of presenting its evidence, the defense filed a demurrer to evidence alleging the illegality of the search and seizure of the items. In her testimony, the accused claimed that she had just come from Choice theatre where she watched a movie Balweg. While about tocross the road an old woman asked her for help in carrying a shoulder bag, when she was later on arrested by the police. She has no knowledge of the identity of the old woman and the woman was nowhere to be found. Also, no search warrant was presented. The trial court convicted the accused in violation of the dangerous drugs of 1972

RULE

126,

RULES

OF

COURT

Section 2. Court where application for search warrant shall be filed. An application for search warrant shall be filed with the following: a) Any court within whose territorial jurisdiction a crime was committed. b) For compelling reasons stated in the application, any court within the judicial region where the crime was committed if the place of the commission of the crime is known, or any court within the judicial region where the warrant shall be enforced. However, if the criminal action has already been filed, the application shall only be made in the court where the criminal action is pending. Section 7. Right to break door or window to effect search. The officer, if refused admittance to the place of directed search after giving notice of his purpose and authority, may break open any outer or inner door or window of a house or any part of a house or anything therein to execute the warrant or liberate himself or any person lawfully aiding him when unlawfully detained therein. Section 12. Delivery of property and inventory thereof to court; return and proceedings thereon. (a) The officer must forthwith deliver the property seized to the judge who issued the warrant, together with a true inventory thereof duly verified under oath. (b) Ten (10) days after issuance of the search warrant, the issuing judge shall ascertain if the return has been made, and if none, shall summon the person to whom the warrant was issued and require him to explain why no return was made. If the return has been made, the judge shall ascertain whether section 11 of this Rule has been complained with and shall require that the property seized be delivered to him. The judge shall see to it that subsection (a) hereof has been complied with. (c) The return on the search warrant shall be filed and kept by the custodian of the log book on search warrants who shall enter therein the date of the return, the result, and other actions of the judge. A violation of this section shall constitute contempt of court. custom search CASE: Roldan v. Arca, 65 SCRA 336 FACTS:Respondent company filed a case against Roldan, Jr. for the recovery of fishing vessel Tony Lex VI whichhad been seized and impounded by petitioner Fisheries Commissioner through the Philippine Navy. TheCFI Manila granted it, thus respondent company took Possession of the vessel Tony Lex VI.- P e t i t i o n e r r e q u e s t e d t h e Philippine Navy to apprehend vessels Tony Lex VI a n d T o n y L e x I I I , a l s o respectively called Srta. Winnie and Srta. Agnes, for alleged violations of some provisions of the FisheriesAct. On August 5 or 6, 1965, the two fishing boats were actually seized for illegal fishing with dynamite.

Issue: Whether or Not the police correctly searched and seized the drugs from the accused.

Held: The following cases are specifically provided or allowed by law: 1. Warrantless search incidental to a lawful arrest recognized under Section 12, Rule 126 of the Rules of Court 8 and by prevailing jurisprudence 2. Seizure of evidence in "plain view," the elements of which are: (a) a prior valid intrusion based on the valid warrantless arrest in which the police are legally present in the pursuit of their official duties; (b) the evidence was inadvertently discovered by the police who had the right to be where they are; (c) the evidence must be immediately apparent, and (d) "plain view" justified mere seizure of evidence without further search; 3. Search of a moving vehicle. Highly regulated by the government, the vehicle's inherent mobility reduces expectation of privacy especially when its transit in public thoroughfares furnishes a highly reasonable suspicion amounting to probable cause that the occupant committed a criminal activity; 4. 5. 6. 7. Exigent Stop and Consented Customs and Emergency warrantless search; search; Frisk; Circumstances.

The essential requisite of probable cause must still be satisfied before a warrantless search and seizure can be lawfully conducted. The accused cannot be said to be committing a crime, she was merely crossing the street and was not acting suspiciously for the Narcom agents to conclude that she was committing a crime. There was no legal basis to effect a warrantless arrest of the accuseds bag, there was no probable cause and the accused was not lawfully arrested. The police had more than 24 hours to procure a search warrant and they did not do so. The seized marijuana was illegal and inadmissible evidence.

ISSUE : WON the seizure of the vessel, its equipment and dynamites therein was valid. HELD: YES. Search and seizure without search warrant of vessels and air crafts for violations of thecustoms laws have been the traditional exception to the constitutional requirement of a search warrant,because the vessel can be quickly moved out of the locality or jurisdiction in which the search warrantmust be sought before such warrant could be secured; hence it is not practicable to require a searchwarrant before such search or seizure can be constitutionally effected. The same exception should applyto seizures of fishing vessels breaching our fishery laws. They are usually equipped with powerful motorsthat enable them to elude pursuing ships of the Philippine Navy or Coast Guard.Under our Rules of Court, a police officer or a private individual may, without a warrant, arrest a person(a) who has committed, is actually committing or is about to commit an offense in his presence; (b) whois reasonably believed to have committed an offense which has been actually committed; or (c) who is aprisoner who has escaped from confinement while serving a final judgment or from temporary detentionduring the pendency of his case or while being transferred from one confinement to another. In the casea t b a r , t h e members of the crew of the two vessels were c a u g h t in flagrante illegally fishing with dynamite and without the requisite license. Thus their apprehension without a warrant of arrest while committing a crime is lawful. Consequently, the seizure of the vessel, its equipment and dynamites therein was equally valid as an incident to a lawful arrest.

When accused failed to comply, the officer required him to bring out whatever it was that was bulging on his waist. The bulging object turned out to be a pouch bag and when accused opened the same bag, as ordered, the officer noticed four (4) suspicious-looking objects wrapped in brown packing tape, prompting the officer to open one of the wrapped objects. The wrapped objects turned out to contain hashish, a derivative of marijuana. Thereafter, accused was invited outside the bus for questioning. But before he alighted from the bus, accused stopped to get two (2) travelling bags from the luggage carrier. Upon stepping out of the bus, the officers got the bags and opened them. A teddy bear was found in each bag. Feeling the teddy bears, the officer noticed that there were bulges inside the same which did not feel like foam stuffing. It was only after the officers had opened the bags that accused finally presented his passport. Accused was then brought to the headquarters of the NARCOM at CampDangwa, La Trinidad, Benguet for further investigation. At the investigation room, the officers opened the teddy bears and they were found to also contain hashish. Representative samples were taken from the hashish found among the personal effects of accused and the same were brought to the PC Crime Laboratory for chemical analysis. In the chemistry report, it was established that the objects examined were hashish. a prohibited drug which is a derivative of marijuana. Thus, an information was filed against accused for violation of the Dangerous Drugs Act. ACCUSEDS DEFENSE During the arraignment, accused entered a plea of "not guilty." For his defense, he raised the issue of illegal search of his personal effects. He also claimed that the hashish was planted by the NARCOM officers in his pouch bag and that the two (2) travelling bags were not owned by him, but were merely entrusted to him by an Australian couple whom he met in Sagada. He further claimed that the Australian couple intended to take the same bus with him but because there were no more seats available in said bus, they decided to take the next ride and asked accused to take charge of the bags, and that they would meet each other at the Dangwa Station. The trial court found the guilt of the accused Mikael Malmstedt established beyond reasonable doubt. Seeking the reversal of the decision of the trial court finding him guilty of the crime charged, accused argues that the search of his personal effects was illegal because it was made without a search warrant and, therefore, the prohibited drugs which were discovered during the illegal search are not admissible as evidence against him. Issue: Whether or Not the contention of the accused is valid, and therefore the RTC ruling be reversed. Held: The Constitution guarantees the right of the people to be secure in their persons, houses, papers and effects against unreasonable searches and seizures. However, where the search is made pursuant to a lawful arrest, there is no need to obtain a search warrant. A lawful arrest without a warrant may be made by a peace officer or a private person under the following circumstances. Sec. 5 Arrest without warrant; when lawful. A peace officer or a private person may, without a warrant, arrest a person: (a) When, in his presence, the person to be arrested has committed is actually committing, or is attempting to commit an offense; (b) When an offense has in fact just been committed, and he has personal knowledge of facts indicating that the person to be arrested has committed it; and

when there is probable cause to effect warrantless search

CASES: Pp v. Malmstedt, 198 SCRA 401 (read the dissent of Narvasa and Cruz, JJ.) Facts: In an information filed against the accused- appellant Mikael Malmstead was charged before the RTC of La Trinidad, Benguet, for violation of Section 4, Art. II of Republic Act 6425, as amended, otherwise known as the Dangerous Drugs Act of 1972, as amended. Accused Mikael Malmstedt, a Swedish national, entered the Philippines for the third time in December 1988 as a tourist. He had visited the country sometime in 1982 and 1985. In the evening of 7 May 1989, accused left for Baguio City. Upon his arrival thereat in the morning of the following day, he took a bus to Sagada and stayed in that place for two (2) days. Then in the 7 in the morning of May 11, 1989, the accused went to Nangonogan bus stop in Sagada.

At about 8: 00 o'clock in the morning of that same day (11 May 1989), Captain Alen Vasco, the Commanding Officer of the First Regional Command (NARCOM) stationed at Camp Dangwa, ordered his men to set up a temporary checkpoint at Kilometer 14, Acop, Tublay, MountainProvince, for the purpose of checking all vehicles coming from the Cordillera Region. The order to establish a checkpoint in the said area was prompted by persistent reports that vehicles coming from Sagada were transporting marijuana and other prohibited drugs. Moreover, information was received by the Commanding Officer of NARCOM, that same morning that a Caucasian coming from Sagada had in his possession prohibited drugs. The group composed of seven (7) NARCOM officers, in coordination with Tublay Police Station, set up a checkpoint at the designated area at about 10:00 o'clock in the morning and inspected all vehicles coming from the Cordillera Region. The two (2) NARCOM officers started their inspection from the front going towards the rear of the bus. Accused who was the sole foreigner riding the bus was seated at the rear thereof. During the inspection, CIC Galutan noticed a bulge on accused's waist. Suspecting the bulge on accused's waist to be a gun, the officer asked for accused's passport and other identification papers.

(c) When the person to be arrested is a prisoner who has escaped from a penal establishment or place where he is serving final judgment or temporarily confined while his case is pending, or has escaped while being transferred from one confinement to another. Accused was searched and arrested while transporting prohibited drugs (hashish). A crime was actually being committed by the accused and he was caught in flagrante delicto. Thus, the search made upon his personal effects falls squarely under paragraph (1) of the foregoing provisions of law, which allow a warrantless search incident to a lawful arrest. While it is true that the NARCOM officers were not armed with a search warrant when the search was made over the personal effects of accused, however, under the circumstances of the case, there was sufficient probable cause for said officers to believe that accused was then and there committing a crime. Probable cause has been defined as such facts and circumstances which could lead a reasonable, discreet and prudent man to believe that an offense has been committed, and that the objects sought in connection with the offense are in the place sought to be searched. Warrantless search of the personal effects of an accused has been declared by this Court as valid, because of existence of probable cause, where the smell of marijuana emanated from a plastic bag 10 owned by the accused, or where the accused was acting 11 suspiciously, and attempted to flee. The appealed judgment of conviction by the trial court is hereby affirmed. Costs against the accused-appellant.

(c) When the person to be arrested is a prisoner who has escaped from a penal establishment or place where he is serving final judgment or temporarily confined while his case is pending, or has escaped while being transferred from one confinement to another. In cases falling under paragraphs (a) and (b) hereof, the person arrested without a warrant shall be forthwith delivered to the nearest police station or jail, and he shall be proceeded against in accordance with Rule 112, Section 7. In any of these instances of a lawful arrest, the person arrested "may be searched for dangerous weapons or anything which may be used as proof of the commission of an offense, without a search 6 warrant." And it has been held that the search may extend to the area "within his immediate control," i.e., the area from which said person arrested might gain possession of a weapon or destructible 7 evidence. Apart from "search incidental to an arrest," a warrantless search has also been held to be proper in cases of "search of a moving 8 9 vehicle, and "seizure of evidence in plain view." This was the pronouncement in Manipon, Jr. v. Sandiganbayan, 143 SCRA 267, 10 276, which drew attention to Moreno v. Ago Chi; Alvero 11 12 v. Dizon, Papa v. Mago, and an American precedent, Harris v. 13 U.S. If, on the other, a person is searched without a warrant, or under circumstances other than those justifying an arrest without warrant in accordance with law, supra, merely on suspicion that he is engaged in some felonious enterprise, and in order to discover if he has indeed committed a crime, it is not only the arrest which is illegal but also, the search on the occasion thereof, as being "the 14 fruit of the poisonous tree. In that event, any evidence taken, even if confirmatory of the initial suspicion, is inadmissible "for any 15 purpose in any proceeding." But the right against an unreasonable search and seizure may be waived by the person arrested, provided 16 he knew of such right and knowingly decided not to invoke it. There is unanimity among the members of the Court upon the continuing validity of these established principles. However, the Court is divided as regards the ultimate conclusions which may properly be derived from the proven facts and consequently, the manner in which the principles just cited should apply thereto. The proofs of the prosecution and those of the defense are diametrically at odds. What is certain, however, is that the soldiers had no warrant of arrest when they conducted a search of Malmstedt's person and the things in his possession at the time. Indeed, the Court a quo acknowledged that the soldiers could "not be expected to be armed with a warrant or arrest nor a search warrant everytime they establish a temporary checkpoint . . . (and) no judge would issue them one considering that searching questions have to be asked before a warrant could be issued." Equally plain is that prior to the search, a warrantless arrest of Malmstedt could not validly have been in accordance with the norms of the law. For Malmstedt had not committed, nor was he actually committing or attempting to commit a crime, in the soldiers' presence, nor did said soldiers have personal and competent knowledge that Malmstedt had in fact just committed a crime. All they had was a suspicion that Malmstedt might have some prohibited drug on him or in his bags; all they had was, in the words of the Trial Court, "the hope of intercepting any dangerous drug being transported," or, as the Office of the Solicitor General asserts, "information that most of the buses coming . . . (from the Cordillera) were transporting marijuana and other prohibited drugs." This case, is remarkably similar to Peo. v. Aminnudin, decided on July 17 6, 1988 also by the First Division. There, Aminnudin was arrested without a warrant by PC officers as he was disembarking from an inter-island vessel. The officers were waiting for him because he was, according to an informer's report, then transporting marijuana. The search of Aminnudin's bag confirmed the informer's report; the bag indeed contained marijuana. The Court nevertheless held that

NARVASA, J., concurring and dissenting: The ancient tradition that a man's home is his castle, safe from intrusion even by the king, has not only found its niche in all our charters, from 1935 to the present; it has also received unvarying 1 recognition and acceptance in our case law. The present 2 Constitution declares that The right of the people to be secure in their persons, houses, papers, and effects against unreasonable searches and seizures of whatever nature and for any purpose, shall be inviolable, and no search warrant or warrant of arrest shall issue except upon probable cause to be determined personally by the judge after examination under oath or affirmation of the complainant and the witnesses he may produce, and particularly describing the place to be searched, and the persons or things to be seized. It further ordains that any evidence obtained in violation of said right, among others, "shall be inadmissible for any purpose in any 3 proceeding." The rule is that no person may be subjected by the police or other government authority to a search of his body, or his personal effects or belongings, or his residence except by virtue of a search warrant 4 or on the occasion of a legitimate arrest. An arrest is legitimate, of course, if effected by virtue of a warrant of arrest. Even without a warrant, an arrest may also be lawfully made by a peace officer or a 5 private person: (a) when, in his presence, the person to be arrested has committed is actually committing, or is attempting to commit an offense; (b) When an offense has in fact just been committed, and he has personal knowledge of facts indicating that the person to be arrested has committed it; and

since the PC officers had failed to procure a search warrant although they had sufficient time (two days) to do so and therefore, the case presented no such urgency as to justify a warrantless search, the search of Aminnudin's person and bag, the seizure of the marijuana and his subsequent arrest were illegal; and the marijuana was inadmissible in evidence in the criminal action subsequently instituted against Aminnudin for violating the Dangerous Drugs Act. There are, on the other hand, other cases adjudicated by this Court in which apparently different conclusions were reached. It is needful to devote a few words to them so that the relevant constitutional and legal propositions are not misunderstood. In People v. Claudio (decision promulgated on April 15, 1988), the accused boarded a "Victory Liner" passenger bus going to Olongapo from Baguio City. She placed the plastic bag she was carrying at the back of the seat then occupied by Obia, an INP member "on Detached Service with the Anti-Narcotics Unit." This avowedly aroused Obia's suspicion, and at the first opportunity, and without Claudio's knowledge, he surreptitiously looked into the plastic bag and noted that it contained camote tops as well as a package, and that there emanated from the package the smell of marijuana with which he had become familiar on account of his work. So when the bus stopped at Sta. Rita, and Claudio alighted, Obia accosted her, showed her his ID, identified himself as a policeman, and announced his intention to search her bag which he said contained marijuana because of the distinctive odor detected by him. Ignoring her plea "Please go with me, let us settle this at home" he brought her to the police headquarters., where examination of the package in Claudio's bag confirmed his suspicion that it indeed contained marijuana. The Court held the warrantless arrest under the circumstances to be lawful, the search justified, and the evidence thus discovered admissible in evidence against the accused. In People v. Tangliben (decision promulgated on April 6, 19 1990), two police officers and a barangay tanod were conducting a "surveillance mission" at the Victory Liner Terminal at San Nicolas, San Fernando, Pampanga, "aimed not only against persons who may commit misdemeanors . . . (there) but also on persons who may be engaging in the traffic of dangerous drugs based on information supplied by informers; . . . they noticed a person carrying a red travelling bag . . who was acting suspiciously;" they asked him to open the bag; the person did so only after they identified themselves as peace officers; found in the bag were marijuana leaves wrapped in plastic weighing one kilogram, more or less; the person was then taken to the police headquarters at San Fernando, Pampanga, where he was investigated; and an information was thereafter filed against that person, Tangliben, charging him with a violation of the Dangerous Drugs Act of 1972 (RA 6425), as amended. Upon these facts it was ruled, citing Claudio, supra, that there was a valid warrantless arrest and a proper warrantless search incident thereto. The facts in Tangliben were pronounced to be different from those in People v. Aminnudin, supra. "In contrast" toAminnudin where the Court perceived no urgency as to preclude the application for and obtention of a search warrant, it was declared that the Tangliben case . . . presented urgency. . . (The evidence revealed) that there was an informer who pointed to the accused-appellant as carrying marijuana . . . Faced with such on-the-spot information, the police officers had to act quickly. There was not enough time to secure a search warrant . . . To require search warrants during on-the-spot apprehensions of drug pushers, illegal possessors of firearms, jueteng collectors, smugglers of contraband goods, robber, etc. would make it extremely difficult, if not impossible to contain the crimes with which these persons are associated. In Tangliben, therefore, there was in the Court's view sufficient evidence on hand to enable the PC officers to secure a search warrant, had there been time. But because
18

there was actually no time to get the warrant, and there were "on-the-spot" indications that Tangliben was then actually committing a crime, the search of his person and his effects was considered valid. Two other decisions presented substantially similar circumstance instances: Posadas v. C.A., et al., decided on August 2, 20 1990, and People v. Moises Maspil, Jr., et al., decided on August 21 20, 1990. In the first case, Posadas was seen to be acting suspiciously by two members of the INP, Davao Metrodiscom, and when he was accosted by the two, who identified themselves as police officers, he suddenly fled. He was pursued, overtaken and, notwithstanding his resistance, placed in custody. The buri bag Posadas was then carrying was found to contain a revolver, for which he could produce no license or authority to possess, four rounds of live ammunition, and a tear gas grenade. He was prosecuted for illegal possession of firearms and ammunition and convicted after trial. This Court affirmed Posadas' conviction, holding that there was, in the premises, probable cause for a search without warrant, i.e., the appellant was acting suspiciously and attempted to flee with the buribag he had with him at the time. The Court cited with approval the ruling of the U.S. Federal Supreme Court inJohn 22 W. Terry v. State of Ohio, a 1968 case, which the Solicitor General had invoked to justify the search. In the case of Maspil, et al., a checkpoint was set up by elements of the First Narcotics Regional Unit of the Narcotics Command at Sayangan, Atok, Benguet, to monitor, inspect and scrutinize vehicles on the highway going towards Baguio City. This was done because of a confidential report by informers that Maspil and another person, Bagking, would be transporting a large quantity of marijuana to Baguio City. In fact, the informers were with the policemen manning the checkpoint. As expected, at about 2 o'clock in the early morning of November 1, 1986, a jeepney approached the checkpoint, driven by Maspil, with Bagking as passenger. The officers stopped the vehicle and saw that on it were loaded 2 plastic sacks, a jute sack, and 3 big round tin cans. When opened, the sacks and cans were seen to contain what appeared to be marijuana leaves. The policemen thereupon placed Maspil and Bagking under arrest, and confiscated the leaves which, upon scientific examination, were verified to be marijuana leaves. The Court upheld the validity of the search thus conducted, as being incidental to a lawful warrantless 23 arrest, and declared that, as in Tangliben, supra, Maspil and Bagking had been caught in flagrante delicto transporting prohibited drugs at the time of their arrest. Again, the Court took occasion to 24 distinguish the case from Aminnudin in which, as aforestated, it appeared that the police officers were aware of Aminnudin's identity, his projected criminal enterprise and the vessel on which he would be arriving, and, equally as importantly, had sufficient time and opportunity to obtain a search warrant. In the case of Maspil and Bagking, the Court found that the officers concerned had no exact description of the vehicle the former would be using to transport marijuana, and no inkling of the definite time of the suspects' arrival, and pointed out that a jeepney on the road is not the same as a passenger boat on the high seas whose route and time of arrival are more or less certain, and which ordinarily cannot deviate from or otherwise alter its course, or select another 25 destination. The most recent decision treating of warrantless search and seizure appears to be People v. Lo Ho Wing; et al., G.R. No. 88017, decided on January 21, 1991 (per Gancayco, J.). In that case, an undercover or "deep penetration" agent, Tia, managed somehow to gain acceptance into a group of suspected drug smugglers, which included Peter Lo and Lim Ching Huat. Tia accompanied Peter Lo to Guangzhou, China, where he saw him and other person empty the contents of six (6) tins of tea and replace them with white powder. On their return to Manila with the cans of substituted "tea," they were met at the airport by Lim. As they were leaving the airport in separate vehicles, they were intercepted by officers and operatives of the Narcotics Command (NARCOM), who had earlier been tipped off by Tia, and placed under arrest. As search of the luggage brought in by Tia and Peter Lo, loaded on the group's vehicles, quickly disclosed the six (6) tin cans containing fifty-six (56) bags of white crystalline powder which, upon analysis, was identified as

metamphetamine. Tia, Lo and Lim were indicted for violation of the Dangerous Drugs Act of 1972. Tia was discharged as state witness. Lo and Lim were subsequently convicted and sentenced to life imprisonment. One of the questions raised by them in this Court on appeal was whether the warrantless search of their vehicles and personal effects was legal. The Court, citing Manipon, 26 Jr. v.Sandiganbayan, 143 SCRA 267 (1986), held legal the search of the appellants' moving vehicles and the seizure therefrom of the dangerous drug, considering that there was intelligence information, including clandestine reports by a planted spy actually participating in the activity, that the appellants were bringing prohibited drugs into the country; that the requirement of obtaining a search warrant "borders on the impossible in the case of smuggling effected by the use of a moving vehicle that can transport contraband from one place to another with impunity," and "it is not practicable to secure a warrant because the vehicle can be quickly moved out of the 27 locality or jurisdiction in which the warrant must be sought. In all five cases, Claudio, Tangliben, Posadas, Maspil, and Lo Ho Wing, facts existed which were found by the Court as justifying warantless arrests. In Claudio, the arresting officer had secretly ascertained that the woman he was arresting was in fact in possession of marijuana; he had personally seen that her bag contained not only vegetables but also a package emitting the odor of marijuana. In Tangliben, the person arrested and searched was acting suspiciously, and had been positively pointed to as carrying marijuana. And in both cases, the accused were about to board passenger buses, making it urgent for the police officers concerned to take quick and decisive action. In Posadas, the person arrested and searched was acting suspiciously, too, and when accosted had attempted to flee from the police officers. And in Maspil and Lo Ho Wing, there was definite information of the precise identity of the persons engaged in transporting prohibited drugs at a particular time and place. Now, as regards the precise issue at hand, whether or not the facts in the case at bar make out a legitimate instance of a warrantless search and seizure, there is, as earlier pointed out, a regrettable divergence of views among the members of the Court. Contrary to the conclusion reached by the majority, I believe that the appellant should be absolved on reasonable doubt. There was in this case no confidential report from, or positive identification by an informer; no attempt to flee; no bag or package emitting tell-tale odors; no other reasonably persuasive indications that Malmstedt was at the time in process of perpetrating the offense for which he was subsequently prosecuted. Hence, when the soldiers searched Malmstedt's pouch and the bags in his possession, they were simply "fishing" for evidence. It matters not that the search disclosed that the bags contained prohibited substances, confirming their initial information and suspicion. The search was not made by virtue of a warrant or as an incident of a lawful warrantless arrest, i.e., under circumstances sufficient to engender a reasonable belief that some crime was being or about to be committed, or adjust been committed. There was no intelligent and intentional waiver of the right against unreasonable searches and seizure. The search was therefore illegal, since the law requires that there first be a lawful arrest of an individual before a search of his body and his belongings may licitly be made. The process cannot be reversed, i.e., a search be first undertaken, and then an arrest effected, on the strength of the evidence yielded by the search. An arrest made in that case would be unlawful, and the search undertaken as an incident of such an unlawful arrest, also unlawful. The fact that when investigated at the headquarters of the Narcotic Command at Camp Dangwa, La Trinidad, Malmstedt had, it is said, willingly admitted that there were was hashish inside the "teddy bears" in the luggage found in his possession an admission subsequently confirmed by laboratory examination does not help the cause of the prosecution one bit. Nothing in the record even remotely suggests that Malmstedt was accorded the rights guaranteed by the Constitution to all persons under custodial 28 investigation. He was not informed, prior to being interrogated, that he had the "right to remain silent and to have competent and independent counsel preferably of his own choice," and that if he could not afford the services of counsel, he would be provided with one; not does it appear at all that he waived those rights "in writing

and in the presence of counsel." The soldiers and the police officers simply went ahead with the investigation of Malmstedt, without counsel. The admissions elicited from Malmstedt under these circumstances, as the Constitution clearly states, are "inadmissible in 29 evidence against him. The prohibited drugs supposedly discovered in Malmstedt's bags, having been taken in violation of the constitutional right against unreasonable searches and seizures, are inadmissible against him "for any purpose in any proceeding." Also pronounced as incompetent evidence against him are the admissions supposedly made by him without his first being accorded the constitutional rights of persons under custodial investigation. Without such object evidence and admissions, nothing remains of the case against Malmstedt. It may be conceded that, as the Trial Court points out, the evidence presented by Malmstedt in his defense is feeble, unworthy of credence. This is beside the point; for conformably to the familiar axiom, the State must rely on the strength of its evidence and not on the weakness of the defense. The unfortunate fact is that although the existence of the hashish is an objective physical reality that cannot but be conceded, there is in law no evidence to demonstrate with any degree of persuasion, much less beyond reasonable doubt, that Malmstedt was engaged in a criminal activity. This is the paradox created by the disregard of the applicable constitutional safeguards. The tangible benefit is that the hashish in question has been correctly confiscated and thus effectively withdrawn from private use. What is here said should not by any means be taken as a disapproval or a disparagement of the efforts of the police and military authorities to deter and detect offenses, whether they be possession of and traffic in prohibited drugs, or some other. Those efforts obviously merit the support and commendation of the Courts and indeed of every responsible citizen. But those efforts must take account of the basic rights granted by the Constitution and the law to persons who may fall under suspicion of engaging in criminal acts. Disregard of those rights may not be justified by the objective of ferreting out and punishing crime, no matter how eminently desirable attainment of that objective might be. Disregard of those rights, as this Court has earlier stressed, may result in the escape of the guilty, and all because the "constable has blundered," rendering 30 the evidence inadmissible even if truthful or otherwise credible. I therefore vote to reverse the Trial Court's judgment of October 12, 1989 and to acquit the appellant on reasonable doubt.

CRUZ, J., dissenting: I join Mr. Justice Andres R. Narvasa in his dissent, which I believe represents the correct application to the facts of this case of the provisions of the Bill of Rights and the Rules of Court on searches and seizures. It is consistent with my ponencia in People v. Aminnudin, 163 SCRA 402, and also with Alih v. Castro, 151 SCRA 279, the latter being a unanimous decision of the Court en banc, and my dissents in Umil v. Ramos (on warrantless arrests, 187 SCRA 311, Valmonte v. De Villa (on checkpoints), 178, SCRA 211, 185 SCRA 665, and Guazon v. De Villa (on "zonas"), 181 SCRA 623. I write this separate opinion merely to remark on an observation made during the deliberation on this case that some members of the Court seem to be coddling criminals instead of extending its protection to society, which deserves our higher concern. The inference is that because of our wrong priorities, criminals are being imprudently let free, to violate our laws again; and it is all our fault. Believing myself to be among those alluded to, I will say without apology that I do not consider a person a criminal, until he is convicted by final judgment after a fair trial by a competent and impartial court. Until then, the Constitution bids us to presume him innocent. He may seem boorish or speak crudely or sport tattoos or dress weirdly or otherwise fall short of our own standards of

propriety and decorum. None of these makes him a criminal although he may look like a criminal. PADILLA, J.: It is so easy to condemn a person on the basis of his appearance but it is also so wrong. On the question before us, it seems to be the inclination of some judges to wink at an illegal search and seizure as long as the suspect has been actually found in possession of a prohibited article That fact will retroactively validate the violation of the Bill of Rights for after all, as they would rationalize, the suspect is a criminal. What matters to them is the fact of illegal possession, not the fact of illegal search and seizure. This kind of thinking takes us back to the intolerant days of Moncado v. People's Court, 80 Phil. 1, which was discredited in Stonehill v. Diokno, 20 SCRA 383, even before it was definitely rejected by an express provision in the 1973 Constitution. That provision, which has been retained in the present Constitution, again explicitly declares that any evidence illegally obtained "shall be inadmissible for any purpose in any proceeding." The fruit of the poisonous tree should not be allowed to poison our system of criminal justice. In the case at bar, the search was made at a checkpoint established for the preposterous reason that the route was being used by marijuana dealers and on an individual who had something bulging at his waist that excited the soldier's suspicion. Was that probable cause? The ponencia notes that the military had advance information that a Caucasian was coming from the Sagada with prohibited drugs in his possession. This is what the military says now, after the fact, to justify the warrantless search. It is so easy to make such a claim, and I am surprised that the majority should readily accept it. The conclusion that there was probable cause may have been influenced by the subsequent discovery that the accused was carrying a prohibited drug. This is supposed to justify the soldier's suspicion. In other words, it was the fact of illegal possession that retroactively established the probable cause that validated the illegal search and seizure. It was the fruit of the poisonous tree that washed clean the tree itself. In Olmstead v. U.S., 277 U.S. 438, Justice Holmes said sixty-four years ago: . . . It is desirable that criminals should be detected, and to that end that all available evidence should be used. It is also desirable that the government should not itself foster and pay for other crimes, when they are the means by which the evidence is to be obtained. If it pays its officers for having got evidence by crime, I do not see why it may not as well pay them for getting it in the same way, and I can attach no importance to protestations of disapproval if it knowingly accepts and pays and announces that in the future it will pay for the fruits. We have to choose, and for my part I think it a less evil that some criminals should escape than that the government should play an ignoble part. If by deterring the government from playing "an ignoble part," I am "coddling criminals," I welcome the accusation and take pride in it. I would rather err in favor of the accused who is impaled with outlawed evidence than exalt order at the price of liberty. I. THE FACTS

DECISION

On 20 January 1987, the National Capital Region District Command (NCRDC) was activated pursuant to Letter of Instruction 02/87 of the Philippine General Headquarters, AFP, with the mission of conducting security operations within its area of responsibility and peripheral areas, for the purpose of establishing an effective territorial defense, maintaining peace and order, and providing an atmosphere conducive to the social, economic and political development of the National Capital Region. As part of its duty to maintain peace and order, the NCRDC installed checkpoints in various parts of Valenzuela, Metro Manila. Petitioners Atty. Ricardo Valmonte, who is a resident of Valenzuela, Metro Manila, and the Union of Lawyers and Advocates For Peoples Rights (ULAP) sought the declaration of checkpoints in Valenzuela, Metro Manila and elsewhere as unconstitutional. In the alternative, they prayed that respondents Renato De Villa and the National Capital Region District Command (NCRDC) be directed to formulate guidelines in the implementation of checkpoints for the protection of the people. Petitioners contended that the checkpoints gave the respondents blanket authority to make searches and seizures without search warrant or court order in violation of the Constitution. II. THE ISSUE

Do the military and police checkpoints violate the right of the people against unreasonable search and seizures? III. THE RULING [The Court, voting 13-2, DISMISSED the petition.] NO, military and police checkpoints DO NOT violate the right of the people against unreasonable search and seizures. xxx. Not all searches and seizures are prohibited. Those which are reasonable are not forbidden. A reasonable search is not to be determined by any fixed formula but is to be resolved according to the facts of each case. Where, for example, the officer merely draws aside the curtain of a vacant vehicle which is parked on the public fair grounds, or simply looks into a vehicle, or flashes a light therein, these do not constitute unreasonable search. The setting up of the questioned checkpoints in Valenzuela (and probably in other areas) may be considered as a security measure to enable the NCRDC to pursue its mission of establishing effective territorial defense and maintaining peace and order for the benefit of the public. Checkpoints may also be regarded as measures to thwart plots to destabilize the government, in the interest of public security. In this connection, the Court may take judicial notice of the shift to urban centers and their suburbs of the insurgency movement, so clearly reflected in the increased killings in cities of police and military men by NPA sparrow units, not to mention the abundance of unlicensed firearms and the alarming rise in lawlessness and violence in such urban centers, not all of which are reported in media, most likely brought about by deteriorating economic conditions which all sum up to what one can rightly consider, at the very least, as abnormal times. Between the inherent right of the state to protect its existence and promote public welfare and an individual's right against a warrantless search which is however reasonably conducted, the former should prevail.

Pp v. Aruta, supra validity of check points CASE: Valmonte v. de Villa, 173 SCRA 211 (read the dissent of Cruz and Sarmiento, JJ.)

True, the manning of checkpoints by the military is susceptible of abuse by the men in uniform, in the same manner that all governmental power is susceptible of abuse. But, at the cost of occasional inconvenience, discomfort and even irritation to the citizen, the checkpoints during these abnormal times, when conducted within reasonable limits, are part of the price we pay for an orderly society and a peaceful community. when to question validity of the warrantless arrest CASE: Pp v. Salvatierra, 276 SCRA 55

connection therewith constitute direct assaults against the state and are in the nature of continuing crimes. CRUZ, J., Separate Opinion: I reiterate my concurrence with the ponencia insofar as it dismissed the petitions of those who were arrested inflagrante, or subsequently posted bail or chose to remain in the custody of the military, or voluntarily permitted the search of the house without warrant. I do not think that under the applicable circumstances the petitioners can validly complain that they are being unlawfully detained. But I must again express may dissent to the continued observance of Garcia-Padilla vs. Enrile, 121 SCRA 472, to justify the warrantless arrest and detention of the other petitioners on the ground that they were apprehended for the continuing offenses of rebellion and other allied crimes. We find in the said decision this partltularly disturbing observation, which was quoted with approval in the originalponencia: The arrest of persons involved in the rebellion, whether as its fighting armed elements, or for committing non-violent acts but in furtherance of the rebellion, is more an act of capturing them in the course of an armed conflict, to quell the rebellion, than for the purpose of immediately prosecuting them in court for a statutory offense. The arrest, therefore, need not follow the usual procedure in the prosecution of offenses which requires the determination by a judge of the existence of probable cause before the issuance of arrest and the granting of bail of the offense is bailable. Obviously, the absence of a judicial warrant is no legal impediment to arresting or capturing persons committing overt acts of violence against govenment forces, or any other milder acts but equally in pursuance of the rebellious movement. (Emphasis supplied.) The treatment suggested envisions an actual state of war and is justified only when a recognition of beuigerency is accorded by the legitimate government to the rebels, resulting in the application of the laws of war in the regulation of their relations. The rebels are then considered alien enemies-to be treated as prisoners of war when captured-and cannot invoke the municipal law of the legitimate government they have disowned. It is in such a situation that the processes of the local courts are not observed and the rebels cannot demand the protection of the Bill of Rights that they are deemed to have renounced by their defiance of the government. But as long as that recognition has not yet been extended, the legitimate govenment must treat the rebels as its citizens, subject to its municipal law and entitled to all the rights provided thereunder, including and especially those guaranteed by the Constitution. Principal among these in our country are whose embodied in the Bill of Rights, particularly those guaranteeing due process, prohibiting unreasonable searches and seizures, allowing bail, and presuming the innocence of the accused. The legitimate government cannot excuse the suppression of these rights by the "exigencies" of an armed conflict that at this time remains an intemal matter governed exclusively by the laws of the Republic of the Philippines. Treatment of the rebels as if they were foreign invaders or combatants is not justified in the present situation as our government continues to prosecute them as violators of our own laws. Under the doctrine announced in Garcia-Padilla, however, all persons suspected as rebels are by such suspicion alone made subject to summary arrest no different from the unceremonious capture of an enemy soldier in the course of a battle. The decision itself says that the arrest "need not follow the usual procedure in the prosecution of offenses" and "the absence of a judicial warrant is no impediment" as long as the person arrested is suspected by the authorities of the "continuing offense" of subversion or rebellion or other related crimes. International law is thus substituted for

While Charlie Fernandez was walking towards Quiapo, appellant and the 2 accused lunged a pointed instrument at Charlie, hitting the latter at the left breast. This resulted to his death. On Nov. 15, 1990 the police received a complaint that appellant was creating a commotion. He was thereafter taken in custody. Appellant put up the defense of alibi alleging that he was having merienda with his wife and children when the incident occurred. He also alleged that he had an altercation with a woman on that day who caused his arrest for the crime of malicious mischief, wherein he was detained after. Then, when police arrived, they brought him to the Homicide Section where he was investigated for the stabbing of Fernandez. Appellant claimed that the arrest was made almost 3 months after the commission of the crime and no warrant had been obtained during the 3-month intervening period between the commission of the crime and his apprehension, thus making the arrest illegal. Issue: W/N the arrest is violative of his constitutional rights? Held: No. Appellant is estopped from questioning the legality of his arrest considering that he never raised it before entering his plea. Any objection involving a warrant of arrest or the procedure in the acquisition of jurisdiction over the person of an accused must be made before he enters his plea, otherwise, the objection is deemed waived. other important rulings CASES: Umil v. Ramos, 187 SCRA 311 (read the dissents of Cruz and Sarmiento, JJ.) Facts: On 1 February 1988, military agents were dispatched to the St. Agnes Hospital, Roosevelt Avenue, Quezon City, to verify a confidential information which was received by their office, about a "sparrow man" (NPA member) who had been admitted to the said hospital with a gunshot wound. That the wounded man in the said hospital was among the five (5) male "sparrows" who murdered two (2) Capcom mobile patrols the day before, or on 31 January 1988 at about 12:00 o'clock noon, before a road hump along Macanining St., Bagong Barrio, Caloocan City. The wounded man's name was listed by the hospital management as "Ronnie Javellon," twenty-two (22) years old of Block 10, Lot 4, South City Homes, Bian, Laguna however it was disclosed later that the true name of the wounded man was Rolando Dural. In view of this verification, Rolando Dural was transferred to the Regional Medical Servicesof the CAPCOM, for security reasons. While confined thereat, he was positively identified by the eyewitnesses as the one who murdered the 2 CAPCOM mobile patrols. Issue: Whether or Not Rolando was lawfully arrested. Held: Rolando Dural was arrested for being a member of the NPA, an outlawed subversive organization. Subversion being a continuing offense, the arrest without warrant is justified as it can be said that he was committing as offense when arrested. The crimes rebellion, subversion, conspiracy or proposal to commit such crimes, and crimes or offenses committed in furtherance therefore in

municipal law in regulating the relations of the Republic with its own citizens in a purely domestic matter. As for the duration of the offenses, the decision contained the following pronouncement which this Court has also adopted as its own: . . . The crimes of insurrection or rebellion, subversion, conspiracy or proposal to commit such crimes, and other crimes and offenses committed in the furtherance on the occasion thereof, or incident thereto, or in connection therewith under Presidential Proclamation No. 2045, are all in the nature ofcontinuing offenses which set them apart from the common offenses, aside front their essentially involving a massive conspiracy of nationwide manitude. (Emphasis supplied.) The beginning of the "continuing offense" may be arbitrarily fixed by the authorities, usually by simply placing the suspect "under surveillance," to lay the basis for his eventual apprehension. Once so placed, he may at any time be arrested without warrant on the specious pretext that he is in the process of committing the "continuing offense," no matter that what he may be actuallly doing at the time is a perfectly innocent act. In the case of Dural. the arrest was made while he was engaged in the passive and innocuous act of undergoing medical treatment. The fiction was indulged that he was even then, as he lay supine in his sickbed, engaged in the continuing offense of rebellion against the State. In further justification, the Court says that the arresting officers acted on "confidential information" that he was in the hospital, which information "was found to be true." This is supposed to have validated the determination of the officers that there was "probable cause" that excused the absence of a warrant. My own impression is that probable cause must be established precisely to justify the issuance of a warrant, not todispense with it; moreover, probable cause must be determined by the judge issuing the warrant, not the arresting officer who says it is not necessary. In the case of Espiritu, the arrest was made while he was actually sleeping, and for allegedly seditious remarks made by him the day before. The Court says his case is not covered by the GarciaPadilla doctrine but approves the arrest just the same because the remarks were supposed to continue their effects even to the following day. The offense was considered as having been just committed (to make it come under Rule 113, Section 5, of the Rules of Court) despite the considerable time lapse. It was worse in the case of Nazareno, who was also arrested without warrant, and no less than fourteen days after the killing. In sustaining this act, the Court says that it was only on the day of his arrest that he was identified as one of the probable killers, thus suggesting that the validity of a warrantless arrest is reckoned not from the time of the commission of an offense but from the time of the Identification of the suspect. Section 5 of Rule 113 says that a peace officer may arrest a person without a warrant if the latter "has committed, is actually committing, or is attempting to commit an offense" or when an offense "has in fact just been committed." The requirement of immediacy is obvious from the word "just," which, according to Webster, means "a very short time ago." The arrest must be made almost immediately or soon after these acts, not at any time after the suspicion of the arresting officer begins, no matter how long ago the offense was committed. I am also uneasy over the following observations in the present resolution which I hope will not be the start of another dangerous doctrine: The Court, it is true, took into account the admissions of the arrested persons of their

membership in the CPP/NPA, as well as their ownership of the unlicensed firearms, ammunitions and documents in their possession. But again, these admissions, as revealed by the records, strengthen the Court's perception that truly the grounds upon wmch the arresting officers based their arrests without warrant, are supported by probable cause, i.e., that the persons arrested were probably guilty of the commission of certain offenses, in compliance with Section 5, Rule 113 of the Rules of Court. I can only repeat my own misgivings when I dissented in the recent case of People vs. Malmstedt, G.R. No. 91107, June 19, 1991, where I noted: "The conclusion that there was probable cause may have been influenced by the subsequent discovery that the accused was carrying a prohibited drug. This is supposed to justify the soldier's suspicion. In other words, it was the fact of illegal possession that retroactively established the probable cause that validated the illegal search and seizure. It was the fruit of the poisonous tree that washed clean the tree itself." I submit that the affirmation by this Court of the Garcia-Padilla decision to justify the illegal arrests made in the cases before us is a step back to that shameful past when individual rights were wantonly and systematically violated by the Marcos dictatorship. It seems some of us have short memories of that repressive regime, but I for one am not one to forget so soon. As the ultimate defender of the Constitution, this Court should not gloss over the abuses of those who, out of mistaken zeal, would violate individual liberty in the dubious name of national security. Whatever their ideology and even if it be hostile to ours, the petitioners are entitled to the protection of the Bill of Rights, no more and no less than any other person in this country. That is what democracy is all about. SARMIENTO, J.: dissenting: I reiterate my dissent. I submit that in spite of its "clarificatory" resolution, 1 the majority has not shown why the arrests in question should after all be sustained. According to the majority, Rolando Dural (G.R. No. 815667) was validly arrested without a warrant and that his arrest was sufficient compliance with the provisions of Section 5, paragraph (b), Rule 113, of the Rules of Court. According to the majority, he, Dural, was after all committing an offense (subversion being supposedly a continuing offense) and that the military did have personal knowledge that he had committed it. "Personal knowledge," according to the majority, is supposedly no more than "actual belief or reasonable grounds . . . of suspicion," and suspicion is supposedly reasonable: . . . when, in the absence of actual belief of the arresting officers, the suspicion that the person to be arrested is probably guilty of committing the offense, is based on actual facts, i.e., supported by circumstances sufficiently strong in themselves to create the probable cause of guilty of the person to be arrested. A reasonable suspicion therefore must be founded on probable cause, coupled with good faith on the part of the peace officers making the arrest. 2 As I said, I dissent. First, and as I held, subversion, as an offense punished by Executive Order No. 167, as amended by Executive Order No. 276, in relation to Republic Act No. 1700, 3 is made up of "overt acts." 4 In People vs. Ferrer 5 this Court defined "overt acts" as follows: . . . Indeed, were the Anti-Subversion Act a bill of attainder, it would be totally unnecessary to charge Communists in court, as the law alone, without more would suffice to secure their punishment. But the undeniable fact is that their guilt still has to be judicially established. The Government has yet to prove at the trial that the

accused joined the Party knowingly, willfully and by overt acts, and that they joined the Party, knowing its subversive character and with specific intent to further its basic objective,i.e., to overthrow the existing government by force, deceit, and other illegal means and place the country under the control and domination of a foreign power. As Ferrer held, that above "overt acts" constitute the essence of "subversion," and as Ferrer has taken pains to explain, the law requires more than mere membership in a subversive organization to make the accused liable. I respectfully submit that for purposes of arrest without a warrant, that above "overt acts" should be visible to the eyes of the police officers (if that is possible), otherwise the accused can not be said to be committing any offense within the contemplation of the Rules of Court, to justify police action, and otherwise, we would have made "subversion" to mean mere "membership" when, as Ferrer tells us, subversion means more that mere membership. I find strained that majority's interpretation of "personal knowledge," as the majority would interpret it, as no more than "actual belief or reasonable suspicion," that is, "suspicion . . . based on actual facts . . . [and] founded on probable cause, coupled with good faith . . . " 6 I submit that personal knowledge means exactly what it says that the peace officer is aware that the accused has committed an offense, in this case, membership in a subversive organization with intent to further the objectives thereof. It is to be noted that prior to their amendment, the Rules (then Section 6) spoke of simple "reasonable ground" which would have arguably encompassed "actual belief or suspicion . . . coupled with good faith" referred to by the majority. Section 5(b) as amended, however, speaks of "personal knowledge"; I respectfully submit that to give to "personal knowledge" the same meaning as "reasonable ground" is to make the amendment as useless exercise. What, furthermore, we have here was a mere "confidential information" that a "sparrow man" had been wounded and was recuperating in the hospital, and that that person was Rolando Dural. Clearly, what we have is second-hand, indeed, hearsay, information, and needless to say, not personal knowledge. I would like to point out that in the case of People vs. Burgos 7 this Court rejected a similar arrest because of lack of personal knowledge, and, as the Court held, "[w]hatever knowledge was possessed by the arresting officers came in its entirety from the information furnished by [another] . . ." 8 I do not see how We can act differently here. I do not find the majority's reliance on the case of United States vs. Santos 9 to be well-taken. Santos involved a prosecution for coercion (against a peace officer for affecting an arrest without a warrant). Santos, however, did in fact affirm the illegality of the arrest but absolved the peace officer on grounds of good faith. Santos did not say that so long as he, the peace officer, was acting in good faith, as the majority here says that the military was acting in good faith, the arrest is valid. Quite to the contrary, Santos suggested that notwithstanding good faith on the part of the police, the arrest is nevertheless subject to question. As far as the information leading to the arrest of Dural is concerned, the majority would quite evidently swallow the version of the military as if in the first place, there truly was an information, and that it was reliable, and that "it was found to be true;" 10 and as if, in the second place, the hospital authorities (the alleged informants) could have legally tipped the military under existing laws. We have, it should be noted, previously rejected such a species of information because of the lack of "compulsion for [the informant] to state truthfully his charges under pain of criminal prosecution." 11 Here, it is worse, because we do not even know who that informant was. The majority is apparently unaware that under Executive Order No. 212, amending Presidential Decree No. 169, hospital establishments are required to report cases of acts of violence to "government health authorities" not to the military.

I am concerned that if the military were truly armed with reliable information and if it did have personal knowledge to believe that Dural had committed an offense, there was no reason for the military to ignore the courts, to which the Constitution after all, gives the authority to issue warrants. As People vs. Burgos held: More important, we find no compelling reason for the haste with which the arresting officers sought to arrest the accused. We fail to see why they failed to first go through the process of obtaining a warrant of arrest, if indeed they had reasonable ground to believe that the accused had truly committed a crime. There is no showing that there was a real apprehension that the accused was on the verge of flight or escape. Likewise, there is no showing that the whereabouts of the accused were unknown. 12 I do not likewise see how the petitioners Amelia Roque, Wilfredo Buenaobra, Domingo Anonuevo, Ramon Caspile, and Vicky Ocaya (G.R. Nos. 84581-82; 83162) could have been lawfully picked up under similar circumstances. As the majority points out, the military had (again) acted on a mere tip-the military had no personal knowledge (as I elaborated what personal knowledge means). Second, I do not think that the majority can say that since Amelia Roque, et al. "were NPA's anyway" (As Roque, et al. allegedly admitted), immediate arrests were "prudent" and necessary. As I said, that Roque, et al. were admitted "NPA's" is (was) the question before the trial court and precisely, the subject of controversy. I think it is imprudent for this Court to pass judgment on the guilt of the petitioners-since after all, and as the majority points out, we are talking simply of the legality of the petitioner's arrests. More important, that Roque, et al. "were NPA's anyway" is evidently, a mere say-so of the military, and evidently, the Court is not bound by bare say-so's. Evidently, we can not approve an arrest simply because the military says it is a valid arrest (the accused being "NPA's anyway") that would be abdication of judicial duty and when, moreover, the very basis of the claim rests on dubious "confidential information." According to the majority, we are speaking of simple arrests; we are not talking of the guilt or innocence of the accused. I certainly hope not, after the majority referred to Rolando Dural as a "sparrow man" and having Amelia Roque, et al. admit to being NPA's." It is to gloss over at any rate, the nature of arrest as a restraining on liberty. It is to me immaterial that the guilt of the accused still has to be established, since meanwhile, the accused are in fact being deprived of liberty. Arrest to me, is something to crow about, even if in the opinion of the majority, it is nothing to crow about (a mere "administrative measure"). I can not, again, accept the validity of the arrests of Deogracia Espiritu or Narciso Nazareno (G.R. Nos. 85727; 86332). Espiritu was supposedly picked up for inciting to sedition, in uttering supposedly, on November 22, 1988, the following: Bukas tuloy and welga natin . . . hanggang sa magkagulo na. 13 Espiritu however was arrested on November 23, 1988, a day laterand in no way is "inciting to sedition" a continuing offense. Obviously, the majority is not saying that it is either, but that: . . . Many persons may differ as to the validity of such perception and regard the language as falling within free speech guaranteed by the Constitution. But, then, Espiritu has not lost the right to insist, during the trial on the merits, that he was just exercising his right to free speech regardless of the charged atmosphere in which it was uttered. But, the authority of the peace officers to make the arrest, without warrant, at the time the words were uttered, or soon thereafter, is still another thing. In the balancing of authority and freedom, which obviously

becomes difficult at times, the Court has, in this case, titled the scale in favor of authority but only for purposes of the arrest (not conviction). Let it be noted that the Court has ordered the bail for Espiritu's release to be reduced from P60,000.00 to P10,000.00. 14 And obviously, the majority is concerned about whether or not Espiritu's speech was after all, protected speech, but apparently, that is also of no moment, since: (1) that is a matter of defense; (2) we are talking of mere arrests, and as far as arrests are concerned, "the Court has, in this case, titled in favor of authority," 15 and (3) we have, anyway, given a reduced bail to the accused. First, that the accused's statement is in the category of free speech is not only plain to my mind, it is a question I do not think the majority can rightly evade in these petitions without shirking the Court's constitutional duty. It is to my mind plain, because it does not contain enough "fighting words" recognized to be seditious. 16 Secondly, it is the very question before the Court whether or not the statement in question constitutes an offense for purposes of a warrantless arrest. It is a perfectly legal question to my mind and I am wondering why we can not answer it. What the majority has not answered, as I indicated, is that inciting to sedition is in no way a continuing offense, and as I said, the majority is not apparently convicted that it is, either. Of course, the majority would anyway force the issue: "But the authority of the peace officers to make the arrest, without warrant, at the time the words were uttered, or soon thereafter, is still another thing." 17 First, Espiritu was picked up the following day, and in no way is "the following day" "soon thereafter". Second, we would have stretched the authority of peace officers to make warrantless arrests for acts done days before. I do not think this is the contemplation of the Rules of Court. As in the case of Burgos in People vs. Burgos, 18 Espiritu was neither "on the verge of flight or escape" 19 and there was no impediment for the military to go through the judicial processes, as there was none in the case of Burgos. In the case of People vs. Aminnudin, 20 this Court held that unless there "was a crime about to be committed or had just been committed," and unless there existed an urgency as where a moving vehicle is involved, instant police action can not be justified. "In the balancing of authority and freedom," states the majority, "the Court has, in this case, titled in favor of authority but only for purposes of the arrest (not conviction)." 21 It is a strange declaration, first, because it is supported by no authority (why the Court should "tilt" on the side of Government), and second, because this Court has leaned, by tradition, on the side of liberty as the custodian of the Bill of Rights even if we were talking of "simple" arrests. I do not understand why this Court should "tilt" . . . the scale in favor of authority . . . in this case," 22 as if to say that normally, this Court would have tilted the scales the other way. I do not understand why these cases are apparently, special cases, and apparently, the majority is not telling us neither. I am wondering why, apart from the fact that these cases involved, incidentally, people who think differently from the rest of us. The majority goes on: Although the killing of Bunye II occurred on 14 December 1988, while Nazareno's arrest without warrant was made only on 28 December 1988, or 14 days later, the arrest falls under Section 5(b) of Rule 113, since it was only on 28 December 1988 that the police authorities came to know that Nazareno was probably one of those guilty in the killing of Bunye II. 23

With all due respect, I do not think that the majority is aware of the serious implications of its pronouncement on individual rights (and statutory construction in general), and I feel I am appropriately concerned because as a member of the Court, I am co-responsible for the acts of my colleagues and I am afraid that I may, rightly or wrongly, be in time made to defend such an indefensible pronouncement. Section 5(b) of Rule 113 is clear and categorical: the offense must have been "just committed" and the authorities must have "personal knowledge." In no way can an offense be said to have been "just committed" fourteen days after it was in fact (allegedly) committed. In no way can the authorities be said to have "personal knowledge" two weeks thereafter; whatever "personal knowledge" they have can not possibly be "personal knowledge" of a crime that had "just been committed;" whatever "personal knowledge" they have is necessarily "personal knowledge" of a crime committed two weeks before. In no way can Nazareno's arrest be said to be an arrest sanctioned by the exceptional provisions of the Rules. I am not saying that the military can not act in all cases, and it is sheer ignorance to suppose that I am saying it, (or worse, that I am "coddling criminals"). I am not saying that a suspected criminal, if he can not be arrested without a warrant, can not be arrested at all but that the military should first procure a warrant from a judge before effecting an arrest. It is not too much to ask of so-called law enforcers. As it is, the majority has enlarged the authority of peace officers to act, when the Rules have purposely limited it by way of an exception, precisely, to the general rule, mandated by the Constitution no less, that arrests may be done only through a judicial warrant. As it is, the majority has in fact given the military the broadest discretion to act, a discretion the law denies even judges 24 today it is fourteen days, tomorrow, one year, and sooner, a decade. I submit that a year, a decade, would not be in fact unreasonable, following the theory of the majority, since the military can claim anytime that it "found out only later," as the majority did not find it unreasonable for the Capital Command to claim that it "came to know that Nazareno was probably one of those guilty in the killing of Bunye II"25and none of us can possibly dispute it. I would like to stress strongly that we are not talking of a simple "administrative measure" alonewe are talking ofarrests, of depriving people of libertyeven if we are not yet talking of whether or not people are guilty. That we are not concerned with guilt or innocence is hardly the point, I respectfully submit, and it will not minimize the significance of the petitioners' predicament. With respect to Wilfredo Buenaobra, I submit that the majority has, as in the cases of Amelia Roque, et al., ignored the fact that Buenaobra's alleged "admission" (actually, an uncounselled confession) was precisely, the basis for Buenaobra's arrest. It is to beg the question, I respectfully submit, to approve the military's action for the reason that Buenaobra confessed, because Buenaobra confessed for the reason that the military, precisely, pounced on him. I am not to be mistaken for prejudging Buenaobra's innocence (although it is supposed to be presumed) but I can not imagine that Buenaobra would have voluntarily proclaimed to the military that he was an NPA courier so that the military could pounce on him. I respectfully submit that the cases Garcia vs. Padilla 26 and Ilagan vs. Enrile 27 have been better days. I do not see how this court can continuously sustain them "where national security and stability are still directly challenged perhaps with greater vigor from the communist rebels." 28 First and foremost, and as the majority has conceded, we do not know if we are in fact dealing with "Communists." The case of Deogracias Espiritu, for one, hardly involves subversion. Second, "Communism" and "national security" are old hat the dictator's own excuses to perpetuate tyranny, and I am genuinely disappointed that we would still fall for old excuses.

Third, Garcia and Ilagan rested on supposed grounds that can not be possibly justified in a regime that respects the rule of law that the Presidential Commitment Order (PCO) is a valid presidential document (Garcia) and that the filing of an information cures a defective arrest (Ilagan). Fourth and finally, it is evident that neither "Communist threat" nor "national security" are valid grounds for warrantless arrests under Section 5(b) of Rule 113. I most respectfully submit that Garcia and Ilagan have not only been diluted by subsequent jurisprudence (e.g., People vs. Burgos, supra), they are relics of authoritarian rule that can no longer be defended, if they could have been defended, in Plaza Miranda or before our own peers in the bar. "What is important," says the majority, "is that every arrest without warrant be tested as to its legality, via habeas corpus proceedings." 29 I supposed that goes without saying. But it is also to patronize the petitioners and simply, to offer a small consolation, when after all, this Court is validating their continued detention. 30 With all due respect, I submit that it is nothing for which the public should be elated. A Final Word As I began my dissent, in this Resolution and the Decision sought to be reconsidered, I reiterate one principle: The State has no right to bother citizens without infringing their right against arbitrary State action. "The right of the people," states the Constitution, "to be secure in their persons, houses, papers, and effects against unreasonable searchers and seizures of whatever nature and for any purpose shall be inviolable . . . ." 31 "The State," the Charter likewise states, "values the dignity of every human person and guarantees full respect for human rights."32 The Constitution states the general rule the majority would make the exception the rule, and the rule the exception. With all due respect, this is not what constitutionalism is all about. I submit that the "actual facts and circumstances" the majority refers to are, in the first place, doubtful, the "actual facts and circumstances" being no more than "confidential information" (manufactured or genuine, we have no way of telling) and in the second place, any information with which the military (or police) were armed could no more than be hearsay, not personal, information. I submit that the "actual facts and circumstances" the majority insists on can not justify the arrests in question under Section 5(b) of Rule 113, the rule the majority insists is the applicable rule. Apparently, Section 5(b) is not the applicable rule, as far as Deogracias Espiritu and Narciso Nazareno are concerned; certainly, it is not the Section 5(b) I know. As I indicated, Espiritu was arrested one day after the act, allegedly, inciting to sedition; Nazareno was picked up fourteen days after it (allegedly, murder). Yet, the majority would approve the police's actions nonetheless because the police supposedly "found out only later." I submit that the majority has read into Section 5(b) a provision that has not been written there. "More than the allure of popularity of palatability to some groups," concludes the majority, "what is important is that the Court be right." 33 Nobody has suggested in the first place, that Umil was and is a question of popularity or palatability. Umil is a question, on the contrary, of whether or not the military (or police), in effecting the arrests assailed, had complied with the requirements of law on warrantless arrests. Umil is a question of whether or not this Court, in approving the military's actions, is right. In spite of "EDSA", a climate of fear persists in the country, as incidences of disappearances, torture, hamletting, bombings, saturation drives, and various human rights violations increase in alarming rates. In its update for October, 1990, the Task Force Detainees of the Philippines found:

An average of 209 arrested for political reasons monthly since 1988, 94% of them illegally; Four thousand four hundred eight (4,408) political detentions from January, 1989 to September, 1990, 4,419, illegally; Of those arrested, 535 showed signs of torture; 280 were eventually salvaged, 40, of frustrated salvage, and 109 remained missing after their arrest; Forty (40) cases of massacres, with 218 killed; 54 cases of frustrated massacre, in which 157 were wounded; The victims belonged to neighborhood and union organizations; Since February, 1986, 532 of those illegally arrested were women; From January to June 1990, 361 children were detained for no apparent reason; One million ten thousand four hundred nine (1,010,409) have been injured as a consequence of bombing, shellings, and food blockades undertaken by the military since 1988. 34 It is a bleak picture, and I am disturbed that this Court should express very little concern. I am also disappointed that it is the portrait of the Court I am soon leaving. Nonetheless, I am hopeful that despite my departure, it will not be too late. Motions denied.

David v. Arroyo, 489 SCRA 160 I. THE FACTS On February 24, 2006, as the Filipino nation celebrated the 20th Anniversary of the EDSA People Power I, President Arroyo issued PP 1017, implemented by G.O. No. 5, declaring a state of national emergency, thus: NOW, THEREFORE, I, Gloria Macapagal-Arroyo, President of the Republic of the Philippines and Commander-in-Chief of the Armed Forces of the Philippines, by virtue of the powers vested upon me by Section 18, Article 7 of the Philippine Constitution which states that: The President. . . whenever it becomes necessary, . . . may call out (the) armed forces to prevent or suppress. . .rebellion. . ., and in my capacity as their Commander-in-Chief, do hereby command the Armed Forces of the Philippines, to maintain law and order throughout the Philippines, prevent or suppress all forms of lawless violence as well as any act of insurrection or rebellion and to enforce obedience to all the laws and to all decrees, orders and regulations promulgated by me personally or upon my direction; and as provided in Section 17, Article 12 of the Constitution do hereby declare a State of National Emergency. In their presentation of the factual bases of PP 1017 and G.O. No. 5, respondents stated that the proximate cause behind the executive issuances was the conspiracy among some military officers, leftist insurgents of the New Peoples Army, and some members of the political opposition in a plot to unseat or assassinate President Arroyo. They considered the aim to oust or assassinate the President and take-over the reins of government as a clear and present danger. Petitioners David and Llamas were arrested without warrants on February 24, 2006 on their way to EDSA. Meanwhile, the offices of the newspaper Daily Tribune, which was perceived to be anti-Arroyo, was searched without warrant at about 1:00 A.M. on February 25, 2006. Seized from the premises in the absence of any official of the Daily Tribune except the security guard of the building were several materials for publication. The law enforcers, a

composite team of PNP and AFP officers, cited as basis of the warrantless arrests and the warrantless search and seizure was Presidential Proclamation 1017 issued by then President Gloria Macapagal-Arroyo in the exercise of her constitutional power to call out the Armed Forces of the Philippines to prevent or suppress lawless violence. I. II. THE ISSUE 1. Were the warrantless arrests of petitioners David, et al., made pursuant to PP 1017, valid? 2. Was the warrantless search and seizure on the Daily Tribunes officesconducted pursuant to PP 1017 valid?

Pollo v. Constantino-David, G.R. No. 181881, Oct. 18, 2011 OConnor v. Ortega, 480 U. S. 709 (1987) VILLARAMA, JR., J.: THE FACTS [This case involves a search of office computer assigned to a government employee who was then charged administratively and was eventually dismissed from the service. The employees personal files stored in the computer were used by the government employer as evidence of his misconduct.] On January 3, 2007, an anonymous letter-complaint was received by the respondent Civil Service Commission (CSC) Chairperson alleging that the chief of the Mamamayan muna hindi mamaya na division of Civil Service Commission Regional Office No. IV (CSC-ROIV) has been lawyering for public officials with pending cases in the CSC. Chairperson David immediately formed a team with background in information technology and issued a memorandum directing them to back up all the files in the computers found in the [CSC-ROIV] Mamamayan Muna (PALD) and Legal divisions. The team proceeded at once to the CSC-ROIV office and backed up all files in the hard disk of computers at the Public Assistance and Liaison Division (PALD) and the Legal Services Division. This was witnessed by several employees. At around 10:00 p.m. of the same day, the investigating team finished their task. The next day, all the computers in the PALD were sealed and secured. The diskettes containing the back-up files sourced from the hard disk of PALD and LSD computers were then turned over to Chairperson David. It was found that most of the files in the 17 diskettes containing files copied from the computer assigned to and being used by the petitioner, numbering about 40 to 42 documents, were draft pleadings or letters in connection with administrative cases in the CSC and other tribunals. Chairperson David thus issued a Show-Cause Order requiring the petitioner to submit his explanation or counter-affidavit within five days from notice. Petitioner filed his Comment, denying that he is the person referred to in the anonymous letter-complaint. He asserted that he had protested the unlawful taking of his computer done while he was on leave, citing the letter dated January 8, 2007 in which he informed Director Castillo of CSC-ROIV that the files in his computer were his personal files and those of his sister, relatives, friends and some associates and that he is not authorizing their sealing, copying, duplicating and printing as these would violate his constitutional right to privacy and protection against self-incrimination and warrantless search and seizure. He pointed out that though government property, the temporary use and ownership of the computer issued under a Memorandum of Receipt is ceded to the employee who may exercise all attributes of ownership, including its use for personal purposes. In view of the illegal search, the files/documents copied from his computer without his consent [are] thus inadmissible as evidence, being fruits of a poisonous tree. The CSC found prima facie case against the petitioner and charged him withDishonesty, Grave Misconduct, Conduct Prejudicial to the Best Interest of the Service and Violation of R.A. No. 6713 (Code of Conduct and Ethical Standards for Public Officials and Employees). Petitioner then filed an Omnibus Motion (For Reconsideration, to Dismiss and/or to Defer) assailing the formal charge as without basis having proceeded from an illegal search, which is beyond the authority of the CSC Chairman, such power pertaining solely to the court. The CSC denied this omnibus motion. On March 14, 2007, petitioner filed an Urgent Petition before the Court of Appeals (CA) assailing both the January 11, 2007

III. THE RULING [The Court partially GRANTED the petitions.] 1. NO, the warrantless arrests of petitioners David, et al., made pursuant to PP 1017, were NOT valid. [S]earches, seizures and arrests are normally unreasonable unless authorized by a validly issued search warrant or warrant of arrest. Section 5, Rule 113 of the Revised Rules on Criminal Procedure provides [for the following circumstances of valid warrantless arrests]: Sec. 5. Arrest without warrant; when lawful. - A peace officer or a private person may, without a warrant, arrest a person: (a) When, in his presence, the person to be arrested has committed, is actually committing, or is attempting to commit an offense. (b) When an offense has just been committed and he has probable cause to believe based on personal knowledge of facts or circumstances that the person to be arrested has committed it; and x x x. Neither of the [provisions on in flagrante nor hot pursuit warrantless arrests+ justifies petitioner Davids warrantless arrest. During the inquest for the charges of inciting to sedition and violation of BP 880, all that the arresting officers could invoke was their observation that some rallyists were wearing t-shirts with the invective Oust Gloria Nowand their erroneous assumption that petitioner David was the leader of the rally.Consequently, the Inquest Prosecutor ordered his immediate release on the ground of insufficiency of evidence. He noted that petitioner David was not wearing the subject t-shirt and even if he was wearing it, such fact is insufficient to charge him with inciting to sedition. 2. NO, the warrantless search and seizure on the Daily Tribunes officesconducted pursuant to PP 1017 was NOT valid. [T]he search [and seizure in the Daily Tribune premises] is illegal. Rule 126 of The Revised Rules on Criminal Procedure lays down the steps in the conduct of search and seizure. Section 4 requires that a search warrant be issued upon probable cause in connection with one specific offence to be determined personally by the judge after examination under oath or affirmation of the complainant and the witnesses he may produce. Section 8 mandates that the search of a house, room, or any other premise be made in the presence of the lawful occupant thereof or any member of his family or in the absence of the latter, in the presence of two (2) witnesses of sufficient age and discretion residing in the same locality. And Section 9 states that the warrant must direct that it be served in the daytime, unless the property is on the person or in the place ordered to be searched, in which case a direction may be inserted that it be served at any time of the day or night. All these rules were violated by the CIDG operatives. UPDATES

Show-Cause Order and February 26, 2007 Resolution as having been issued with grave abuse of discretion amounting to excess or total absence of jurisdiction. On July 24, 2007, the CSC issued a Resolution finding petitioner GUILTY of Dishonesty, Grave Misconduct, Conduct Prejudicial to the Best Interest of the Service and Violation of Republic Act 6713. He is meted the penalty of DISMISSAL FROM THE SERVICE with all its accessory penalties. This Resolution was also brought to the CA by herein petitioner. By a Decision dated October 11, 2007, the CA dismissed the petitioners petition for certiorari after finding no grave abuse of discretion committed by respondents CSC officials. His motion for reconsideration having been denied by the CA, petitioner brought this appeal before the Supreme Court. II. THE ISSUE Was the search conducted on petitioners office computer and the copying of his personal files without his knowledge and consent alleged as a transgression on his constitutional right to privacy lawful? III. THE RULING [The Supreme Court DENIED the petition and AFFIRMED the CA, which in turn upheld the CSC resolution dismissing the petitioner from service. The High Tribunal held that the search on petitioners office computer and the copying of his personal files were both LAWFUL and DID NOT VIOLATE his constitutional right to privacy.] The right to privacy has been accorded recognition in this jurisdiction as a facet of the right protected by the guarantee against unreasonable search and seizure under Section 2, Article III of the 1987 Constitution. The constitutional guarantee is not a prohibition of all searches and seizures but only of unreasonable searches and seizures. [The Supreme Court then discussed the American cases that served as jurisprudential bases for its ruling: That the Fourth Amendment [of the U.S. Constitution] equally applies to a government workplace was addressed in the 1987 case of OConnor v. Ortega.In OConnor the [U.S. Supreme] Court recognized that special needs authorize warrantless searches involving public employees for work-related reasons. The [U.S. Supreme] Court thus laid down a balancing test under which government interests are weighed against the employees reasonable expectation of privacy. This reasonableness test implicates neither probable cause nor the warrant requirement, which are related to law enforcement. OConnor was applied in subsequent cases raising issues on employees privacy rights in the workplace. One of these cases involved a government employers search of an office computer, United States v. Mark L. Simons where the defendant Simons, an employee of a division of the Central Intelligence Agency (CIA), was convicted of receiving and possessing materials containing child pornography. In this case, the US Supreme Court held that the search remains valid under the OConnor exception to the warrant requirement because evidence of the crime was discovered in the course of an otherwise proper administrative inspection. Simons violation of the agencys Internet policy happened also to be a violation of criminal law; this does not mean that said employer lost the capacity and interests of an employer. The warrantless entry into Simons office was reasonable under the Fourth Amendment standard announced in OConnor because at the inception of the search, the employer had reasonable grounds for suspecting that the hard drive would yield evidence of misconduct, as the employer was already aware that Simons had misused his Internet access to download over a thousand pornographic images. The retrieval of

the hard drive was reasonably related to the objective of the search, and the search was not excessively intrusive. Thus, while Simons had a reasonable expectation of privacy in his office, he did not have such legitimate expectation of privacy with regard to the files in his computer.] Applying the analysis and principles announced in OConnor and Simons to the case at bar, we now address the following questions: (1) Did petitioner have a reasonable expectation of privacy in his office and computer files?; and (2) Was the search authorized by the CSC Chair, [which involved] the copying of the contents of the hard drive on petitioners computer, reasonable in its inception and scope? (1) NO, the petitioner had no reasonable expectation of privacy in his office and computer files. Petitioner failed to prove that he had an actual (subjective) expectation of privacy either in his office or government-issued computer which contained his personal files. Petitioner did not allege that he had a separate enclosed office which he did not share with anyone, or that his office was always locked and not open to other employees or visitors. Neither did he allege that he used passwords or adopted any means to prevent other employees from accessing his computer files. On the contrary, he submits that being in the public assistance office of the CSC-ROIV, he normally would have visitors in his office like friends, associates and even unknown people, whom he even allowed to use his computer which to him seemed a trivial request. He described his office as full of people, his friends, unknown people and that in the past 22 years he had been discharging his functions at the PALD, he is personally assisting incoming clients, receiving documents, drafting cases on appeals, in charge of accomplishment report, Mamamayan Muna Program, Public Sector Unionism, Correction of name, accreditation of service, and hardly had any time for himself alone, that in fact he stays in the office as a paying customer. Under this scenario, it can hardly be deduced that petitioner had such expectation of privacy that society would recognize as reasonable. Moreover, even assuming arguendo, in the absence of allegation or proof of the aforementioned factual circumstances, that petitioner had at least a subjective expectation of privacy in his computer as he claims, such is negated by the presence of policy regulating the use of office computers [CSC Office Memorandum No. 10, S. 2002 Computer Use Policy (CUP)+, as in Simons. The CSC in this case had implemented a policy that put its employees on notice that they have no expectation of privacy in anything they create, store, send or receive on the office computers, and that the CSC may monitor the use of the computer resources using both automated or human means. This implies that on-the-spot inspections may be done to ensure that the computer resources were used only for such legitimate business purposes. (2) YES, the search authorized by the respondent CSC Chair, which involved the copying of the contents of the hard drive on petitioners computer, was reasonable in its inception and scope. The search of petitioners computer files was conducted in connection with investigation of work-related misconduct prompted by an anonymous letter-complaint addressed to Chairperson David regarding anomalies in the CSC-ROIV where the head of the Mamamayan Muna Hindi Mamaya Na division is supposedly lawyering for individuals with pending cases in the CSC. A search by a government employer of an employees office is justified at inception when there are reasonable grounds for suspecting that it will turn up evidence that the employee is guilty of work-related misconduct. Under the facts obtaining, the search conducted on petitioners computer was justified at its inception and scope. We quote with approval the CSCs discussion on the reasonableness of

its actions, consistent as it were with the guidelines established by OConnor: Even conceding for a moment that there is no such administrative policy, there is no doubt in the mind of the Commission that the search of Pollos computer has successfully passed the test of reasonableness for warrantless searches in the workplace as enunciated in the above-discussed American authorities. It bears emphasis that the Commission pursued the search in its capacity as a government employer and that it was undertaken in connection with an investigation involving a workrelated misconduct, one of the circumstances exempted from the warrant requirement. At the inception of the search, a complaint was received recounting that a certain division chief in the CSCRO No. IV was lawyering for parties having pending cases with the said regional office or in the Commission. The nature of the imputation was serious, as it was grievously disturbing. If, indeed, a CSC employee was found to be furtively engaged in the practice of lawyering for parties with pending cases before the Commission would be a highly repugnant scenario, then such a case would have shattering repercussions. It would undeniably cast clouds of doubt upon the institutional integrity of the Commission as a quasi-judicial agency, and in the process, render it less effective in fulfilling its mandate as an impartial and objective dispenser of administrative justice. It is settled that a court or an administrative tribunal must not only be actually impartial but must be seen to be so, otherwise the general public would not have any trust and confidence in it. Considering the damaging nature of the accusation, the Commission had to act fast, if only to arrest or limit any possible adverse consequence or fall-out. Thus, on the same date that the complaint was received, a search was forthwith conducted involving the computer resources in the concerned regional office. That it was the computers that were subjected to the search was justified since these furnished the easiest means for an employee to encode and store documents. Indeed, the computers would be a likely starting point in ferreting out incriminating evidence. Concomitantly, the ephemeral nature of computer files, that is, they could easily be destroyed at a click of a button, necessitated drastic and immediate action. Pointedly, to impose the need to comply with the probable cause requirement would invariably defeat the purpose of the wok-related investigation. Thus, petitioners claim of violation of his constitutional right to privacy must necessarily fail. His other argument invoking the privacy of communication and correspondence under Section 3(1), Article III of the 1987 Constitution is also untenable considering the recognition accorded to certain legitimate intrusions into the privacy of employees in the government workplace under the aforecited authorities. We likewise find no merit in his contention that OConnor and Simons are not relevant because the present case does not involve a criminal offense like child pornography. As already mentioned, the search of petitioners computer was justified there being reasonable ground for suspecting that the files stored therein would yield incriminating evidence relevant to the investigation being conducted by CSC as government employer of such misconduct subject of the anonymous complaint. This situation clearly falls under the exception to the warrantless requirement in administrative searches defined in OConnor. 8. Privacy of Communications and Correspondence (Sec. 3 (1), Art. III) The Constitutional Right to Privacy CASES: Griswold v. Connecticut Brief Fact Summary. Appellants were charged with violating a statute preventing the distribution of advice to married couples regarding the prevention of conception. Appellants claimed that the statute violated the 14th Amendment to the United States Constitution.

Synopsis of Rule of Law. The right of a married couple to privacy is protected by the Constitution. Facts. Appellant Griswold, Executive Director of the Planned Parenthood League of Connecticut and Appellant Buxton, a licensed physician who served as Medical Director for the League at its Center in New Haven, were arrested and charged with giving information, instruction, and medical advice to married persons on means of preventing conception. Appellants were found guilty as accessories and fined $100 each. Appellants appealed on the theory that the accessory statute as applied violated the 14th Amendment to the United States Constitution. Appellants claimed standing based on their professional relationship with the married people they advised. Issue. Does the Constitution provide for a privacy right for married couples? Held. The First Amendment has a penumbra where privacy is protected from governmental intrusion, which although not expressly included in the Amendment, is necessary to make the express guarantees meaningful. The association of marriage is a privacy right older than the Bill of Rights, and the States effort to control marital activities in this case is unnecessarily broad and therefore impinges on protected Constitutional freedoms. Dissent. Justice Stewart and Justice Black. Although the law is silly, it is not unconstitutional. The citizens of Connecticut should use their rights under the 9th and 10th Amendment to convince their elected representatives to repeal it if the law does not conform to their community standards. Concurrence. Justice Goldberg, the Chief Justice, and Justice Brennan. The right to privacy in marriage is so basic and fundamental that to allow it to be infringed because it is not specifically addressed in the first eight amendments is to give the 9th Amendment no effect. Justice Harlan. The relevant statute violates the Due Process Clause of the 14th Amendment because if violates the basic values implicit in the concept of ordered liberty. Discussion. The right to privacy in marriage is not specifically protected in either the Bill of Rights or the Constitution. Nonetheless, it is a right so firmly rooted in tradition that its protection is mandated by various Constitutional Amendments, including the 1st, 9th and 14th Amendments. Morfe v. Motuc, citing with approval Griswold Facts:

The Law: Anti-Graft and Corrupt Practices Act of 1960 (RA No. 3019)

Every public officer within 30 days after its approval or after his assumption of office and within the month of January of every year thereafter, as well as upon termination of his position, shall prepare and file with the head of the office to which he belongs, a true detailed and sworn statement of assets and liabilities, including a statement of the amounts and sources of his income, the amounts of his personal and family expenses

and the amount of income taxes paid for the next preceding calendar year.

inherent and plenary power in the state which enables it to prohibit all things hurtful to the comfort, safety and welfare of society (Justice Malcolm)

Plaintiff Morfe, a judge of a CFI, contends that the periodical submission within the month of January of every other year thereafter of their sworn statement of assets and liabilities (SAL) is violative of due process as an oppressive exercise of police power and as an unlawful invasion of the constitutional right to privacy implicit on the ban against unreasonable search and seizure construed together with the prohibition against self-incrimination.

The power of sovereignty, the power to govern men and things within the limits of its domain (Justice Taney, going beyond curtailment of rights)

Anyone with an alleged grievance regarding the extension of police power to regulatory action affecting persons in public or private life can invoke the protection of due process.

It has been held that due process may be relied upon by public official to protect the security of tenure which in a limited sense is analogous to property. Therefore he could also use due process to strike down what he considers as an infringement of his liberty.

Executive Secretary and DOJ Sec:

Acceptance of public position = voluntary assumption of obligation Merely seeks to adopt a reasonable measure of insuring the interest of general welfare in honest and clean public service and is therefore a legitimate exercise of police power.

Under the Constitution, the challenged provision is allowable as long as due process is observed. The standard for due process is REASONABLENESS. Test: Official action must not outrun the bounds of reason and result in sheer oppression.

CFI of Pangasinan held that the requirement exceeds the permissible limit of the police power and is thus offensive to the due process clause

It would be to dwell in the realm of abstractions and to ignore the harsh and compelling realities of public service with its ever-present temptation to heed the call of greed and avarice to condemn as arbitrary and oppressive a requirement as that

Issue/s:

imposed upon public officials and employees to file such sworn statement of assets and liabilities every two years after having done so upon assuming officeThere was therefore no unconstitutional exercise of police power. 1. Right to privacy Right to be let alone It cannot be said that the challenged statutory provision calls for disclosure of information which infringes on the right of a person to privacy. It cannot be denied that the rational relationship such a requirement possesses with the objective

Whether the periodical submission of SAL for public officers is: 1. An oppressive exercise of police power; 2. Violative of due process and an unlawful invasion of the right to privacy implicit in the ban against unreasonable search and seizure construed together with the prohibition against self-incrimination; 3. An insult to the personal integrity and official dignity of public officials.

Ruling: Decision reversed.

of a valid statute goes very far in precluding assent to an objection of such character. This is not to say that a public officer, by virtue of position he holds, is bereft of constitutional protection; it is only to emphasize that in

Ratio:

1.

Presumption of validity Plaintiff asserted that the submission of SAL was a reasonable requirement for employment so a public officer can make of record his assets and liabilities upon assumption of office. Plaintiff did not present evidence to rebut the presumption of validity.

subjecting him to such a further compulsory revelation of his assets and liabilities, including the statement of the amounts of personal and family expenses, and the amount of income taxes paid for the next preceding calendar year, there is no unconstitutional intrusion into what otherwise would be a private sphere. 1. Unreasonable Search and Seizure The constitutional guarantee against unreasonable search and seizure does not give freedom from testimonial compulsion. 1. Right against self-incrimination

If the liberty involved were freedom of the mind or the person, the standard for the validity of governmental acts is much more rigorous and exacting, but where the liberty curtailed affects the most rights of property, the permissible scope of regulatory measure is wider. (Ermita-Malate Hotel v. Mayor of Manila)

1.

Exercise of Police power and the defense provided by the Due Process Clause

We are not aware of any constitutional provision designed to protect a mans conduct from judicial inquiry, or aid him in fleeing from justice.

1.

Insult to personal integrity and official dignity Only congressional power or competence, not the wisdom of the action taken, mey be the basis for declaring a statute invalid.

system. It is plain and we hold that A.O. No. 308 falls short of assuring thatpersonal information which will be gathered about our people will onlybe processed for unequivocally specified purposes. The lack of propersafeguards in this regard of A.O. No. 308 may interfere with theindividual's liberty of abode and travel by enabling authorities to trackdown his movement; it may also enable unscrupulous persons toaccess confidential information and circumvent the right against selfincrimination; it may pave the way for "fishingexpeditions" bygovernment authorities and evade the right against unreasonablesearches and seizures. The possibilities of abuse and misuse of the PRN,biometrics and computer technology are accentuated when weconsider that the individual lacks control over what can be read orplaced on his ID, much less verify the correctness of the dataencoded. They threaten the very abuses that the Bill of Rights seeks toprevent.Excerpts from the concurring terrifying are the possibilities of a law such as Administrative OrderNo. 308 in making inroads into the private lives of the citizens, a virtualBig Brother looking over our shoulders, that it must, without delay, be"slain upon sight" before our society turns totalitarian with e Order No. 308 appears to be so extensively drawn thatcould, indeed, allow unbridled options to become available to itimplementors beyond the reasonable comfort of the citizens and of residents alike.RIGHT TO PRIVACY RECOGNIZED UNDER THE CONSTITUTIONHereunder are the provisions in the 1987 Constitution which recognize ourRight to Privacy :Section 3(1) of the Bill of Rights:"Sec. 3. (1) The privacy of communication and correspondence shall beinviolable except upon lawful order of the court, or when public safety ororder requires otherwise as prescribed by law."Other facets of the right to privacy are protected in various provisions of the Bill of Rights, viz: 34"Sec. 1. No person shall be deprived of life, liberty, or property without due process of law, nor shall any person be denied the equal protection of thelaws. Sec. 2. The right of the people to be secure in their persons, houses, papers, and effects against unreasonable searches and seizures of whatever nature and for any purpose shall be inviolable, and no searchwarrant or warrant of arrest shall issue except upon probable cause to bedetermined personally by the judge after examination under oath oraffirmation of the complainant and the witnesses he may produce, andparticularly describing the place to be searched and the persons or thingsto be seized.xxx xxx xxxSec. 6. The liberty of abode and of changing the same within the limitsprescribed by law shall not be impaired except upon lawful order of thecourt. Neither shall the right to travel be impaired except in the interest of national security, public safety, or public health, as may be provided bylaw.xxx xxx xxx.Sec. 8. The right of the people, including those employed in the publicand private sectors, to form unions, associations, or societies for purposesnot contrary to law shall not be abridged.Sec. 17. No person shall be compelled to be a witness against himself.Personal Analysis:A.O. 308 was declared unconstitutional by the Supreme Court en banc forreasons above stated. It bears stressing that the bulk of discussion in thecase focused more on the issue of infringement of the right to privacy. Ascan be gleaned from A.O. 308, the provisions were so general that therewere no clear and vital guidelines to safeguard the information stored inthe Identification Card. Had President Fidel V. Ramos issued a morecomplete and detailed guidelines providing for the metes and bounds of the ID System, the decision could have been otherwise.Even the argument of the respondents that rules and regulations wouldbe issued by the committee later, the court still reject the same. The courtsaid.: The rules and regulations to be drawn by the IACC cannot remedythis fatal defect. Rules and regulations merely implement the policy of thelaw or order. On its face, A.O. No. 308 gives the Inter-AgencyCoordinating Committee (IACC) virtually unfettered discretion todetermine the metes and bounds of the ID System.In one press conference last month, Presidential Spokesperson IgnacioBunye said that there is really no need to pass a law to push through withthe plan of the National ID System. An executive issuance by thePresident would suffice provided this time the said order will now bedetailed, comprehensive and contains all the vital safeguards. From hisstatement, it can be deduced therefrom that the reservation andbacklash by the supreme court on the on the Ople case (A.O 308) havebeen taken into consideration by Malacanang

Ople v. Torres, citing both Griswold and Morfe V. Ramos issued Administrative Order (A.O.)

filed a petition before the Supreme Courtquestioning the constitutionality of the said executive issuance ontwo important grounds, viz: one, it is a usurpation of the power of Congress to legislate, and two, it impermissibly intrudes on ourcitizenry's protected zone of privacy. We grant the petition for therights sought to be vindicated by the petitioner need strongerbarriers against further erosion.ISSUE: DOES A.0 308 VIOLATE THE RIGHT TO PRIVACY? SUPREME COURT:Yes. Assuming, arguendo, that A.O. No. 308 need not be the subject of a law, still it cannot pass constitutional muster as an administrativelegislation because facially it violates the right to privacy. A.O. 308 is sovague. The vagueness, the overbreadth of A.O. No. 308 which if implemented will put our people's right to privacy in clear and presentdanger. There are no vital safeguards.A.O. No. 308 should also raise our antennas for a further look will showthat it does not state whether encoding of data is limited to biologicalinformation alone for identification purposes. In fact, the SolicitorGeneral claims that the adoption of the Identification ReferenceSystem will contribute to the "generation of population data fordevelopment planning." 54 This is an admission that the PopulationReference Number (PRN) will not be used solely for identification butfor the generation of other data with remote relation to the avowedpurposes of A.O. No. 308. Clearly, the indefiniteness of A.O. No. 308can give the government the roving authority to store and retrieveinformation for a purpose other than the identification of the individualthrough his PRN .The potential for misuse of the data to be gathered under A.O. No. 308cannot be underplayed as the dissenters do. Pursuant to saidadministrative order, an individual must present his PRN everytime hedeals with a government agency to avail of basic services andsecurity. His transactions with the government agency will necessarilybe recorded -- whether it be in the computer or in the documentaryfile of the agency. The individual's file may include his transactions forloan availments, income tax returns, statement of assets and liabilities,reimbursements for medication, hospitalization, etc. The more frequentthe use of the PRN, the better the chance of building a huge andformidable information base through the electronic linkage of the files.The data may be gathered for gainful and useful governmentpurposes; but the existence of this vast reservoir of personal informationconstitutes a covert invitation to misuse, a temptation that may be toogreat for some of our authorities to resist.We can even grant, arguendo, that the computer data file will be limited to the name, address and other basic personal informationabout the individual. Even that hospitable assumption will not saveA.O. No. 308 from constitutional infirmity for again said order does nottell us in clear and categorical terms how these information gatheredshall be handled. It does not provide who shall control and access thedata, under what circumstances and for what purpose. These factorsare essential to safeguard the privacy and guaranty the integrity of theinformation. Well to note, the computer linkage gives othergovernment agencies access to the information. Yet, there are nocontrols to guard against leakage of information. When the accesscode of the control programs of the particular computer system isbroken, an intruder, without fear of sanction or penalty, can make useof the data for whatever purpose, or worse, manipulate the datastored within the

CASE: Facts: y

Read also RA 4200, the Anti-Wiretapping Act Gaanan v. IAC, 145 SCRA 112

crime might hesitate to inform police authorities if he knows that he could be accusedunder Rep. Act 4200 of using his own telephone to secretly overhear the private communications of the would becriminals. Surely the law was never intended for such mischievous results. y Telephones or extension telephones are not included in the enumeration of "commonly known" listening or recordingdevices, nor do they belong to the same class of enumerated electronic devices contemplated by law. Telephoneparty lines were intentionally deleted from the provisions of the Act. y W hat the law refers to is a "tap" of a wire or cable or the use of a "device or arrangement" for the purpose of secretlyoverhearing, intercepting, or recording the communication. An extension telephone cannot be placed in the samecategory as a dictaphone, dictagraph or the other devices enumerated in Section 1 of RA 4200 . There must be eithera physical interruption through a wiretap or the deliberate installation of a device or arrangement in order tooverhear, intercept, or record the spoken words. y I n statutory construction, in order to determine the true intent of the legislature, the particular clauses and phrases of the statute should not be taken as detached and isolated expressions, but the whole and every part thereof must beconsidered in fixing the meaning of any of its parts. Hence, the phrase "device or arrangement" in Section 1 of RA4200, should be construed to comprehend instruments of the same or similar nature , that is, instruments the use of which would be tantamount to tapping the main line of a telephone. I t refers to instruments whose installation orpresence cannot be presumed by the party or parties being overheard because, by their very nature, they are not of common usage and their purpose is precisely for tapping, intercepting or recording a telephone conversation. y The court also ruled that the conduct of the party would differ in no way if instead of repeating the message he heldout his hand-set so that another could hear out of it and that there is no distinction between that sort of action andpermitting an outsider to use an extension telephone for the same purpose. y F urthermore, it is a general rule that penal statutes must be construed strictly in favor of the accused. Thus, in case of doubt as in the case at bar, on whether or not an extension telephone is included in the phrase "device orarrangement", the penal statute must be construed as not including an extension telephone as ruled in PP vs.Purisima y W herefore, the petition is granted, decision of the I AC is annulled and set aside and petitioner is acquitted for thecrime of violating RA 4200. Jurisprudence

In the morning of October 22, 1975, complainant Atty. Tito Pintor and his client Manuel Montebon were in the livingroom of complainant's residence discussing the terms for the withdrawal of the complaint for direct assault

That same morning, Laconico, another lawyer, telephoned the appellant to come to his office and advise him on thesettlement of the direct assault case because his regular lawyer, Atty. Leon Gonzaga, went on a business trip.

When complainant called up, Laconico requested appellant to secretly listen to the telephone conversation through a telephone extension so as to hear personally the proposed conditions for the settlement y Twenty minutes later, complainant called up again to ask Laconico if he was agreeable to the conditions, which the latter answered in affirmative. Complainant then told Laconico to wait for instructions on where to deliver the money, he told Laconico to give the money to his wife but the latter insisted that complainant himself should receive the money. And when he received the money at a restaurant, complainant was arrested by agents of the Philippine Constabulary. y Appellant Laconico executed on the following day an affidavit stating that he heard complainant demand P8,000.00for the withdrawal of the case for direct assault. Complainant then charged Laconico with violation of RA 4200 forlistening to the telephone conversation without complainant's consent. y The lower court found both Gaanan and Laconico guilty of violating Section 1 of Republic Act No. 4200. The two wereeach sentenced to one (1) year imprisonment with costs y The I ntermediate Appellate Court affirmed the decision of the trial court, holding that the communication betweenthe omplainant and Laconico was private in nature therefore was covered by RA 4200; and that the petitioneroverheard such communication without the knowledge and consent of the complainant; and that the extensiontelephone which was used by the petitioner to overhear the telephone conversation between complainant andLaconico is covered in the term "device' as provided in Rep. Act No. 4200. Issue: WON extension telephone is among the prohibited devices in Section 1 of the Act, such that its use to overhear a privateconversation would constitute unlawful interception of communications between the two parties using a telephone line. Held: No.Ruling: y Our lawmakers intended to discourage, through punishment, persons such as government authorities orrepresentatives of organized groups from installing devices in order to gather evidence for use in court or tointimidate, blackmail or gain some unwarranted advantage over the telephone users. Consequently, the mere act of listening, in order to be punishable must strictly be with the use of the enumerated devices in RA No. 4200 or othersof similar nature. We are of the view that an extension telephone is not among such devices or arrangements

CASES: Olmstead v. U.S., 227 U.S. 438 (read also the dissents of Holmes and Brandeis, JJ.) Background information Until 1914, the American judicial system, including the Supreme Court of the United States, largely followed the precepts of English common law when it came to matters dealing with the validity of introducing evidence in criminal trials. In most cases, the general philosophy was that the process by which the evidence was obtained had little, if anything to do with the permissibility of its use in court. The only limiting factor was that the police agents could not break the law to seize the evidence; however, since what is now illegal seizure was then permitted by the courts, it rarely presented a significant challenge.

y There is no question that the telephone conversation between complainant Atty. Pintor and accused Atty. Laconicowas "private" made between one person and another as distinguished from words between a speaker and a public;the affirmance of the criminal conviction would, in effect, mean that a caller by merely using a telephone line canforce the listener to secrecy no matter how obscene, criminal, or annoying the call may be. I t would be the word of the caller against the listener's. Such that ". An unwary citizzen who happens to pick up his telephone and whooverhears the details of a

In 1914, however, in the landmark case of Weeks v. United States, the Court held unanimously that illegal seizure of items from a private residence was a violation of the Fourth Amendment, and established the exclusionary rule that prohibits admission of illegally obtained evidence in federal courts. It is important to underline that because the Bill of Rights did not at the time extend to cover states, such a prohibition extended only to federal agents, and covered only federal trials. It was not until the case of Mapp v. Ohio (1961) that the exclusionary rule was extended to state courts as well. The question here, then, was whether the recordings of wiretapped private telephone conversations constituted impermissibly-seized evidence and thus constituted a violation of the federal exclusionary rule. Case details The case concerned several petitioners, including Roy Olmstead, who challenged their convictions, arguing that the use of evidence of wiretapped private telephone conversations amounted to a violation of the Fourth and Fifth Amendments. The petitioners were convicted for alleged conspiracy to violate the National Prohibition Act by unlawfully possessing, transporting, and selling alcohol. Seventy-two additional persons, aside from the petitioners, were indicted. The evidence provided by the wiretapped telephone conversations disclosed "a conspiracy of amazing magnitude" to engage in bootlegging, involving the employment of some fifty persons, use of sea vessels for transportation, an underground storage facility in Seattle, and the maintenance of a central office fully equipped with executives, bookkeepers, salesmen, and an attorney. According to the record, even in a bad month, the sales amounted to some $176,000; the grand total for a year probably came out to some $2 million (or about $26.8 million in [1] today's dollars). . Olmstead was the general manager of this business, receiving fifty percent of all the profits. The information that led to the discovery of his involvement and the conspiracy itself was largely obtained by four federal prohibition officers who were able to intercept messages on his, and other conspirators' telephones. No laws were violated in installing the wiretapping equipment, as the officers did not trespass upon either the homes or the offices of the defendants; instead, the equipment was placed in the streets near the houses and in the basement of the large office building. The wiretapping went on for several months, and the records revealed significant details on the business transactions of the petitioners and their employees. Stenographic notes were made of the conversations, and their accuracy was testified to by the government witnesses. The evidence disclosed full details of the operations of the bootlegging business; in addition, it showed the relationship between Olmstead with members of the Seattle police, which resulted in prompt release of some of the arrested members of the conspiracy and promises to officers of payment. MR. JUSTICE HOLMES: My brother BRANDEIS has given this case so exhaustive an examination that I desire to add but a few words. While I do not deny it, I am not prepared to say that the penumbra of the Fourth and Fifth Amendments covers the defendant, although I fully agree that Courts are apt to err by sticking too closely to the words of a law where those words import a policy that goes beyond them. Gooch v. Oregon Short line R.R. Co., 258 U. S. 22, 258 U. S. 24. But I think, as MR. JUSTICE BRANDEIS says, that, apart from the Constitution, the Government ought not to use Page 277 U. S. 470 evidence obtained and only obtainable by a criminal act. There is no body of precedents by which we are bound, and which confines us to logical deduction from established rules. Therefore we must consider the two objects of desire, both of which we cannot have, and make up our minds which to choose. It is desirable that

criminals should be detected, and, to that end, that all available evidence should be used. It also is desirable that the Government should not itself foster and pay for other crime, when they are the means by which the evidence is to be obtained. If it pays its officers for having got evidence by crime I do not see why it may not as well pay them for getting it in the same way, and I can attach no importance to protestations of disapproval if it knowingly accepts and pays and announces that, in future it will pay for the fruits. We have to choose, and, for my part, I think it a less evil that some criminals should escape than that the Government should play an ignoble part. For those who agree with me, no distinction can be taken between the Government as prosecutor and the Government as judge. If the existing code does not permit district attorneys to have a hand in such dirty business, it does not permit the judge to allow such iniquities to succeed. See Silverthorne Lumber Co. v. United States, 251 U. S. 385. And if all that I have said so far be accepted, it makes no difference that, in this case, wiretapping is made a crime by the law of the State, not by the law of the United States. It is true that a State cannot make rules of evidence for Courts of the United States, but the State has authority over the conduct in question, and I hardly think that the United States would appear to greater advantage when paying for an odious crime against State law than when inciting to the disregard of its own. I am aware of the often repeated statement that, in a criminal proceeding, the Court will not take notice of the manner in which papers offered in evidence have been Page 277 U. S. 471 obtained. But that somewhat rudimentary mode of disposing of the question has been overthrown by Weeks v. United States, 232 U. S. 383, and the cases that have followed it. I have said that we are free to choose between two principles of policy. But if we are to confine ourselves to precedent and logic, the reason for excluding evidence obtained by violating the Constitution seems to me logically to lead to excluding evidence obtained by a crime of the officers of the law. Goldman v. U.S., 316 U.S. 29 (ditto) Katz v. U.S., 389 U.S. 347 (read also the dissent of Justice Black) In his dissent, Justice Hugo Black argued that the Fourth Amendment, as a whole, was only meant to protect "things" from physical search and seizure; it was not meant to protect personal privacy. Additionally, Black argued that the modern act of wiretapping was analogous to the act of eavesdropping, which was around even when the Bill of Rights was drafted. Black concluded that if the drafters of the Fourth Amendment had meant for it to protect against eavesdropping they would have included the proper language.

Anda mungkin juga menyukai